Anda di halaman 1dari 194

Phng Phap Dch VIET ANH - NG PHAP

LI M AU
ay la lan th t quyen sach nay c tai ban.
lan tai ban nay sach c viet lai, chnh ly va bo
sung ay u va ro rang hn.

Muc ch cua quyen sach nay nham cung cap cho


ban nhng nguyen tac c ban e dch cau t tieng
Viet sang tieng Anh. Song, vi mot he thong cac
nguyen tac va cac bai hoc, sach se giup cho cac ban
co kha nang dch tng oan van, tuy vao von t
vng cua cac ban. Sach nay chu yeu gom 15 bai hoc
chnh, tng bc giup cac ban luyen dch.

e m au, cac ban phai nam cau truc cua 7


Mau C Ban (the basic sentence pattern). Cac bai hoc ke
tiep noi ve yeu - to - mo - ta (modifiers) se giup cac
ban biet cach m rong cac Mau C Ban e co the
dien at y tng ro rang, chnh xac hn. Tng bc,
cac ban se lam quen vi cac hnh thc cua cac yeu -
to - mo - ta.

Trong tieng Anh, yeu to nao co chc nang mo ta


hay bo ngha cho mot danh t th yeu to o c coi nh
tnh t (the adjective) hoac tng ng vi tnh t (the
adjective equivalent). Yeu - to - mo - ta cho ong t, tnh
t hoac trang t th yeu to o c coi nh trang t
(the adverb). Adjective va Adverb la hai yeu - to - mo - ta
quan trong.

Cac yeu - to - mo - ta di dang mot cum t (the


phrase) c he thong hoa theo hnh thc cau tao cua
chung ta va gom co:

P-group, V-ing group, V-ed group, To-group, S-group. P la


ch au cua t Proposition, V la ch au cua t
Verb, To la dau hieu cua ong t nguyen mau, S la
ch au cua t Subordinator.

Le Th Thuy Loan Trang 1


Phng Phap Dch VIET ANH - NG PHAP

Hai bai hoc tiep theo la hai qui tac quan trong trong
ng phap tieng Anh: Qui tac tng can (the Rule of
Proximity) va Qui tac song hanh (the Rule of Parallelism).

Qui tac tng can giup cac ban xep at cac yeu -
to - mo - ta vao ung v tr cua chung trong cau noi.
Qui tac song hanh giup cac ban can bang cac yeu - to
- mo - ta va cac yeu to co chc nang khac trong
cau noi.

Hai bai hoc ve Phrace, Clause va Sentence giup cac


ban biet cach phan biet, s dung cac loai menh e
va cac loai cau trong tieng Anh.

Bai hoc ve Phng phap hnh o (Diagraming


sentence) giup cac ban co cai nhn cu the hn trong viec
phan tch mot cau tieng Anh khi ta trnh bay cac yeu to
co chc nang ng phap len tren mot hnh o (a
diagram).

Chuc cac ban thanh cong.

Mua he nam 2000


SOAN GIA

PHNG PHAP

Khi dch mot cau t tieng Viet sang tieng Anh, sau
khi a chon mot th thch hp cho ong t, ta theo cac
trnh t sau ay:

Le Th Thuy Loan Trang 2


Phng Phap Dch VIET ANH - NG PHAP

Bc 1: Chon Mau C Ban


Bc 2: Chon yeu - to - mo - ta (hoac mot
cau truc chc nang) thch hp.
Bc 3: Ap dung Luat tng can hoac Luat
song hanh.

Th du 1:
Phong trao chong chien tranh hat nhan ang lan rong
khap TG.

Bc 1: Chon Pattern 1: Subject x Verb


The movement is spreading.
Bc 2: Chon yeu to mo ta:
chong chien tranh hat nhan
P-group: against nuclear war
khap the gii:
P-group: throughout the world.
Bc 3: Ap dung Luat tng can

The against
is spreading
movement nuclear war

throughout
the world

Th du 2:
Ve tinh c a len khong gian hom qua la mot
ve tinh kh tng.

Basic sentence:
Pattern 4: Subject x Be x Noun
The satellite was weather one.

Modifier:
c a len khong gian hom qua
V-ed group: sent into space yesterday.
Rule of Proximity:

Le Th Thuy Loan Trang 3


Phng Phap Dch VIET ANH - NG PHAP

The sent into space was a


setellite yesterday weather one.

Th du 3:
Chuyen banh cho Gomez, Figo nhay lui lai.

1. Basic sentence:
Pattern 1: Subject x Verb x Adverb
Figo dances back
2. Modifier:
chuyen banh cho Gomez
V-ing group: passing the ball to Gomez
3. Proximity:

Passing the ball Figo


to Gomez, dances back.

Th du 4:
Chnh phu a chap thuan e an xay dng mot phi
trng mi.

1. Basic sentence:
Pattern 5: Subject x Verb x Object
The Government has approved the project
2. Modifier:
Xay mot phi trng mi
To-group: to build a new airport
3. Proximity:
The Government has approved the project

to build a new
airport.
Th du 5:
Chc nang cua mot enzyme la ay manh cac tien
trnh hoa hoc.

1. Pattern 4: Subject x Be x Subject complement


The function of an enzyme is to speed up chemical
processes.
Le Th Thuy Loan Trang 4
Phng Phap Dch VIET ANH - NG PHAP

Th du 6:
Chung ta can mot ngi lanh ao co u thong
minh e quyet nh chnh xac, u can am e quyet
nh tao bao va u nhay ben e quyet nh nhanh
chong.

1.Basic sentence: Subject x Verb x Object


We need a leader
2.Modifiers: Cho 3 S-group i song hanh:

We need a leader who has intelligence enough to


make correct decisions,
who has courage enough to make
bold decisions, and
who has wit enough to make
rapid
decisions.

1. THE SEVEN BASIC SENTENCE PATTERNS


(BAY MAU C BAN)

e cat ngha nhng cach khac nhau ma cac t


ng trong tieng Anh co the phoi hp lai tao thanh cau
noi, ta phai biet phan biet gia hai loai cau khac nhau
trong tieng Anh. o la cau c ban (the basic sentence) va
cau chuyen hoa hay bien oi (the transformed sentence).
Hai loai cau nay c tao thanh bang nhng cach khac

Le Th Thuy Loan Trang 5


Phng Phap Dch VIET ANH - NG PHAP

nhau. Quan niem nh the e ta biet rang mot cau c


ban co the c bien oi thanh mot hoac nhieu dang
khac nhau, nhng chung cung mang mot y ngha.
phan nay ta ch xet cac Mau C Ban trong tieng Anh.

Moi cau noi tieng Anh gom hai phan: Chu ng (the
subject) va v ng (the predicate). Chu ng c goi ten
mot ngi, mot o vat hoac mot s kien va v ng
noi mot ieu g o ve chu ng. Chu ng luon luon
la mot danh t, ai t, mot cum t c dung nh
danh t (a noun clause). V ng trong cau noi luon luon
bat au bang mot ong t. Phan con lai cua cau
noi - ngha la phan nam trong v ng theo sau ong t
c goi la bo tuc t (the complement).

dang n gian nhat, predicate co khi ch dung


mot ong t hu han - tc la mot ong t a c
chia (a finite verb). Complement la yeu to (mot t hoac
cum t) giup hoan tat y ngha cua mot cum t.

hai bang di ay, bang au tien cho thay cau


noi c chia thanh hai phan: Subject va Predicate. Bang
th nh cho thay Predicate gom co Verb va Complement.

Sentence = Subject x Predicate


The dog barked
They were at the station
We are friends
Mary looks happy
Mr. Green visited Tokyo
He drove to town

Predicate = Verb x Complement


barked -
were at the station
are friends
looks happy
visited Tokyo

Le Th Thuy Loan Trang 6


Phng Phap Dch VIET ANH - NG PHAP

drove to town

Tat ca cac cau c ban gom co mot danh t c


theo sau bi ong t va complement. Nhng complement
lai gom nhieu t loai khac nhau. Cac cau c ban v the
thng c xep loai theo t loai cua complement. Hay
noi mot cach khac, ta xep loai Mau C Ban tuy theo
loai ong t c dung.

ong t co the la t ong t (intransitive verb), tha


ong t (transitive verb) hoac ong t ch trang thai
(state verb/state of being verb). Ta xep cac cau tieng
Anh vao bay mau sau ay:

PATTERN 1:

Noun Verb Adverbial


1 Everybody laughed.
2 The dogs barked.
3 The dogs are barking at the boys
4 Northing happens here.
5 My father works at night.
6 The hour passed pleasantly.
7 Several
arrived late.
guest
8 Everybody is going.
9 Sales have fallen off.
1 Water
occurs in the air.
0 vapor

Chu thch:
No. 6: Mot gi a troi qua mot cach thu v.
Le Th Thuy Loan Trang 7
Phng Phap Dch VIET ANH - NG PHAP

No. 9: Doanh thu a sut giam


No. 10: Hi nc co (hien dien) trong khong kh.

Cac cau Mau C Ban so 1 tren ay co the ch


gom mot tieng danh t lam chu ng va mot ong
t. ong t co the c bo ngha bi mot trang t
(an adverb). Trang t co the la mot t ng n oc (a
single word) hoac mot cum t (an adverb phrase): at night,
in the air, to a movie. Ta dung Adverbial e goi chung hai
trang t nay.

ong t dung Mau C Ban so 1 la cac t ong


t (the intransitice verb), ngha la cac ong t
khong can mot tan ng (an object).

PATTERN 2:

Noun Verb Adverbial


1 at the
Jonh is
party.
2 should
Everyone there.
be
3 tomorrow
The party will be
.
4 Mary was out.
5 on the
Your hat is
desk.
6 out of
Mr. Lee is
work.
7 My in the
are
parents country.
8 The
were not there.
policemen

Trong cac cau thuoc Mau C Ban so 2, ong t


luon luon la mot dang cua ong t Be (is, am, are, was,
were) va complement nam sau ong t Be la trang t.
Le Th Thuy Loan Trang 8
Phng Phap Dch VIET ANH - NG PHAP

Trang t trong cac cau Mau C Ban so 2 ch thi


gian hoac ni chon.

PATTERN 3:

Noun Verb Adjective


1 The desk was rude.
2 The
seemed friendly.
natives
3 The sky became cloudy.
4 The
looks young.
leader
5 His story sounded untrue.
6 The unchange
remains
situation d.
7 The air is damp.
8 The
tastes good.
coffee

Chu thch:
No. 2: Cac tho dan co ve than thien.
No. 5: Cau chuyen cua anh ta nghe co ve khong
that.
No. 6: Tnh trang van khong thay oi.
No. 8: Ca phe co v ngon.
ong t dung trong cac cau thuoc Mau C Ban so
3 la nhng ong t ch trang thai (the state verb/the
state - of - being verb). Co ca ngan t ong t (the
intransitive verb), nhng ch co khoang hn chuc ong t
ch trang thai.
Ngoai ong t To Be ra, cac ong t trang thai
thng gap la: seem (hnh nh), appear (co ve), become
(tr nen), sound (co ve), remain (van c), stay (van c),
smell (co mui), taste (co v).
Complement trong cac cau Mau C Ban so 3 luon
luon la mot tnh t. Tnh t nay oi khi c bo ngha
bi cac trang t nh very, quite, rather va too.

Chu thch:
Le Th Thuy Loan Trang 9
Phng Phap Dch VIET ANH - NG PHAP

Cac ong t ch trang thai oi khi c goi la


linking verbs.
Cac tnh t nam sau ong t ch trang thai
Mau C Ban so 3 cung c goi la predicate adjective -
ngha la mot tnh t nam phan v ng cua cau noi
e mo ta tnh trang cua chu ng. Tnh t nay cung
con c goi la subject complement.
Frank looks exhausted. (Frank = exhausted)
Frank trong co ve kiet sc
The room remained noisy all afternoon.
Can phong van c on ao suot buoi tra.
Co nhng trng hp ong t Mau C Ban so 3
la ong t ch trang thai, nhng neu c dung
Mau C Ban so 5 th no lai la ong t ch hanh ong
(an action adverb).

Th du:
1. a) The music sounded interesting. (state verb)
Nhac co ve hay.
b) The angry drivers sounded their horn. (action verb).
Cac tai xe phan no bop coi.
2. a) The children appear sleepy (state verb).
Cac chau be co ve buon ngu.
b) The moon appeared on the horizon. (action verb)
Mat trang xuat hien chan tri.
3. a) The weather remained hot for a long time.
(state verb)
Thi tiet van c nong.
b) She remained there for three weeks. (action verb)
Co ta lu lai o 3 tuan le.
4. a) Father stayed home; Mother went to the
meeting
Cha nha; Me th i hop. (state verb)
b) The man stayed silent. (action verb)
Ngi an ong van c yen lang.
5. a) The milk tasted sour. (state verb)
Sa co v chua.
b) Mary tasted her soup. (action verb)
Mary nem mon sup cua co ay.

Le Th Thuy Loan Trang 10


Phng Phap Dch VIET ANH - NG PHAP

6. a) The flowers smell sweet (state verb)


Cac bong hoa co hng v ngot ngao.
b) The dog smells the bone. (action verb)
Con cho ngi cuc xng.
7. a) Dont turn to the left. (action verb)
ng queo trai.
b) The leaves turn yellow in autumn. (state verb)
Mua thu la tr vang.
c) My hair has turn the grey. (state verb)
Toc toi a bac mau.
8. a) The villagers here grow cacao tree. (action
verb)
Dan lang ay trong cay cacao.
b) Its growing dark. (state verb)
Tri ang tr toi.
9. a) He fell four metres. (action verb)
Anh ta ri bon met.
b) He fell ill. (state verb)
Anh ta nga benh.
c) It fell dark. (state verb)
Tri tr toi.

10. a) The milk went sour. (action verb)


Sa tr chua.
b) She went mad and drowned herself. (state verb)
Co ta phat ien va t tram mnh.
11. a) Mr. Green is running a bid company. (action
verb)
Ong Green ang ieu hanh mot cong ty ln.
b) The well has run dry. (state verb)
Gieng nc a tr nen kho can.
c) Supplies are running low. (state verb)
o tiep lieu ang can dan.
b) The cows are running dry. (state verb)
Nhng con bo ang can sa.

PATTERN 4:

Noun Verb Noun

Le Th Thuy Loan Trang 11


Phng Phap Dch VIET ANH - NG PHAP

1 His
are teachers.
parents
2 a
Mr. Jones became
millionaire.
3 a
Linda remained
secretary.
4 used to a
His uncle
be footballer.
5 an
His car is
antique.
6 The have the best
books become sellers.

Chu thch:
No. 3: Linda van la th ky.
No. 4: Trc kia chu cau ta la mot cau thu.
No. 6: Sach a tr thanh nhng quyen ban chay
nhat.

Ba ong t be, become va remain la nhng ong t


duy nhat c dung Mau C Ban so 4. Cac ong t
nay cung c goi la state verbs hoac linking verbs.

PATTERN 5:

Noun Verb Noun


1 several
Peter asked
questions.
2 Hemingwa that
wrote
y novel.
3 My Hong
visited
parents Kong.
4 Dr. the
won
Fleming Nobel Prize.
5 many
Paris attracts
tourists.
6 accuracy
Science requires
.

Le Th Thuy Loan Trang 12


Phng Phap Dch VIET ANH - NG PHAP

7 guarante my
The store
es radio.
8 Carelessn accident
causes
ess s.
9 this
should
The city street.
repave
(Object)

Chu thch:
No. 4: Bac s Fleming a at giai Nobel.
No. 5: Paris hap dan nhieu du khach.
No. 6: Khoa hoc oi hoi s chnh xac.
No. 7: Ca hang bao hanh chiec radio cua toi.
No. 8: S bat can gay ra tai nan.
No. 9: Thanh pho nen lat gach lai con ng nay.

Nhng cau Mau C Ban so 4 va 5 co cung cau


tao:
Noun + Verb + Noun

Nhng ta tach chung ra thanh hai mau khac nhau v


chung cha hai loai ong t khac nhau. ong t c
dung Mau C Ban so 4 tao nen nhng cau trong o
danh t v tr chu ng va danh t v tr complement
am ch cung mot ngi hoac mot vat, hoac mot s
viec. Trai lai, ong t c dung Mau C Ban so 5
tao nen nhng cau trong o hai danh t v tr Subject
va Complement am ch hai ngi hoac hai vat khac
nhau. Cac ong t nay c goi la tha ong t
(transitive verbs).
Danh t theo sau ong t Mau C Ban so 4 c
goi la predicate noun hoac subject complement. Con
Mau C Ban so 5 danh t c goi la direct object (tan
ng hay tuc t).

PATTERN 6:

Noun Verb Noun Noun


1 We told our the
Le Th Thuy Loan Trang 13
Phng Phap Dch VIET ANH - NG PHAP

mother news.
2 Mr. taugh us Englis
Moore t h.
3 My me a
sent
uncle telegram.
4 The radioe the signals
polit d station .
5 The
gives us light.
sun
6 The the an
gave
Mayor press interview.
7 His that
got him
aunt bicycle.
8 The a
offere Frank
college scholarship.
d
(I.O) (D.O)

Chu thch:
No. 4: Vien phi cong ien cho tram cac tn hieu vo
tuyen.
No. 6: Ong th trng cho bao ch mot cuoc phong
van.
No. 8: Trng ai hoc a cap cho Frank mot hoc
bong.

Mau C Ban so 6 nay, ong t c dung cung


la cac transitive verb. Sau ong t la hai danh t i lien
nhau. Danh t i lien sau ong t c goi la indirect
object (I.O) va danh t ke tiep c goi la direct object
(D.O) (tan ng trc tiep).

Co ca ngan transitive verbs c dung Mau C


Ban so 5, nhng ch mot so rat t transitive verbs c
dung Mau C Ban so 6 va so 7. Nen nh rang hai
danh t mau nay am ch hai ngi hoac hai vat
khac nhau.

PATTERN 7A:

Le Th Thuy Loan Trang 14


Phng Phap Dch VIET ANH - NG PHAP

Noun Verb Noun Noun


1 Her Maria presiden
elected
classmates t.
2 the Mekong
They named
ship .
3 appointe him manager
They
d .
4 consider him
His mother a genius.
s
5 the a
The Mayor declared
campaign success.
6 Perseveran him a
made
ce champion.
7 christen the
The pastor Jennifer.
ed child
8 The mango made his
water.
mouth
(Obje (O.
ct Complement)

Chu thch:
No. 1: Ban cung lp a bau Maria lam lp trng.
No. 2: Ho a at ten con tau la Mekong.
No. 3: Ngi ta a ch nh ong ay la Giam oc.
No. 4: Ma cau ta coi cau ta nh mot thien tai.
No. 5: Th trng a tuyen bo chien dch la 1
thanh cong.
No. 6: S kien tr a bien thanh nha vo ch.
No. 7: V muc s a at ten thanh cho cau be la
Jennifer.
No. 8: Qua xoai a lam cau ay chay nc
mieng.

ong t dung Mau C Ban so 7 nay cung la


cac transitive verbs. mau cau nay, sau ong t cung
co hai danh t i lien nhau. Danh t ng lien sau ong
t c goi la direct object; danh t th nh ng lien
sau giup giai thch object va do o no c goi la object
complement - o la mot dang tuc t cua tuc t vay.
Le Th Thuy Loan Trang 15
Phng Phap Dch VIET ANH - NG PHAP

Mot so t cac transitive verbs ng .oo


gom cac ong t ch s at ten, goi ten, bau
c, bnh chon, ch nh ai vao chc vu g. Th du nh:
name (at ten), call (goi), elect (bau), appoint (ch nh),
choose (chon), nominate (bo nhiem, tien c),
christen (ra toi, at ten thanh), crown (ton lam vua)
v.v...
Ngoai ra, cac ong t find, leave, make cung c
dung cau nay.
Trong cac cau Mau So 7 tren, hai danh t v tr
complement am ch cung mot ngi, mot vat hoac
mot s viec.
Vi mot t ong t Mau C Ban so 7A, mot tnh
t co the thay the danh t th nh cot cuoi cung
va tnh t nay cung van c goi la complement, nh
trong Mau C Ban so 7B sau ay:

PATTERN 7B:

Nou
Noun Verb Adjective
n
1 conside him quite
His mother
rs handsome.
2 our
The meat made very happy.
dog
3 Perseveran him
made successful.
ce
4 The news made her disappointed.
5 declare the unconstitutio
The court
d law nal.
6 the
They found useless.
machnie

Chu thch:
No. 2: Mieng tht lam con cho chung toi rat vui.
No. 3: S kien tr a lam anh ay thanh cong.
No. 5: Toa an tuyen bo ao luat vi hien.
No. 6: Ngi ta thay rang co may vo dung.
Le Th Thuy Loan Trang 16
Phng Phap Dch VIET ANH - NG PHAP

LUYEN DCH:
Cac nha a ly a at ten bien ngoai khi VN la
bien ong.
Geographers named the sea on the Vietnamese coast the
East Sea.
Co thi ngi La Ma a ton Caesar lam vua.
The Romans had once crowned Caesar King.
Ngi ta a bnh chon Ronaldo la cau thu hay nhat
cua mua bong nay.
Theyve chosen Ronaldo the best player of this football
season.
Cha Me cua cau be a at ten cho cau be ay la
Hung, nhng thng goi cau ay la Tam.
The boys presents named him Hung but often called him
Tam
Trc ay ngi ta thng goi thuoc aspirin la than
dc.
People used to consider aspirin a miraculous cure.
Ong ay a lam cho cong ty c nh ngay hom
nay.
He had made the company what it is today.
Chung toi coi Quang Trung nh v anh hung dan toc.
We consider Quang Trung a national hero.
Cac nha khoa hoc a tuyen bo chuyen bay th
nghiem hoan toan that bai.
The scientists declared the test flight a complete failure.
Nam 1961 ngi My a bau Kenedy lam Tong
thong Hoa Ky.
In 1961 the American people elected Mr. Kennedy President
of the United States.
Nhng no lc to ln trong cuoc i a khien ong
ta noi tieng.
His great effort in life made him famous.
Tran hoa hoan va qua a khien 5 ngi t vong
va hang tram ngi khac ri vao canh khong nha.
(Pattern 7).
The recent fire left five people dead and hundreds of others
homeless.

Le Th Thuy Loan Trang 17


Phng Phap Dch VIET ANH - NG PHAP

FINAL NOTES ON THE BASIC SENTENCE PATTERN


CHU THCH SAU CUNG VE MAU CAU C BAN
Khi dch mot cau t tieng Viet sang tieng Anh, ta
thng oi chieu mot t ng gia hai ngon ng. Sau
khi biet c t loai (the part of speech) cua t tieng
Anh, ta mi la chon mau cau c ban va chon mot th
thch hp cho ong t.
Th du 1:
e dch cau: Chuyen bay th nghiem a thanh
cong.
Ta biet t thanh cong co ba t loai: (1) ong t
(to succeed), (2) tnh t (successful), (3) danh t (success).
Vay ta co the dung ba mau cau c ban khac nhau e
dch.

Pattern Subject x Verb


1: The test flight succeeded.
Pattern Subject x Be x Adjective
3: The test flight was successful.
Pattern Subject x Be x Noun
4: The test flight was a successs.

Th du 2:
Dch cau Anh ta s.
Pattern Subject x Be x Adjective
3: He is frightened.
Subject x Verb x Noun (Object)
Pattern
He has a fright.
5:
(He gets a fright)

Th du 3:
Dch cau Chan toi au. (Toi b au chan)
Ta biet t pain va la danh t va la ong t va
la mot transitive verb.
Subject x Verb x Object
Pattern
My foot is paining me.
5:
Pattern I have a pain in my foot.
5:

Le Th Thuy Loan Trang 18


Phng Phap Dch VIET ANH - NG PHAP

Pattern My foot gives me a lot of pains.


6:

Neu ta chon ong t to hurt, th ta biet ong t co


the la transitive hoac intransitive:
Pattern Subject x Verb
1: My foot hurts.

Pattern My foot hurts me.


5:

EXERCISES: UNDERSTANDING BASIC SENTENCE


PATTERNS.

Exercise 1A: The sentences below are all basic sentences.


Read each sentence carefully and tell which of the seven
Patterns it follows:
(Cac cau di ay tat ca eu la cau c ban. oc
moi cau can than va cho biet no thuoc mau cau
nao).

We call this ocean the Pacific. (Pattern 7)


The guests are out side. (Pattern 2)
This material feels luxurious.
The neighbours became friendly.
Frank leaves tomorrow.
Our guests waited outside.
My father remained in the house.
The natives remained friendly.
No one invited them.
It rained very hard.
We find this medicine an effective one.
The accident taught him a lesson.
Platini remained captain.
She looks ill.
Galileo discovered the first telescope.
The suburbs appeared at last.
The moon rose at midnight.
The jet planes roared from afar.

Le Th Thuy Loan Trang 19


Phng Phap Dch VIET ANH - NG PHAP

His achivement sounds unusual.


The place had once been a business center.
The plan was a success.
The machine performed silently.
Geographers named the river the Yellow River.
Please keep it a secret.
The satellite is orbiting the earth.
The journalists were at the airport.

Exercise 1B: Write five sentences that conform to each of


the following sentence Patterns. Nouns that are marked with an
asterisk (*) should refer to the same person or thing.
(Viet 5 cau cho moi mau c ban sau ay. Cac danh
t c anh dau (*) phai am ch cung ngi hoac
cung vat).

1. Noun Verb (Adverb)


2. Noun (*) Verb Noun (*)
3. Noun Verb Adjective.
4. Noun Verb Noun (*) Noun (*)
5. Noun Verb Noun
6. Noun Be Adverb.
7. Noun Verb Noun (*) Noun (*)
Exercise 1C: In each of the following sentences, underline
the adverbial. ( moi cau di ay, gach di trang
t).

Mr. Green is at the station.


We went to a concert.
The guests arrived late.
Jackson is in a hurry.
The baby slept soundly.
Sales have fallen off.
The mountain appeared in the distance.
Everybody is at work.
They escaped in a car.
They came with their friends.
Carbon dioxide (CO2) exists in the car.
The key is there.

Le Th Thuy Loan Trang 20


Phng Phap Dch VIET ANH - NG PHAP

She swam across the river.


It rained very hard.
We ate in a restaurant.
She put the music on the chair.

Chu thch: The adverbial object.


Trong tieng Anh co mot so ong t c theo sau
bi danh t, khien ta lam lan danh t nay la tan ng
(hay tuc t) ca ong t nh Mau C Ban so 5. Thc
ra cac danh t nay co chc nang cua trang t va ta
goi no la adverbial objective. Ta de dang nhan ra chc
nang trang t cua danh t v ong t trong cau la
intransitive verb.

nh ngha:
Adverbial objective la mot danh t c dung nh
trang t. Cac Adverbial objective thng ch khoang
cach, thi gian, trong lng hoac gia tr.

Bang ben la cac th du vi cac ong t c theo


sau bi mot adverbial objective. Gii t for co the c
dung trc cac trang t ch khoang cach hoac thi
gian, trong lng.

Adverbial
Noun Verb
objective
1 (for) five
We walked
miles.
2 has thousands of
He
traveled miles.
3 had
They a long way.
come
4 He fell ten metres.
5 The rain lasted all day.
6 (for) two
We waited
hours.
7 The (for) two
lasted
meeting hours.
8 Wont stay the night?
Le Th Thuy Loan Trang 21
Phng Phap Dch VIET ANH - NG PHAP

you
9 The book cost three dollars.
1
The baby weighs five kilos.
0
1 The several
fell
1 temperature degrees.
1 The measur 4 metres by 6
2 room es metres.

2. THE SINGLE - WORD MODIFIER OF NOUNS


(YEU - TO - MO - TA CUA DANH T)

Ta dch t Modifier la yeu - to - mo - ta. Trong tieng


Anh, modifier la mot t ng n oc (a single word), mot
cum t (a phrase), hoac mot menh e (a clause)
co chc nang mo ta hoac bo ngha mot t khac
trong cau noi.

bai hoc nay, ta ch xet en yeu - to - mo - ta


cua danh t trong trng hp no la mot t ng n
oc. dang nay, yeu - to - mo - ta thng c at
trc danh t ma no mo ta.

Cac t loai sau ay co the la yeu - to - mo - ta


cua danh t:
1. Mao t (the article), 2. Tnh t ch th (the
demonstrative adjective), 3. T ch s s hu (the
possessive), 4. Tnh t bat nh (the indefinite adjective),
5.Tnh t ch so em va so th t (the numeral), 6.
Tnh t (the adjective), 7. Danh t (the noun), 8. Phan t
(the participle), 9. Trang t (the adverb), 10. ong t
nguyen mau (the infinitive).

Articales: a, an, the


a factory, an employe, the workers.
Le Th Thuy Loan Trang 22
Phng Phap Dch VIET ANH - NG PHAP

Demonstratives: this, that, these, those


This center, that house, these pictures, those cars.
Possessives: My, your, his, their, Marys, the workers.
his job, their school, Marys hat, the mans office, the
childrens toys.
Indefinite adjectives: some, many, several, much, little, few.
some books, many products, little water, much money, few
people.
Numerals: one, two, fifteen, first, second, twentieth.
two factories, (the) second time, (the) sixth orbit, the
Twentieth Centery.
Adjectives: young, small, expensive, difficult...

Modifier
(Adjective Noun
)
1 (a) young man.
2 (the) happy women.
3 (som companies
big
e) .
4 expensive cars.
5 complicat
situation.
ed

Chu thch:
No. 5: Nhng tnh huong phc tap...

Nouns: tea, history, evening, flowers.


Mot danh t co the la yeu - to - mo - ta cho mot
danh t khac va nh the ta bao no c dung nh mot
tnh t.

Modifier
Noun
(Adjective)
1 (a) tea cup.
2 (a) history book.
3 (a) return ticket.
4 (som college students.

Le Th Thuy Loan Trang 23


Phng Phap Dch VIET ANH - NG PHAP

e)
5 (a) highway engineer.
6 newspaper
(an) evening
.
7 garden flowers.
8 (a) flower garden.

Chu thch:
No. 1: Mot tach uong tra.
No. 3: Mot ve kh hoi.
No. 5: Mot ky s cau ng
No. 6: Mot t bao chieu.
No. 7: Hoa trong trong vn.
No. 8: Vn trong hoa.
Khi dch mot cum danh t (a noun phrase) gom mot
danh t va mot hoac nhieu yeu - to - mo - ta cung i
vi no, ta dch danh t trc va yeu - to - mo - ta
c dch sau.
Vay: Cane sugar c dch la ng ma (ng lay
t cay ma).
Khac vi: Sugar cane co ngha la cay ma.

Cac th du khac:
Mot nhan vien van phong an office
worker
Mot mau cau a sentence pattern
Mot quyen t ien dung ni hoc ng a school
dictionary
Cn gio ngc head winds
Sc gio the wind power
Giao vien trung hoc a high-school
teacher
Mot hoc s va he a sidewalk
artist
Mot en e ban a desk clamp
Mot bc tng a a stone wall
Mot b vua uyen bac a philosopher king
Mot mau quang cao trong tap ch a magazin
ad.
Le Th Thuy Loan Trang 24
Phng Phap Dch VIET ANH - NG PHAP

Mot cau chuyen trinh tham a detective story


Mot ca hang ban ca a fish store.

Participles: running, coming, growing, spoken, visited, shot.


Ca hai hien tai phan t (the present participle) va
qua kh phan t (the past participle) eu co the la
yeu - to - mo - ta cua danh t, no thng c goi la
participial adjective.

Modifier
Noun
(Adjective)
1 th
coming months.
e
2 running water.
3 th
following sentences.
e
4 growing children.
5 a swimming pool.
6 a dying soldier.
7 a dancing Master.
Chu thch:
No. 1: Nhng thang ti ay...
No. 2: Dong nc chay...
No. 4: Nhng tre con ang a phat trien
No. 6: Mot chien s ang hap hoi
No.7: Mot vu s

Bang di ay cho thay mot qua kh phan t co


chc nang cua mot tnh t.

Modifier
Noun
(Participle)
1 an invited guest.
2 spoken languages.
3 th
bombed city.
e
4 a stolen cat.
5 baked potatoes.

Le Th Thuy Loan Trang 25


Phng Phap Dch VIET ANH - NG PHAP

6 th
injured tiger.
e
7 th
surrounded platoon.
e

Chu thch:
No. 1: Mot khach mi
No. 3: Thanh pho b doi bom
No. 5: Khoai tay nng
No. 6: Con ho b thng
No. 7: Trung oi b bao vay

LUYEN DCH: Dung participle nh tnh t e dch cac


cau sau:

o la mot chiec o to a s dung roi.


That is used car.
Con cho b giet la con cho ien.
The killed dog was a mad one.
Tia laser co the diet cac te bao b nhiem benh.
The laser beam can destroy diseased cells.
Thanh pho b doi bom la thanh pho cong nghiep.
The bombed city was an industrial one.
Tri se ma nhieu vao nhng thang sap ti.
It will rain a lot in the coming months.
Cac lc lng tan cong a c gi ti Somali.
The attacking forces were sent to Somali.
Con tau b tan cong la mot con tau ch dau
The attacked ship was an oil tanker.
Thuy tinh c lam t cat nau chay, so a va
voi.

Le Th Thuy Loan Trang 26


Phng Phap Dch VIET ANH - NG PHAP

Glass is made from melted sand, soda and lime.


Anh ta cung tra tien cho cai ly be.
He also paid for the broken glass.
Ong ay thch khoai tay nng an vi ga nng.
He likes baked potatoes with fried chicken.
Sa la thc an rat tot cho cac tre em ang ln.
Milk is good food for growing children.
Con nga thua giai a lam buon long chu no.
The prize-losing horse saddened its master.
Con gau b thng can nang 100 ki-lo.
The injured bear weighed one hundred kilograms.
Dong nc chay t loc (lam cho trong)
Running water purified itself.
Cac nha khao co va tm thay mot thanh pho b
chon vui co 2000 nam tuoi.
Archeologists have just found a 2000 year-old buried city.
9. Adverbs:
Mot vai trang t cung co the la yeu - to - mo -
ta cua mot danh t, ngha la no c dung nh mot
adjective. Co trng hp trang t c at trc danh
t, co trng hp no c at sau.

Noun Modifier (Adverb)


1 (the) boy upstair
2 (the) sky above
3 (the) people inside
4 (the) plains below
5 (the) villagers here
6 (the) day after

Chu thch:
No. 1: Cau be tren gac
No. 2: Bau tri ben tren
No. 4: Nhng canh ong ben di
No. 5: Nhng dan lang ay

Bang di ay cho thay yeu - to - mo - ta c


at trc danh t.

Le Th Thuy Loan Trang 27


Phng Phap Dch VIET ANH - NG PHAP

Noun Modifier
(Adverb)
1 (the) down train
2 (the) up train
3 (the) above statement
4 (the) then government
5 (the) through train

Chu thch:
No. 1: Chuyen tau xuoi.
No. 2: Chuyen tau ngc.
No. 3: Li phat bieu tren.
No. 4: Chnh phu cam quyen luc bay gi.
No. 5: Chuyen tau suot.
Cac trang t ghep co gach noi cung co the la
yeu - to - mo - ta cua mot danh t. Chung c at
trc danh t.

Noun Modifier (Adverb)


1 (the) half past train
ten
2 (an) out and out failure
3 (a) pen and ink drawing
4 (a) black and T.V set
white
5 (an) up to date report

Chu thch:
No. 1: Chuyen tau 10 gi 30
No. 2: Cuoc that bai hoan toan
No. 5: Ban bao cao a c cap nhat hoa

LUYEN DCH: Using Adverbs As Noun Modifiers

Chnh phu luc bay gi hay con non yeu.


The then government was still young and week.
C dan o rat than tnh.
The people there are very friendly.

Le Th Thuy Loan Trang 28


Phng Phap Dch VIET ANH - NG PHAP

Chung toi co the nhn thay con song Me Kong


uon khuc qua nhng canh ong ben di.
We could see the Mokong River winding its way through the
plains below.
Cac cong nhan ben ngoai ang noi cn thnh no.
The workers outside are getting very angry.
Ngon oi ben kia phu ay tuyet.
The hill beyond is covered with snow.
Nhng li phat bieu tren khong chan that.
The above statements are not honest.
Cau ay hoan toan la ke du thu du thc
He is an out-and out vagabond.

10. Infinities:
ong t nguyen mau (the infinities) co the la yeu -
to - mo - ta cua danh t. No luon luon c at sau
danh t ma no mo ta.

Noun Modifier (To -


infinities)
1 (a) report to finish
2 (the) exercises to do
3 (the) right to vote
4 (the) problems to solve
5 (the) months to come
6 (the) objectives to capture

Chu thch:
No. 1: Mot bao cao phai hoan tat
No. 2: Cac bai tap phai lam
No. 3: Quyen bau phieu
No. 4: Nhng van e phai giai quyet
No. 5: Nhng thang sap ti
No. 6: Nhng muc tieu phai chiem lnh

LUYEN DCH: Using Infinities As Noun Modifiers


Dung ifinities lam yeu - to - mo - ta cua danh t

Ngay xa phu n khong co quyen bau phieu.


Le Th Thuy Loan Trang 29
Phng Phap Dch VIET ANH - NG PHAP

In the old days women didnt have the right to vote.


Quyet nh ra i cua anh ta lam chung toi ngac
nhien.
His decision to go surprised us.
Nhom chung toi co hai bao cao phai hoan tat.
Our group has two reports to finish.
Cac nha nong se co nhieu cong viec ong ang
phai lam vao nhng thang ti.
In the months to come, the farmers will have a lot of
farming work to do.
Anh ay ngh rang y khoa la nghe tot e chon.
He thought that medicine is the best profession to choose.
ng nhien la ban co quyen t choi.
Of course you have the right to refuse
Co le se co nhieu kho khan phai vt qua.
There may be a lot of difficulties to overcome.
Anh ta co mot gia nh ong uc phai bao boc.
He has a large family to support.
o la thi iem e bat au.
Its time to start.
Co ay bay to niem ao c t kiem song.
She expresses a wish to earn her own living.
Ong Giam oc ieu hanh a thong bao y nh
t chc.
The managing director announced his intention to resign.
Quan chung se khong thch ke hoach rut lui cua
ong ay.
People will not like his plan to retire.
S t choi giup cua anh ta la mot ieu that
vong.
His refusal to help was a disappoinment.
ay co phai la cach giai quyet van e khong?
Is this the way to solve the problem?
Toi cam thay nghi ng ve kha nang lam viec cua
anh ta.
I feel doubtful about his ability to do the work.

Le Th Thuy Loan Trang 30


Phng Phap Dch VIET ANH - NG PHAP

3. MODIFIERS: WORD ORDER


(V TR CUA CAC YEU TO MO TA )

Khi cac yeu - to - mo - ta gom nhieu t loai khac


nhau (danh t, tnh t, phan t, v.v...) va c dung
e bo ngha cho mot danh t, th van e v tr cua
chung c at ra. Yeu to nao c at trc yeu to
nao?

Th du 1:
Mot ong ho vang: a gold watch
Mot ong ho xa bang vang: a old gold watch
b) Mot ong ho xa bang vang rat hiem cua
Ong Green.
Mr. Greens very rare, old gold watch.

Trong cum danh t (the noun phrase) tren, danh t


watch c bo ngha bi cac modifiers la: old, gold, rare
va Mr. Greens.

Th du 2:
Mot ky ngh he: a summer holiday
Mot ky ngha he dai rat thu v
a very nice long summer holiday .

Th du 3:
Mot ao s-mi bang vai co-tong: a cotton shirt
Mot ao s mi vai co-tong mau o rat at tien.
A very expensive red cotton shirt.

Le Th Thuy Loan Trang 31


Phng Phap Dch VIET ANH - NG PHAP

Ta hay xem v tr cac modifiers trong hnh o (diagram)


cac th du di ay.
th du (1) danh t engine co 5 yeu - to - mo - ta.
th du (2) danh t wall co 4 yeu - to - mo - ta.

Th du 1:
James Walt a phat minh chiec ong c hi nc noi
tieng au tien cua the gii.
James Walt invented the worlds first famous steam engine.

Pattern 5: Subject x verb x object


James Walt invented engine
the worlds first famous steam
(1) (1) (2) (3) (6)
Th du 2:
Cac cong nhan a xay mot bc tng gach o
cao.
The workers have built a high red brick wall.
workers have buil wall
the a high red brick
(1) (4) (5) (6)
Bang di ay ch dan cach xep at cac
modifiers khi chung cung bo ngha cho mot danh t.

V TR CAC MODIFIERS

Khi mot danh t c bo ngha bi yeu - to - mo -


ta la mot danh t khac, hoac gerund, hoac participle th
yeu - to - mo - ta o c at lien trc danh t.
Neu co mot yeu - to - mo - ta khac la mot tnh t
ch mau sac, th tnh t nay c at trc gerund
hoac participle noi tren.
Cac tnh t ch pham chat (the quality) thng xuat
hien trc cac tnh t ch tam c (size), hnh dang
(shape).
Cac yeu - to - mo - ta noi tren c at sau cac
tnh t ch so th t va so em (the nummeral). Cac
tnh t ch th (the demonstrative adjective), mao

Le Th Thuy Loan Trang 32


Phng Phap Dch VIET ANH - NG PHAP

t (the article) va tnh t s hu (the possessive adjective)


la cac yeu - to - mo - ta xuat hien sm.

MODIFIERS
(1) ( (3) ( ( (
Ar 2) Qu 4) 5) 6)
ticles N ality, S C N
De ume etc ize, olou oun,
NO
monstr tal L r G
UN
ative ength erund
Po , P
ssecssi S artici
ves hape, ple
etc
va
a g w
1 luable,
very old atch
old
s
th w w
2 eco
e orld ar
nd
b l
l b
3 a row eathe
ong elt
n r
s
he p f
4 mall,
r ink ace
round
y d
m t ex s
5 ello resse
arys wo pensive ilk
w s
t
to t att g s
6 riang
ms hree ractive reen tamps
ular
1
se st b c
7 5thCe
veral udy ig astles
ntury
us o t b
8 a
eful blong in ox
9 th t fa p p
Le Th Thuy Loan Trang 33
Phng Phap Dch VIET ANH - NG PHAP

rofess
ese wo mous hysics
ors
int p
1 o
an erestin aintin
0 il
g, old g
Chu thch:
No. 1: Mot ong ho vang xa rat co gia tr.
No. 2: Chien tranh the gii lan th hai.
No. 3: Mot that lng da mau nau dai.
No. 4: Khuon mat nho, tron, mau hong cua co ay.
No. 5: Hai chiec ao am luc mau vang, at tien
cua Mary.
No. 6: Ba con tem tam giac mau xanh rat hap dan
cua Tom.
No. 7: Vai toa lau ai to, kien co cua the ky 15.
No. 8: Mot hop thiet hnh khoi ch nhat rat hu
dung.
No. 9: Hai v giao s vat ly noi tieng nay.
No. 10: Mot bc tranh sn dau xa ep.

Exercise 3A: Making Noun Phrases


Sap xep cac modifier vao ung v tr e chung
cung bo ngha cho tieng noun cuoi cung c gach
di e tao thanh Noun Phrase.
Ex: tin/ useful/ a/ box. A useful tin box.
sour / green / some / eating / apples.
small / pink / round / her / face.
very famous / university / two / those / protessors.
pretty / yellow / Lindas / cotton / three / shirts.
swimming / very large / a / rectangular / pool.
two / these / glass / large / front / doors.
cigar / large / old / an / box.
enormous / red / yellow / and / an / umbrella.
three / the / first / questions.

Exercise 3B: Position Of Noun Modifiers

LUYEN DCH:

Le Th Thuy Loan Trang 34


Phng Phap Dch VIET ANH - NG PHAP

Ap dung bang ch dan v tr cua cac modifiers, lan


lt theo 3 bc: a. Tao mot noun phase n gian
Ni rong noun phase phan (a)
a vao cau noi hoan chnh

1. a) Cong nghiep ong tau


The ship building industry.
b) Nen cong nghiep ong tau noi tieng the
gii
The worlds famous ship building industry.
c) Nc Anh va Nhat Ban co nen cong
nghiep ong tau noi tieng the gii.
Eng. and Jap. have the worlds famous shipbuilding industry.
2. a) Mot that lng da dai.
A long leather belt.
b) Mot that lng da dai mau nau rat la.
An unusual long brown leather belt.
c) Cha cau ay a cho cau 01 that lng da dai
mau nau rat la.
His father had given him a very unusual long brown
leather belt.
3. a) Mot san van ong 60.000 cho ngoi.
A 60,000 seat stadium.
b) Mot san van ong 60.000 cho ngoi hnh bau
duc
An oval 60,000 seat stadium.
c) Mot san van ong 60.000 cho ngoi hnh bau
duc va mi c xay dng xong.
An oval 60,000 seat stadium had just been built.
4. a) Hai bc tranh sn dau.
Two oil paintings.
b) Hai bc tranh sn dau noi tieng cua Van
Gogh.
Van Goghs two famous oil paintings.
c) Hai bc tranh sn dau noi tieng cua Van Gogh
a b anh cap hom qua Amsterdam.
Van Goghs two famous oil paintings were stolen in
Amsterdam yesterday.
5. a) Nam hon a mat trang.

Le Th Thuy Loan Trang 35


Phng Phap Dch VIET ANH - NG PHAP

Five moon rocks.


b) 5 hon a mat trang hnh bau duc mau nau
rat quy hiem.
Five precious, rare oval brown moon rocks.
c) Ngi ta trien lam nam hon a mat trang
hnh bau duc mau nau rat quy hiem.
They exhibited 5 precious, rare oval brown moon rocks
6. a) Ngi ta a ban au gia 10 ong tien
xa bang vang rat quy hiem cua ong Green. / (ban
au gia = to auction off, ong tien xa = an ancient coin).
7. Carbon dioxide (CO2) la mot chat kh khong
mui, khong mau va khong the chay c.
(chat kh = a gas, khong the chay c =
incombustible).
8. Vang la mot kim loai quy, mau vang de
dat mong.
(quy = precious, de dat mong = malleable)
9. Dan lang a thay 1 vat the bay hnh tam
giac mau xam rat ky la
(ky la = unusual, vat the bay = a flying object)
10. Mat trang la mot qua cau khong lo khong
co nc, khong co khong kh, khong co ngi c ngu.
(khong co ngi c ngu = uninhabited)
11. T phi c chung toi co the nhn thay nhng
canh ong lua xanh ti bat ngat.
(bat ngat = immense /adj)
12. Anh ta a chon oi giay da nai mau nau
khong at tien lam.
(khong at tien = inexpensive, da nai = deerskin)
4. THE P - GROUP
(THE PREPOSITIONAL PHASE)

Trong tieng Anh, yeu - to - mo - ta co the la mot


Pgroup. Nh a noi phan au, P la ch au cua t
Preposition, Pgroup la mot cum t bat au bang t
ma ta thng goi la Preposition phrase.

e lam quen vi cac bai hoc sau nay, ta nh ngha


cac thuat ng sau:

Le Th Thuy Loan Trang 36


Phng Phap Dch VIET ANH - NG PHAP

The Phrase:
Phrase la mot cum t khong cha mot chu ng
va mot ong t, c dung nh mot t loai.

The Verb Phrase


The verb phrase la hai hay nhieu t c dung nh 1
ong t.

The Noun Phrase


Noun Phrase la hai nhieu t ng c dung nh mot
danh t.

Th du:
Lincoln Highway, Empire State Building, The Five, Star
Saigon Floating Hotel.

The prepositional phrase (the P-goup)


Preposition phrase la mot cum t gom co mot
preposition, tan ng cua no cung vi cac yeu - to -
mo - ta cua tan ng.

Sau ay la mot so cac gii t thong dung va


cac P-group tng ng:

Preposition P-group (Prepositional


phrase)
about about the trip, about
them
across across the ocean
after after war, after dinner
against against the enemy,
against cencer
among among the workers,
among us

Le Th Thuy Loan Trang 37


Phng Phap Dch VIET ANH - NG PHAP

around around the earth


between between two countries
beyond beyond the hills
into into space, into the
water
off off the orbit, off the
coast
over over the world
toward toward the moon
on account of (v, on account of illness
bi v)
with with a friend, with
water
without without a sound,
without money
because of (v) because of the rain
in frond of in front of the
classroom
By means of by means of computers
(bang, bang
phng tien)
in spite of in spite of illness
(mac, bat
chap)
in addition to in addition to his help
(them vao)
thanks to (nh, thanks to new
nh ) medicines
according to (theo according to the report
nh)

Chc nang cua Pgroup


Pgroup la mot yeu - to - mo - ta. No c dung
nh mot tnh t hoac trang t.

Le Th Thuy Loan Trang 38


Phng Phap Dch VIET ANH - NG PHAP

4.1 THE PGROUP AS ADJECTIVE


(Pgroup dung nh tnh t)
Khi mot prepositional phrase c dung nh mot tnh
t, no mo ta hoac bo ngha mot danh t hoac ai t
va cum t nay c goi la adjective phrase, ngha la
mot cum t c dung nh tnh t.

Th du:
The energy of an atom is tremendous.
Nang lng cua hat nhan th khung khiep
Cum t of an atom la mot Pgroup, bo ngha cho danh
t energy, do o no c coi nh co chc nang cua
adjective.

The flight to the planet Mars was a success.


Chuyen bay en Sao Hoa a thanh cong.

Ta at cau hoi: Which flight?. Cau tra li se la


the flight to Mars. Do o cum t to Mars bo ngha cho
danh t flight. No co chc nang cua mot adjective.

Cac th du khac:
We began a struggle against the enemy
Chung toi bat au cuoc tranh au chong quan
thu.

We saw a statue of the dying gladiator


Chung toi thay bc tng cua ngi giac au
ang hap hoi.

The treaty between the two countries had been signed.


Hiep c gia hai quoc gia a c ky ket.

The notice for the meeting will be announced soon.


Bien ban cuoc hop se sm c thong bao.

The essay by Miss Van Meter was selected


Bai tieu luan cua co Van Meter a c chon.

Le Th Thuy Loan Trang 39


Phng Phap Dch VIET ANH - NG PHAP

I have some money for small expenses.


Toi co t tien tieu vat.

4.2 THE PGROUP AS ADVERB (P-group dung nh


trang t)
Mot prepositional phrase co the c dung nh trang
t. Khi c dung nh trang t, cum t nay bo ngha
cho mot ong t. Mot tnh t hoac mot trang t va
c goi la adverb phrase.

The rocket soared to the moon.


(to the moon bo ngha cho soared)

The letter was hidden under some books.


(under some books cho biet v tr ve ong t was
hidden)

We struggled gainst the enemy.


Chung toi au tranh chong quan thu

Tom assembled the toy train according to the directions.


Tom lap rap xe la o chi theo li ch dan.

The boy shouted because of the noise.


(because of noise bo ngha cho ong t shouted)

They were curious about our new car


(about our new car bo ngha cho tnh t curious)

The plan was successful beyond our expectations.


(beyond our expectations bo ngha cho tnh t
successful).
Ke hoach thanh cong ngoai s mong i cua chung
toi.

The came late at night


(at night bo ngha cho trang t late)

Le Th Thuy Loan Trang 40


Phng Phap Dch VIET ANH - NG PHAP

The baby criers early in the morning to awarken his


parents. (in the morning bo ngha cho trang t early)

Khi hai hoac nhieu P-group xuat hien lien nhau,


chung co the bo ngha cho cung mot t, hoac mot P-
group nay co the bo ngha cho tan ng nam trong P-
group ng trc.

We arrived at the station in the morning.


(at the station va in the morning cung bo ngha cho
arrived)

He drove to New Yord for a contract.


(to New Yord va for a contract la hai trang t ch ni
chon va muc ch, bo ngha cho drove).
The Mediterranean Sea is connected with the Red Sea
by the Suez Canal.
(with the Red Sea/ by the Suez Canal cung bo ngha cho
connected).
a Trung Hai c noi vi Hong Hai bi kinh Suez.

Jack hid the leter in the bottom drawer of his desk.


(in the bottom drawer c dung nh trang t, bo
ngha cho ong t hid. Of his desk c dung nh tnh t
bo ngha cho drawer).

Adv Adj Adj


We found the diary in the middle of a box of
rubbish.
(in the middle bo ngha cho ong t found; of a box bo
ngha cho middle; of rubbish bo ngha cho box)

The meaning of the freedom differs widely between


nations
Y ngha cua t do khac nhau nhieu gia cac quoc
gia.

Le Th Thuy Loan Trang 41


Phng Phap Dch VIET ANH - NG PHAP

THE PGROUP AS NOUN (P-group dung nh danh t)


oi khi ta cung thay P-group c dung nh danh t.
Nhng cong dung nay t c thay nh cong dung tnh t
va trang t cua cum t nay.

Before breakfirst is the best time for swimming.


(before breakfirst la chu ng cua is)
From New York to Washington is about 250 miles
We advanced to within a few yards of the house.
Chung toi tien ti cach can nha vai thc.
(within a few yards c dung nh danh t, obj. cua
gii t to).
He comes from across the ocean
Anh ta en t ben kia b ai Dng.

Chu thch:
Khi P-group co chc nang cua tnh t, cum t nay
c at sau danh t ma no bo ngha. Trong nhieu
trng hp, cum t nay co the c thay bang mot
tnh t hoac danh t c nam trc no.

Th du:
Men of honour = honorable men.
A walk in the evening = an evening walk
A swim in the moonlight = a moonlght swim.
Towns by the sea = seaside towns.

Khi P-group c dung nh mot adverb, cum t nay


co v tr au cau hoac sau ong t. No khong bao
gi xuat hien trc ong t.
The farmers will plow their land in April.
Hoac: In April the farmers will plow their land.
At dawn the village women took their laundry to the river
(cac phu n mang o giat ra song luc bnh minh)
In the month the man were working again at their old jobs.
After this treament, the water is free of disease germs.
(sau lan x ly nay, nc khg con nhiem khuan
gay benh)

Le Th Thuy Loan Trang 42


Phng Phap Dch VIET ANH - NG PHAP

Trong cac cau co trat t nghch ao (inverted


sentences - ao ng), P-group thng c at au
cau.
Toward the cliff ran the frightened boy.
From his mistakes emerged my victory.
My victory emerged from his mistakes.
Down the diver floated some fishing boats.
Some fishing boats loated down the river.

ADJECTIVE OR ADVERB?

Khi mot P-group c dung Mau C Ban so 5


(Sub. x V x Obj.) - ngha la no xuat hien sau mot danh
t, ta xac nh chc nang cua P-group tuy theo y ngha
cua cau noi.
Tom held the newspaper before his eyes.
Ro rang la cum t before his eyes tra li cho cau
hoi Where did Tom hold the newspaper?. Do o no la
Adverb.

Nhng trong cau: Tom didnt read the magazine before his
eyes, (before his eyes c dung nh adj, bo ngha cho
magazin).
Cum t before his eye tra li cho cau hoi Which
magazine...?. Do o no la adjective.

Khi hai P-group eu la adverb cung bo ngha cho mot


ong t th trang t ni chon (adverb of place) c at
trc trang t thi gian (adverb of time).

PLACE TIME
1 He went to the before
station lunch
2 Meet me outside at five
the office oclock
3 Please to the before
return the books library Friday
4 He drove in the for
rain hours
Le Th Thuy Loan Trang 43
Phng Phap Dch VIET ANH - NG PHAP

5 The law into on May


went effects 15, 1995

Chu thch:
No. 5: Luat co hieu lc t ngay 15 thang 5 nam
1995

Exercise 4A: Identifying Prepositional Phrase


(Nhan dien P-group)
Underline each Prepositional phrase in the following
sentences and write in above whether it serves as an adjective
or adverb. Draw an arrow from the phrase to the word it
modifies.
Trong cac cau sau ay, gach di moi Prepositional
phrase va ghi ben tren cho biet cum t c dung nh
tnh t hay trang t
Adj Adv
Ex: The tray of food sat on the table.

The kite crashed into a tree.


The notice for the meeting will be posted within a week.
A bottle of glue mended the broken chair.
The white house on the corner has been vacant since July.
One part of the engine was produced in this plant.
June laid the report for Mr. Horst on his desk.
The railroad tracks wound beside the river in the canyon.
Chester had not seen his cousin until the day of the family
reunion.
No one slept during the crisis.
The fisherman went into the sea.
The story of accident appeared on the front page.
They arrived at the airport on time.
The fishing boat appeared on the horizon.
The electron is the giant of momern times.
Hat am ien t la anh khong lo cua thi hien
ai.
The patient still walked with difficully.
News of the general strike spreads throughout the country.
Mr. Green had cancer of the lung.

Le Th Thuy Loan Trang 44


Phng Phap Dch VIET ANH - NG PHAP

The pictures of the astronauts were taken before noon.


The reception for the P. Minister was held in the City Hall.
The man near the gate is very helpful.
The catch of fish was small.
Our supply of water has already been axhausted.
Nguon cung cap nc cua chung ta a can kiet.
Carbon dioxide (CO2) occurs in the air naturally.
The new school will stand on the side of the old church.
The rocket crashed into the lunar surface.
Since 1988 the cost of building material has doubled.

Exercise 4B: P-group as Adjective or Adverb


Dung mot preposition e tao P-group phan (a) roi
a vao cau noi hoan chnh phan (b) hoac (c).

1a. Bang vo tuyen/ gia hai phi cong vu tru.


by radio/ between two astronauts.
b. Cuoc tiep xuc bang vo tuyen gia hai phi
cong vu tru a bat au vao sang sm.
The contact by radio between two astronaus began early
in the morning.
2a. Khong mot ola/ dnh tui
without a dollar/ in his pocket.
b. Erick lai xe i New York khong mot ong ola
dnh tui
Erick drove to New York without a dollar in his pocket.
3a. Bang nhng con so va ch
in figures and words
b. So tien tren mot set thanh toan phai viet
bang nhng con so va ch.
The amount of a check should be written in figures and
words.
4a. Ben phan toi/ cua mat trang/ bi cac
thiet b ien t hien ai nhat.
Of the dark (far) side/ of the moon/ by meams of the most
modern electrinic equipment.
b. Cac bc anh ben phan toi cua mat trang
a c chup bi cac thiet b ien t hien ai nhat.

Le Th Thuy Loan Trang 45


Phng Phap Dch VIET ANH - NG PHAP

The pictures of the dark (far) side of the moon were taken
by means of the most modern electronic equipment.
5a. Sau cung/ cho nen d0oc lap cua nc nha
In the end/ for the independence of the nation.
b. Sau cung cuoc au tranh cho nen oc lap
cua nc nha a thanh cong.
In the end, the struggle for the independence of the nation
was successful.
6a. Lam nguon nhien lieu (dung as)/ cho cac
nha may ien
b. Chung ta co the dung anh sang mat tri
lam nguon nhien lieu cho cac nha may ien.
7a. trng/ ban be/ xa hoi
b. Nhng g chung ta hoc trng, ban be,
xa hoi eu hu ch ca.
8a. Lam benh vien noi (dung gii t as)
b. Trong thi chien ngi ta a dung chiec tau
nay lam benh vien noi.
9a. Nh khoa hoc (dung thanks to)
b. Nh khoa hoc nhieu qui luat cua thien nhien
a c kham pha.
10a. Vt ai Tay Dng/ vao cuoi the ky 15
b. Columbus bat au cuoc hanh trnh vt ai
Tay Dng vao cuoi the ky 15.
11a. Khap the gii chong lai benh SIDA.
b. Cac khoa hoc khap the gii ang th
nghiem mot vac sin mi chong lai benh SIDA.
12a. Nh vao cac dc pham va cac thiet b y
khoa hien ai.
b. Nh vao cac dc pham va cac thiet b y
khoa hien ai ma con ngi co the song lau hn ngay
trc.
13a. (Mot thanh pho) co cong nghiep ong tau
noi tieng (dung gii t with)
b. Hiroshima mot thi a tng la mot thanh
pho ln co nen cong nghiep ong tau noi tieng.
Benh tim la mot trong nhng nguyen nhan chnh
gay t vong.

Le Th Thuy Loan Trang 46


Phng Phap Dch VIET ANH - NG PHAP

Mot ngi khong cong an viec lam thng song


trong buon nan va ngheo kho.
oi vi cac con toi, cho voi an ma la mot phan
cua thu vui i s thu. (thu vui i s thu: the fun of going
to the zoo)
Chiec mu ang troi tren mat nc ay gieng.
( ay gieng: at the bottom of the well)
Cac nha phau thuat co the dung tia laser lam dao
mo rat chnh xac. (tia laser: laser beams/ lam = nh, nh:
as)
S chuyen ong cua trai at quanh mat tri tao
nen bon mua.
Cac nha khoa hoc khong the tnh qu ao cua cac
phi thuyen ma khong s dung may tnh ien t.
(ma khong s dung: without the use of)

FINAL NOTES ON THE P-GROUP


CHU THCH SAU CUNG VE P-GROUP

Co nhieu trng hp ta dung mot Preposition e


khoi phai dch ngha mot ong t, v the ma cau van
tr nen gon va nhe nhang hn.

Th du: Ta thng dung gii t for e ch muc


ch
Toi se ra pho e mua t sach
I will go downtown for some books.
Ong ta a ri thanh pho e i Ha Noi.
He left the city for Hanoi
Phong trao v hoa bnh a lan rong khap Chau Au.
The movement for peace spread throughout Europe.
Ong Green bam chuong goi ba an sang.
Mr. Green rang the bell for his breakfast.
Cuoc au tranh e oi lay cai an cua dan lang
ay that khong de dang.
The struggle for food of the villagers here is not easy.
Cac v ai bieu a en d cuoc tranh luan.

Le Th Thuy Loan Trang 47


Phng Phap Dch VIET ANH - NG PHAP

The delegates came for a debate.


Cuoc tranh au e cu mang song cua a be
a bat au.
The struggle for the boys life has begun.
Bac Ho ra i tm ng cu nc.
Uncle Ho went for the ways of salvation for the country.

e ch tien trnh cho mot hanh ong ang xay ra,


ta dung gii t Under.
Tai nan may bay ang c ieu tra a gay t vong
cho 300 hanh khach.
The plane crash under investigation caused death to 300
passengers.
Chiec cau ang c xay cat dai 500 met.
The bridge under construction is 500 metres long.
Ta thng dung gii t With vi y ngha s hu,
co:
o la mot quoc gia co the che cong hoa.
That is a country with a republican form of government.
Ong ta la ngi co kha nang lanh ao.
He is man with leadership ability.
Mary la co gai co mai toc xoan.
Mary is a girl with curly hair.

Cac th du khac:
Ngi sinh vien eo knh la nha vo ch bong ban.
The student with glasses ia a ping-pong champion.
Ngi cong nhan vac thang la th ho.
The workman with a ladder is a bricklayer.
Co gai be em ten Hoa.
The girl with a baby in her arms is Hoa.
Ngi an ong mac ao s mi xanh ng goc
pho la th may.
The man in the blue shirt on the corner is a mechanic.
Columbus a thanh lap oan thuy thu gom 52
ngi.
Columbus made up a crew of fifty-two men.
Ten cp mang tui bac b bat trc tien.
The thief with the bag of money was arrested first.

Le Th Thuy Loan Trang 48


Phng Phap Dch VIET ANH - NG PHAP

Toa nha ai Hoi ong New York co u cho cho


800 ai bieu ngoi lam viec.
The General Assembly building in New York has enough
room for 800 delegates at the desk.
Ch khi man em buong xuong chung ta mi co
the nghe chim hoa mi hot hay nhat.
It is only in the darkness that we can hear.

Exercise 4C: Using the P-group


Dch cac cau sau ay sang tieng Anh bang cach
bien cac cum t in am thanh P-group.

Tam i vi me en thanh pho lan au tien.


Qua bom no tren khong o cao 20 met ben
tren chiec cau.
Vi s giup cua may tnh ien t, chung ta co
the kiem soat may moc, tnh quy ao cac ve tinh va
tau vu tru, va tham ch dch ngon ng.
Chuyen bay vao khong gian de khao sat bau kh
quyen cua trai at a c hoan lai.
Nhng ngi ng ben ngoai nha may la cac
cong nhan b sa thai. (cong nhan b sa thai: sacked
workers, dismissed workers)
Ong ta a khong en d buoi hop v cam lanh.
Phong trao oi hoa bnh va chong chien tranh hat
nhan lan rong khap Chau Au.
Chnh phu cam quyen luc bay gi hay con non
yeu.
Chung ta can mot th trng e tieu thu cac sa
pham nong nghiep cua chung ta.
Bo phim a thanh cong ngoai s mong i cua moi
ngi.
Ngoi chua toa laic tren nh nui a c xay vao
the ky 15.

Le Th Thuy Loan Trang 49


Phng Phap Dch VIET ANH - NG PHAP

Columbus la mot nha hang hai co tai nang va


co nhieu tham vong. (nha hang hai: a navigator)
Chiec Mekong II ri cang hom qua e i Singapore.
Cuoc chien au chong tham nhng va buon lau
cha thanh cong.
Ngi an ong cam micro ang tiep th mot san
pham mi.
Ngan hang co nhng cuon phim cho thay vu cp
ang tien hanh.

5. VERBALS

Trong tieng Anh co mot nhom cac t ng kho cho


chung ta xep thanh t loai, nhng chung rat hu ch
trong viec giup chung ta dien at y tng. Nhom t
ng nay trong giong nh ong t va c tao thanh
t ong t ma ra, nhng lai c dung nh mot t loai
khac, va chung c coi la verbals.

nh ngha:
Verbal la hnh thai cua mot ong t c dung nh
mot t loai khac (another part of speech).
Co 3 loai verbal trong tieng Anh: (1) the gerund, (2) the
infinitive, (3) the participle.

Moi verbal co nhng ac iem khac nhau:


Gerund c dung nh mot danh t.
Infinitive co the c dung nh danh t, tnh t hoac
trang t.
Participle c dung nh tnh t.

V verbal do ong t ma ra, nen co the c hoan


tat y ngha bi mot object hoac subject complement
(predicate word).
Cung giong nh ong t, mot verbal co the c
bo ngha bi trang t.

5.1 THE GERUND

Le Th Thuy Loan Trang 50


Phng Phap Dch VIET ANH - NG PHAP

Gerund la hnh thai cua mot ong t c dung


nh danh t, v the ma ta dch gerund la danh ong t.
Cac gerund eu tan cung bang ing (going, working,
taking). Gerund c dung e goi ten mot hanh ong
hn la dien at mot hanh ong.
V gerund c dung nh danh t nen co the co moi
chc nang cua danh t trong noi: chu ng, tan ng
cua ong t, tan ng cua gii t (object of a
preposition).
Running is good exercise (subject of verb)
Jane enjoys cooking (object of verb)
I like her singing (object of verb)
His constribution to the show was his singing. (subject
complement)
S ong gop cua anh ta cho buoi dien la viec ca
hat.
Frank delighted us with his singing (object of preposition)
After eating, relax for a while (object of preposition)

5.2 THE INFINITIVE


Infinitive (ong t nguyen mau) la hnh thai cua
ong t c dung nh mot danh t, tnh t hoac trang
t.
Infinitive thng co trc no t TO ma ta hay goi la
dau hieu cua ong t nguyen mau (the sign of the
infinitive).
To know is nothing at all; to imagine is everything
(biet khong la g ca; tng tng mi la tat ca)
John wanted to shout (noun/ object of wanted)
His promise was to wait (noun/ subject complement)
His order, to withdraw, were carry out. (to withdraw c
dung nh danh t, co chc nang ong v)
(menh lenh cua ong ta, rut quan, a c thi
hanh)
This is the book to read. (to read c dung nh tnh t,
bo ngha cho book)
The car to drive is the new TOYOTA. (adjective)
John plays cards to win. (adverb)
The music is good to hear. (adverb, bo ngha cho good)

Le Th Thuy Loan Trang 51


Phng Phap Dch VIET ANH - NG PHAP

A good paragraph is not hard to write. (adverb)


The fire was dreadful to watch. (adverb)
They dived to get pearls. (adverb, bo ngha cho dived)

Qua cac th du ben tren, ta thay khi infinitive c


dung nh danh t, no se xuat hien v tr cua chu
ng, tan ng hoac bo ng va se tra li cho cau hoi
vi what. Khi infinitive c dung nh tnh t, no se
xuat hien sau danh t ma no mo ta va se tra li cho
cau hoi which one hoac what kind.
Khi infinitive c dung nh trang t, no c at sau
tnh t, hoac sau ong t Mau C Ban so 1, hoac
no co the nam sau object Mau C Ban so 5.

THE PARTICIPLE
Participle, ma ta thng dch la phan t co hai
dang:
(1): Present Participle (hien tai phan t) luon luon
tan cung bang ing (working, asking, going)
(2): Past Participle (qua kh phan t) thng tan
cung bang cac tiep - v - ng sau: -ed, -en, -d, -t, -n
(worked, asked, eaten, found, left, born, etc...)
Participle la hnh thai cua ong t c dung nh
tnh t. Trong cac th du sau ay, participle la mot yeu -
to - mo - ta.

The singing waiter spilled the soup.


The laughing boy chased the barking dog.
Running, she managed to reach the station on time.
Running water purifies itself.
(dong nc chay t loc lay no)
Dont buy a used car.
Linda will never forget a broken promise.
Frightened, the driver couldnt explain the accident.
(kinh hai, ngi tai xe khong the giai thch tai nan)
The frightened driver, couldnt explain the accident.
(ngii tai xe kinh hai khong the giai thch tai nan)
The bombed city has been rebuilt.
(thanh pho b anh bom a c xay dng lai)

Le Th Thuy Loan Trang 52


Phng Phap Dch VIET ANH - NG PHAP

Participle co the ng sau t ma no bo ngha:


The city bombed was an industrial one.
(thanh pho b doi bom la thanh pho cong nghiep)
An old hat, faded and torn, rested on his curly hair.
(chiec non cu, bac mau va rach nat, nam tren
mai toc xoan cua anh ta).
The little girl, trembling like a leaf, began to cry.
(co be, run nh chiec la, bat au khoc)
The workers sacked began to make trouble.
(cac cong nhan b sa thai bat au gay roi)

Exercise 5A: Identifying gerund


Underline the gerunds in the following sentences. Decide
wherether each gerund is serving as a subject, an object of a
preposition, or a predicate noun (subject complement)
Gach di gerund trong cac cau sau ay. Cho biet
chc nang moi gerund trong cau (chu nghu, tan ng,
tan ng cua gii t hoac bo ng)
Ex: Dancing was her chief release from boredom with her
life. (subject)

Mrs. Perkins never seemed to tire of complaining.


The whispering of the students drew Mr. Hardins attention.
Skiing continued until late June that year.
The men passed the time by talking and sleeping.
Toms quitting caused a furor at home.
Arguing will not change their opinions.
Marys lack of education is compensated by her
understanding.
The rigging of church bells commemorated the day of
liberation.
The factor that aided most in his recovery was his dieting.
For hoseback riding, golfing, and swimming, come to my
uncles ranch.
Swimming is good exercise.
He has been giving us advise on deer hunting.
Exercise 5B: Identifying Participles

Le Th Thuy Loan Trang 53


Phng Phap Dch VIET ANH - NG PHAP

Underline each participle in the following sentences and


draw an arrow from it to the word it modifies.

Ex: The seats are broken. He also paid for the broken
glass.
Dr. Stones assurance is encouraging.

A young architect explained the reason for the curving lines


of the roof.
Aunt Marys home remedy brought immediate relief to
Toms burning skin.
The tinkling sound of bells drifted toward us in the morning.
To a mature personsuch a defeat is onlytemporary
depressing
New recoursed of enegy are quite promising.
One of the injured passengers hailed a passing automobile.
Her delayed arrival disapointed everybody.
A hurried inspection of the garage failed to reveal the stolen
motorcycle.
The roaring flames could be seen in the distance.

Mr. Ropert an advance student answered our queries.


A thousand vanquished warriors returned to their ravaged
villages.
These earnings are designed for women with pierced ears.
The crying baby threw the spoon on the floor.
The stolen money hasnt been found yet.

Exercise 5C: Identifying infinitives


Underline each infinitive in the following sentences. In the
blank at the right indicate wherether the infinitive is a noun, an
adjective, or an adverb.

Le Th Thuy Loan Trang 54


Phng Phap Dch VIET ANH - NG PHAP

Gach di moi infinitive trong cac cau sau ay.


khoang trong ben phai, ghi t loai ma infinitive c
dung (danh t, tnh t, trang t)
Ex: Mr. Breggs wanted to apologize.
noun

The original owner refused to leave


Finally the representatives decided to compromise.
To delay was a serious mistake.
The most important consideration is to survive.
Horace Bush is not a man to forget.
The prisoners disguised themselves to escape.
Soon they didnt have a dollar to spend.
She sat up to talk.
The advantage to emphasize is the lower cost of living.
His chief purpose in coming here was to rest.
He said he had a condition to impose.
She had to win.
The people rose to their feet to applaud.
On Satuday there will be only one oackage to deliver.
Maria was the first to come and the last to leave the library.

Sau khi biet qua ve verbals, cac bai hoc ke tiep se


noi ve cac cum t c dan au bi verbals ma ta goi
la verbal phrases.
V co 3 loai verbal, nen ta cung co 3 loai verbal
phrases la:
The gerund phrase, ky hieu la V-ing group. Ky hieu
nay cung dung cho Participial phrase khi cum t nay co
chc nang cua tnh t.
The infinitive phrase, ky hieu la To-group.
3. The participial phrase, ky hieu la V-ed group.
6. THE V-ING GROUP

nh ngha:
V-ing group la mot cum t bat au bang mot dang
ing cua ong t (going, talking, working). Sau ay la cac
th du ve V-ing group.

Le Th Thuy Loan Trang 55


Phng Phap Dch VIET ANH - NG PHAP

V-ing V-ing group


sending sending an astronaut into
space.
eating eating his breakfast quickly.
driving driving his car to the city.
moving moving around the earth.
watchin watchig television at night.
g
listenin listening to music thousands
g of miles away.
dancing dancing when you are tired.
being being honest in business.

Nhng ta cung biet rang gerund va present participle


eu giong nhau ve hnh thc cau tao, o la mot dang
ing do ong t ma ra. Ta se phan biet chung bang
chc nang ng phap.

Mot V-ing group co the c dung nh danh t hoac


tnh t trong cau noi. Neu no c dung nh danh t th
cum t nay c goi la gerund phrase. Neu no c
dung nh tnh t th cum t nay c goi la participle
phrase. Ta lan lt xet hai chc nang cua cum t nay.

THE V-ING GROUP AS NOUN (THE GERUND PHRASE)


bai hoc trc ta a noi ve gerund va ta cung
biet rang gerund co ca hai t cach, va danh t va
ong t. Vi t cach danh t, no co the c bo
ngha bi tnh t. Vi t cach ong t, no co the c
bo ngha bi mot trang t. V gerund do ong t ma ra,
nen ng nhien no co the dan theo sau no mot object
hoac complement. Gerund cung co the c bo ngha
bi mot cum t (a phrase) hoac mot menh e (a
clause), va menh e nay c coi nh la thanh phan
cua V-ing group.

Sau ay ta quan sat vai hnh thc cau tao cua


gerund phrase tng oi dai:

Le Th Thuy Loan Trang 56


Phng Phap Dch VIET ANH - NG PHAP

his coming late for the party (his: tnh t, bo ngha cho
gerund)
studying when you watch television. (menh e bo
ngha cho gerund).
loud talking in the bus (loud va in the bus cung bo
ngha cho gerund)

Chc nang danh t cua gerund phrase


Gerund phrase luon c dung nh danh t. Vi chc
nang cua danh t, gerund phrase co the ng lam chu
ng, tan ng, bo ng, tan ng cua gii t, hoac
ong v cach (the appositive).

The gerund phrase as Subject:


(Gerund phrase lam chu ng)
cac th du di ay, gerund phrase la chu ng
cua ong t.

Visiting Ha Long Bay is my wish. (pattern 4)


Tham Vnh Ha Long la niem ao c cua toi.
Painting the entire house will take a week. (pattern 5).
Sn toan bo can nha nay se mat mot tuan.
Exploring tropical jungles is dangerous.
Tham hiem rng gia nhiet i rat nguy hiem.
Studying when you watch television is not effective.
Hoc ang luc xem ti vi khong hieu qua.
Picking apples in the fall is an enjoyable task.
Hai tao vao mua thu la viec lam thu v.
The gerund phrase as Object:
(Gerund phrase lam tan ng)
cac th du di ay gerund la tan ng cua
ong t.

Tom enjoys reading scientific books.


Tom thch oc sach khoa hoc.
We are considering inviting friends to a party.
Chung toi ang tnh mi ban be d tiec.
You must continue making efforts.
Ban phai tiep tuc co gang.

Le Th Thuy Loan Trang 57


Phng Phap Dch VIET ANH - NG PHAP

The scoutmaster tried stating a fire without matches.


Ngi hng ao th nhom la ma khong dung
diem quet.
She cant bear seeing the children cold and hungry.
Trong thay cac chau nho oi lanh, co khong chu
noi.

Cac ong t thng dan theo sau no mot gerund


phrase lam tan ng la: stop, finish, delay (tr hoan), enjoy,
suggest (e ngh), regret (hoi tiec), admit (tha nhan),
deny (phu nhan), avoid (tranh), consider (xet, cu xet),
practise, postpone (hoan lai), cant bear (khong chu c),
begin, commence (bat au), continue, hate intend (d nh),
like, dislike, prefer (thch hn), start...

The Gerund phrase as Object of preposition:


(Gerund lam tan ng cua gii t)
Trong cac th du sau gerund phrase la tan ng cua
gii t.

Man has succeeded in landing on the moon.


Con ngi a thanh cong trong viec o bo len
mat trang.
Before writing a composition, you should prepare an
outline
Trc khi viet mot bai luan, ban nen chuan b mot
dan bai.
We are all familiar with your thinking on this question.
Chung toi eu quen vi cach ngh cua ban ve van
e nay.

He left without saying a word.


Anh ta ra i ma khong li t biet.
The chief delight of going away from home is the joy of
getting back again.
Niem vui chnh cua viec i xa la cai thu luc tr
ve.

The Gerund phrase as Subject Complement:

Le Th Thuy Loan Trang 58


Phng Phap Dch VIET ANH - NG PHAP

(Gerund phrase lam bo ng)


Ta nh lai rang Mau C Ban so 4, mot danh t
giup hoan tat y ngha cho ong t to be va am ch
chu ng th danh t nay c goi la Subject complement
(hoac Predicate noun). Mot gerund phrase cung c dung
mot cach tng t.
His ambition is winning a gold medal.
Tham vong cua anh ta la at huy chng vang.
The villagers chief occupation here is fising and making
fish sauce.
Nghe chnh cua dan lang ay la anh ca &
lam nc mam
Dads fondest wish is visiting the old home village.
Niem ao c sau sac nhat cua cha la tham ngoi
nha lang que cu
Knowing all is forgiving all.
Biet het la tha th tat ca.

The Gerund phrase as Appositive:


(Gerund phrase lam ong v cach)
Gerund phrase co the c at sau mot danh t e
giai thch ro hn ve danh t o. Ta bao cum t nay co
chc nang ong v.

His occupation, testing chemical substances, is dangerous.


Nghe cua anh ta, th cac hoa chat, rat nguy hiem.
Her intention, studying medicine in Paris, pleases her
parents
D nh cua co ta, hoc y khoa Paris, lam hai long
cha me co ta.

THE V-ING GROUP AS ADJECTIVE:


(THE PARTICIPIAL PHRASE)
Nh phan au cua bai hoc nay, ta biet neu mot
V-ing group c dung nh mot danh t th ta goi cum t
nay la gerund phrase. Nhng trong tieng Anh, V-ing group
cung co the c dung nh tnh t bo ngha cho mot
danh t hoac ai t.

Le Th Thuy Loan Trang 59


Phng Phap Dch VIET ANH - NG PHAP

Vi chc nang cua tnh t, cum t nay c mang


mot ten goi khac la participial phrase. Va t ay tr
ve sau, ta nh rang participial phrase luon luon c
dung nh tnh t.
Trong cac th du sau ay, mui ten ch nh t ma
participial phrase bo ngha.
The man driving the sports car is a film star.
Ngi lai chiec xe the thao la mot ngoi sao ien
anh.

Cum t driving the sports car la yeu - to - mo - ta,


bo ngha cho man. V co chc nang cua tnh t, nen
cum t nay c goi la adjective phrase. Ta goi the la
goi theo chc nang ng phap cua no.
Mr. Harrison is a doctor specicializing in heart disease
Ong Harrison la mot bac s chuyen khoa ve benh
tim.

We saw a piano being loaded onto a truck.


Chung toi thay mot cay an piano chat len xe tai.

The satellite moving around the earth studies the


atmosphere
Ve tinh ang quay quanh a cau nghien cu bau kh
quyen.

Participial phrase co the c at bat c v tr


nao trong cau, nhng no phai at ben canh t ma no
mo ta.

Passing the ball to Romario, Bebeto danced back.


Chuyen banh cho Romario, Bebeto nhay tr ve.

Not wishing to offend him, I kept silent.


Khong muon cong kch anh ta toi gi yen lang.

Never stopping to rest, the men plodded on.


Khg bao gi ngng lai ngh ngi, oan ngi tiep
tuc le bc.

Le Th Thuy Loan Trang 60


Phng Phap Dch VIET ANH - NG PHAP

Hoping to get a ticket, Harry stood in line.


Hy vong mua c ve, Harry ng xep hang.

THE TENSE OF PARTICIPLES (Th cua phan t)


Participles co vai th nh sau:

Present Participle: doing, finishing


Past Participle: done, finished
Perfect Participle: having done, having finished.
Passive Perfect Participle: having been done, having been
finished.

Present Participle va Past Participle ch mot hanh ong


hoac mot trang thai xay ra hoac ton tai cung thi
iem vi hanh ong (hoac trang thai) cua ong t
chnh.
Passing the ball to Owen, Shearer danced back.
Giao bong cho Owen, Shearer nhay lui lai.
(2 hanh ong xay ra ong thi).
Turning on the right, I saw Tom.
ang luc queo phai, toi nhn thay Tom.
Entering the hall, the President waved to the journalists.

Perfect Partciple (having done, having finished) ch hanh


ong hoac trang thai xay ra trc (sm hn) hanh
ong cua ong t chnh.
Having passed the ball to Owen, Shearer danced back.
Giao bong cho Owen xong, Shearer nhay lui lai.
Having bought a ticket, Jack went into the stadium.
Mua xong chiec ve, Jack bc vao san van ong.
Having done all the exercises, Mary began to listen to the
radio.
LUYEN DCH: Using tenses of Participles
Viet xong quyen sach ban chay nhat, nha van bat
au viet mot v kch.
Having written a best seller, the novelist began to write a
play.

Le Th Thuy Loan Trang 61


Phng Phap Dch VIET ANH - NG PHAP

Sau khi b c tri t choi, ong ta tr lai the gii kinh


doanh.
Having been rejected by the voters, he returned to the
business world.
Hoan tat xong qu ao th 2, ve tinh gi ve cac tn
hieu vo tuyen.
Having finished the second orbit, the setellite sent back
radio signals.
Sau khi c bao trc, vien hoa tieu chuan b ap
khan cap.
Having been forewarned, the pilot prepared for an
emergency.
Hagi la cau thu hien ang a mot ng chuyen.
Hagi is the player now passing the ball.
Sau khi nhan hoi lo, vien canh sat b sa thai.
The policeman, having accepted a bride, was dismissed.
Ky xong ban hp ong, nha phat minh va nha san
xuat i New York.
Having signed a contract, the inventor and manufacturer
went to New York.
Hoan tat xong hai tuan le hoc tap, Mary cam thay
co a san sang cho ky thi.
Sau khi a kiem c tien, cau ta muon chi tieu tuy
thch.
Thang xong giai vo nh Ha Lan, oi bong a bau
Gullit lam oi trng.
Sau khi chinh phuc nh nui cao nhat hanh tinh.
Edmund Hillary c N Hoang Elizabeth phong tc hiep
s (phong hiep s: to fnight).

Exercise 6A: Using the gerund phrase


Find the complete gerund phrase in these sentences and
tell how each phrase is used (subject, object of a verb, object of
a preposition, subject complement, or appositive)
Tm gerund phrase cac cau sau ay va cho biet
chc nang cua moi cum t.
Ex: Listening to her endless talk drives me mad

Le Th Thuy Loan Trang 62


Phng Phap Dch VIET ANH - NG PHAP

(Subject of drives).

Swimming too soon after a heavy meal is dangerous.


Chinese law forbid the shooting of white bears,
Reading good books makes our life more pleasant.
The dog wouldnt stop barking at the mailman.
Telling a joke effectively requires preparation.
Lets try discussing the matter at the next meeting.
You cant start the dishwater simply by pusing this button.
I prefer jogging in the morning to swimming in the
afternoon.
He left without asking my permission.
An important skill in oratory is knowing when to stop.
Before leaving the house, he locked all the windows.
She is clever at imitating others.
The natives chief occupations are weaving straw hats and
raising cacao.
Clearning up after a party is often harder work than getting
ready for a party.
Dancing when you are tired is not fun.
Being provider for a family is my responsibility.
My father enjoys writing sports events for the Time.
Betty got a ticket for parking too long.
Directing a play is a complex job.
Jims father was delighted at his winning the first prize.

The course requires reading many books.


Complaining about the weather does no good.
A busy person can often find relaxation in gardening.
Brushing your teeth will not necessarily prevent decay.

Exercise 6B: Noun or Adjective?


Underline the complete V-ing group. Decide whether each
phrase is a gerund phrase or participial phrase.
Gach di V-ing group. Cho biet cum t la gerund
phrase hay participial phrase.
Ex: Turning left at the traffic light was our mistake.
(Gerund. phrase)

Le Th Thuy Loan Trang 63


Phng Phap Dch VIET ANH - NG PHAP

The man driving the tractor has a dangerous job.


Testing jet engines is dangerous.
The setellite orbiting the earth studies the atmostphere.
The boss considered dismissing the trouble markers.
The villagers here make a living by raising bananas.
We have pictures showing the far side of the moon.
Hoping to get tickets, we stood in line.
Scientists can use radar for forecasting the weather.
The man earned 5 dollars a day by selling gas for a filling
station
Driving to the city, the young man hoped to see manager
for an interview.
Man has succeeded in conquering the bottoms of the sea.
Having lost the original orbit, the astronaut radioed signals
to the station on earth.
The women sorting potatoes are paid on hourly rate.
The machine is used only in harvesting the crop.
Joe is capable of earning high marks.
Climbing Mont Blanc was a memorable experience.

You can disagree without being disagreeable.


Having broken her ankle. Mary missed the dance.
Talking in the audience disturbs the speaker.
Standing quietly, Mr. Peter waited for the crowd to quiet
down.

Exercise 6C: The tence of participle.


Correct the errors in the following sentences
Sa loi trong cac cau sau ay.
Ex: Working hard all day, Jim was exhausted by
evening.
Correct: Having worked hard all day, Jim was exhausted by
evening.

Studying a week, Mary felt she was ready for the test.
Reading about the new construction work, Tom applied for a
job

Le Th Thuy Loan Trang 64


Phng Phap Dch VIET ANH - NG PHAP

Exploring the cave, the scientists returned to camp.


Buying a ticket, Jack went into the stadium.
Missing the first act, Mary did not enjoy the play.
Reaching the wall, he climbed it and jumped to safety.
Winning the match, the team left the field in high spirits.
Signing the treaty, the President announced the nuclear
weapons ban.
Having been clumsy, the waiter spilled the soup.
Being famous, the actor adopted a suprerior air.

Exrcise 6D: Using the gerund phrase as Noun.


Luyen dch: Dung cau truc V-ing group e dch
phan (a) roi a vao cau noi hoan chnh phan (b)
hoac (c).

1a. Ch ao oi bong a.
Leading a football team.
b. Ch ao oi bong a la cong viec rat
phc tap.
Leading a football team is complicated work.
c. Platini a ngng ch ao oi bong a Phap.
Platini has stopped leading the French team.
2a. Khao sat tai nguyen thien nhien tren mat
a cau.
b. Chc nang cua ve tinh la khao sat tai
nguyen thien nhien tren mat a cau. (tai nguyen
thien nhien: natural resources, khao sat: to study).
3a. Hoc ang luc xem ti vi.
b. Hoc ang luc xem ti vi thng khong hieu
qua.
4a. Giai nhng bai toan kho.
b. Ban toi thch giai nhng bai toan kho.
5a. Hoc y khoa.
b. Hoc y khoa la niem ao c cua co ay.
c. Ban toi khong thch hoc y khoa.
6a. An chiec nut o nay. (an: to push)
b. Ban co the cho may chay bang cach an
chiec nut o nay.
7a. Bi trc ba an sang.

Le Th Thuy Loan Trang 65


Phng Phap Dch VIET ANH - NG PHAP

b. Bi trc ba an sang rat tot cho sc khoe


cua ban.
8a. a mot nha phi hanh len sao hoa/ mang
ong ta tr ve.
b. a mot nha phi hanh len sao hoa va mang
ong ta tr ve la ieu co the c.
9a. Chien au chong ke thu/ bao ve at
nc.
b. Chien au chong ke thu va bao ve at
nc la nhiem vu cua thanh nien thi chien. (bao ve:
to protect, to defend).
10. Ve va viet lach la nhng nghe thuat
oi hoi tai nang va s kien tr.
11a. Lam o nhiem moi trng (to pollute the
envoronment).
b. Chung ta luon luon tranh lam o nhiem moi
trng.
c. nhieu quoc gia tai Chau Au va Chau
My, luat phap cam lam o nhiem moi trng.
12a. Viec co ay mi toi dung buoi toi (her x
gerund phrase)
b. Viec co ay mi toi dung buoi toi a lam
ai cung ngac nhien.
13a. La mang tre con. (to scold: la mang).
b. La mang tre con chang ch li g.
14a. Quyet nh nhanh chong. (to make prompt
decisions)
b. Quyet nh nhanh chong rat can thiet trong
kinh doanh.
15. Kien tr sau khi ban that bai la s trac
nghiem ve nhan cach.
16a. Chi tieu nhieu hn ban kiem c.
b. Chi tieu nhieu hn ban kiem c la con
ng ngan nhat dan en ngheo kho.
17a. Truyen tai dong ien ma khong dung day
dan.
b. Cac nha khoa hoc ngay nay luon m tng
en truyen tai dong ien ma khong dung day dan.

Le Th Thuy Loan Trang 66


Phng Phap Dch VIET ANH - NG PHAP

(truyen tai dong ien: to transmit power, m tng en:


to dream of).
18a. oc nhng li ch dan khong can than.
b. Loi cua co ay la oc nhng li ch dan
khong can than.
19. Leo nui Sapa a cho toi mot kinh
nghiem ang nh.
20. ieu hanh mot trai he la mot trach nhiem
nang ne. (ieu hanh mot trai he: to run a summer camp,
trach nhiem nang ne: a heavy responsibility).

Exercise 6E: Identifying Paticipial Phrases


Underline each participial phrase. Draw an arrow from the
phrase to the word it modifies.
Gach di moi participial phrase. Dung mui ten cho
biet t ma no bo ngha.
Ex: Who knows the boy swimming in the pool?

Being an aviator, Tom enjoyed Wind, Sand and Stars.


She stood at the window, waiting for the postman.
Swinging the racket, Don approached the tennis court.
Figo is the player now passing the ball.
Having heard the signal, the runners pounced off the
starting mark.
Having arrived late, Teff had to stand.
Knowing what you need, you can shop more wisely.
Knowing the exact route, Joe didnt get lost.
I worked until twelve oclock, hoping to finish the report.

The two nurses, having finished their work, left the ward.
The audience grew restless waiting for the show to begin.
Pushing hard, he managed to move the trunk an inch.

Exercise 6F: Using the participial phrase as Adjective.


Dung cau truc V-ing e dch phan (a) roi a vao
cau noi phan (b)
Ex: a. ng gan xe may cua cau ta./ Standing
near his motorcycle

Le Th Thuy Loan Trang 67


Phng Phap Dch VIET ANH - NG PHAP

b. Ngi sinh vien ng gan xe may cua cau


ta la mot nha vo ch bong ban.
The student standing near his motorcycle is a ping-pong
champion.

1. a. ay la nhng bc anh
ay la nhng bc anh trnh bay toan bo Vnh Ha
Long.
2. a. Ngh (rang) mnh se chet
b. Ngh mnh se chet, Roy William muon hoi
hng.
3. a. Hy vong tm c viec lam thanh
pho
b. Hy vong tm c viec lam thanh pho,
chang thanh nien ri bo lang que.
4. a. Hoan tat xong quy ao th sau
b. Hoan tat xong quy ao th sau, Gagarin bat
au gi cac tn hieu vo tuyen ve tram kiem soat tren
mat at (tn hieu vo tuyen: radio signals).
5. a. Tha bom xong (having dropped bombs)
b. Tha bom xong, chiec phan lc c F5 quay i
lap tc va xa het toc lc.
6. a. Lay nhiet o cua cau be
b. Lay nhiet o cua cau be, ngi y ta bat
au san soc cau ta.
7. a. Tap hp binh ma (rallying his foot soldiers
and horses).
b. Tap hp binh ma, Le li chien au va ca
thang quan thu Chi Lang. (ca thang: to sweep to
victory over, to defeat completely).

8. a. Ch rau qua cho cong ty


Ong Lee, ch rau qua cho cong ty, lanh lng cao.
9. a. Quyet nh tr thanh nha ngoai giao
b. Quyet nh tr thanh nha ngoai giao, Erick
bat au hoc ngoai ng quyet liet.
10. a. Nhng ngon oi xuat hien ang xa
b. Nhng ngon oi xuat hien ang xa gi
Tom nh lai ngoi lang que thanh bnh.

Le Th Thuy Loan Trang 68


Phng Phap Dch VIET ANH - NG PHAP

11. a. Mang ro hoa


b. Mang ro hoa, Alice bc len san khau.
12. a. Suy ngh ang luc moi ngi chi
b. Suy ngh ang luc moi ngi chi, Henry a
giai xong bai toan.
13. a. oc xong nhng ieu ch dan
b. oc xong nhng ieu ch dan, Elma co rap
chiec loa.

7. THE V-ED GROUP


(THE PARTICIPIAL PHRASE)

V-ed la ky hieu cua qua kh phan t (the past


participle) chung cho ca ong t qui tac va ong t bat
qui tac (the irregular verb).

nh ngha:
V-ed group la cum t bat au bang mot qua kh
phan t va c dung nh mot tnh t. Cum t nay
con c goi la participle phrase.

Le Th Thuy Loan Trang 69


Phng Phap Dch VIET ANH - NG PHAP

Cung giong nh cum t bat au bang present


participle ma ta ky hieu la V-ing group, mot V-ed group
cung la yeu - to - mo - ta cua mot danh t.

Sau ay la nhng th du trnh bay cau tao cua V-ed


group hay participial phrase.

V-ed V-ed group (participial


group)
visited visited by many tourists.
killed killed in the war.
surround surrounded by mountains.
ed
attacked attacked while it was
crossing the ocean.
bombed bombed heavily in the war.
seen seen from the moon.
made made in Japan.
found found everywhere in the
nature.
built built more than fifty years
ago.

CHC NANG:
V-ed group luon luon c dung nh tnh t e bo
ngha cho mot danh t hoac ai t. Vi chc nang cua
tnh t, cum t nay c goi la adjective phrase.

The dog killed by the villagers was mad.


Cum t killed by the villagers bo ngha cho danh t
dog.

Seen from the moon, the earth looks like a giant ball.
Nhn t mat trang, trai at trong giong nh qua
bong khong lo.

Le Th Thuy Loan Trang 70


Phng Phap Dch VIET ANH - NG PHAP

Kerosene and gasoline atr two liquid fuels obtained from


petroleum.
Dau hoa va dau xang la hai nhien lieu long lay
c t dau tho.

Fossils are remains of living organisms buried in rock.


Vat hoa thach la xac cua cac sinh vat b chon
vui trong a.

These houses, built before 1900, are still sturdy.


Nhng can nha nay, c xay trc nam 1900,
van con kien co.

The fertilizer used in the experiment was high in


nitrogen.
Phan bon dung trong cuoc th nghiem co ham
lng nit cao.

Stopped at the gate, the visitor had to show his card.


B chan lai cong, khach phai trnh the chng
minh.
(cum t bo ngha cho visitor).

V-ED GROUP OR V-ING GROUP


V-ed la ky hieu cua Past Participle va V-ing la ky
hieu cua Present Participle. Nhng ta a biet ca hai cum
t V-ing group va V-ed group eu co chc nang cua tnh
t va cung c goi la Adjective Phrase. Van e at ra
la s khac nhau gia chung nh the nao.

Ta dung Present Participle la yeu - to - mo - ta cho


mot danh t vi y chu ong, ngha la danh t c
mo ta thc hien hanh ong c bieu th bi Participle.
Ngc lai ta dung Past Participle khi y ngha co tnh b
ong.

Le Th Thuy Loan Trang 71


Phng Phap Dch VIET ANH - NG PHAP

The tiger killed by the hunter weighed 120 kilos.


Con ho b ngi th san giet nang 120 kilo.
The tiger killing the hunter has been trapped.
Com ho giet ngi th san a sap bay.
The man awarding the scholarship is a millionaire.
Ngi cap hoc bong la nha trieu phu
This scholarship, awarded annually, is worth 10,000
dollars.
Hoc bong nay, c cap hang nam, tr gia 10.000
o la.
The forces attacking the city have withdrawn.
Cac lc lng tan cong thanh pho a rut lui.
The ship attacked last Sunday was an oil tanker.
Con tau b tan cong Chu nhat roi la mot tau
ch dau
The city bombed heavily yesterday is a port city.
Thanh pho b oanh tac nang hom qua la thanh
pho cang.

Exercise 7A: Identifying V-ed group


Undeline each participial phrase in the following sentences.
Draw an arrow from the phrase to the word it modifies.
Gach di moi V-ed group trong cac cau sau ay.
Dung mui ten ch ra cum t ma no bo ngha.
Ex: The money taken from the desk was returned.

Written hurriedly, the letter contained many errors.


The street, crowded with cars, looked impassable.
Built over 80 years ago, the house is still sturdy.
Struck by lighting, the cottage caught fire
Stopped at the border, the travelers had to show their visas.
This building, once used as a library, is now a hospital.
News written yesterday is history today.

Le Th Thuy Loan Trang 72


Phng Phap Dch VIET ANH - NG PHAP

Guided by radar, the pilot kept in his lane.


We liked the film shown here last week.
Having been badly defeated in the last election, the senator
retierd to private life.
Seen through the water, a straight stick looks crooked or
broken.
We saw the mountains covered with snow.
Placed in a good position, the television set is visible to
everyone in the family.
Stung by his critics, the mayor called a press conference.
Exhausted from the climb, the hikers lay down to rest.
Neatly dressed, the children set out for school.
Armed with a knife, the hunter faced the leopard.

Exercise 7B: Using the V-ed group Adjective.


Luyen dch: Dung cau truc V-ed e dch phan (a)
roi a vao cau noi phan (b). e dch phan (a) roi
a vao cau noi phan (b).

1a. (c) chup t mot ve tinh nhan tao


taken from a man-made satellite.
b. ay la nhng bc anh c chup t ve
tinh nhan tao.
These are the pictures taken a man-made satellite.
2a. trong a Lat (grown in Da Lat)
b. Rau cai trong a Lat c xuat khau sang
Phap va Spore.
Vegetables grown in Da Lat are exported to France &
Spore
3a. (c) bat au nam 1859/ c khanh thanh
nam 1869.
begun in 1859/ opened in 1869.

Le Th Thuy Loan Trang 73


Phng Phap Dch VIET ANH - NG PHAP

b. c bat au khi cong 1859 va khanh


thanh nam 1869, kinh ao Suez la mot trong nhng
thanh tu ky thuat v ai the ky 19.
Begun in 1859 and opened 1869, the Suez Canal is one of
the great engineering triumphs in the 19th Centery.
4a. trong nha knh/ trong trong vn.
b. Hoa hong nha knh chong tan hn hoa
hong trong vi. (tan: to wilt, to fade)
5a. che tao tai Nhat Ban/ san xuat tai Chau
Au.
b. Xe o to che tao tai Nhat Ban thng re
hn o to san xuat tai Chau Au.
6a. c hng dan bi raa.
b. c hng dan bi raa, vien phi cong a
ap an toan trong thi tiet xau.
7a. do Hermmingway viet. (written by Hemmingway)
b. Quyen Ng Ong va Bien Ca, do Hemmingway
viet, rat hay.
8a. do nha may nay che tao. (produced by this
factory)
b. Cac san pham do nha may nay che tao co
chat lng tot.
(co chat lng tot: to be good quality).
9a. c tuyen chon trong so nhng nhan vien
gioi nhat.
b. ay la ba sinh vien c tuyen chon trong so
nhng sinh vien gioi nhat.
10a. do Bac s Fleming kham pha.
b. Penicilin, do Bac s Fleming kham pha, a
cu son hang trieu ngi.
11a. (c) tron vi so a va voi
mixing with soda and lime.
b. Thuy tinh c lam t cat nau chay tron
vi so a va voi.
(cat nau chay: melted sand).
12a. phong thch t cac nha tu.
(phong thch: to release, to set free)

Le Th Thuy Loan Trang 74


Phng Phap Dch VIET ANH - NG PHAP

b. Columbus a phai s dung cac pham nhan


phong thch t cac nha tu e thanh lap oan thuy
thu.
(pham nhan: a criminal , oan thuy thu: crew)
13a. tm thay de dang trong thien nhien
b. Cellulose la mo6t chat de dang tm thay trong
thien nhien trong cay coi, rau qua.
14a. c hang trieu khach du loch tham quan moi
nam.
b. Rome, Paris va Lodon la nhng thanh pho
ln Chau Au c hang trieu khach tham quan moi
nam.
15. The chien th hai, bat au t nam 1939 va
ket thuc nam 1945, a gay t vong hang trieu ngi
khap the gii.
16. ien anh la mot loai hnh nghe thuat c
nhieu ngi a thch. (loai hnh nghe thuat: am art form).
17. Chiec may bay b quan du kch anh ha la
mot may bay chien au F5.
(du kch quan: a guerilla, may bay chien au: a
fighter)
18. Shepard la nha phi hanh My au tien c
phong len khong gian. (to fire/ launch into space).
19. Con ng sieu toc do cac ky s nc ngoai
e ngh a b bac bo. (bac bo: to reject).
20. Chiec cau b pha huy bi tran lut nam roi
van cha c xay dng lai.
21. Vi hai qua bom nguyen t c tha xuong
Hiroshima va Nagasaki, Nhat ban a b anh guc.
(anh guc: to bring... to ones knees)
22. B ay lui New York, Washington dan am
tan quan qua New Jersey.
(ay lui: to repulse, tan quan: dwindling forces)

8. THE TO-GROUP
(THE INFINITIVE PHRASE)

Nh a noi phan m au va bai hoc ve


verbals, To la dau hieu cua ong t nguyen mau.

Le Th Thuy Loan Trang 75


Phng Phap Dch VIET ANH - NG PHAP

To-group la mot cum t bat au bang mot ong


t nguyen mau, v vay ma To-group c goi la infinitive
phrase. ong t nguyen mau trong mot infinitive phrase
co the co yeu - to - mo - ta cua no di dang mot
cum t hoac mot menh e va menh e nay cung la
thanh phan cua infinitive phrase o.

dang phu nh, infinitive phrase bao gom luon ca


trang t NOT cua infinitive. Sau ay la nhng th du tieu
bieu ve cach cau tao cua infinitive phrase.

To- To-group (Infinitive phrase)


infinitive
to drive to drive quickly, not to drive
fast
to work to drive his car through the
to be rain
to send to work hard for the test
to follow to be sad at the news
to study to send an astronaut into
to see space
not to follow the directions
to study while you are
watching T.V
to see the African exhibit at
the museum.

Infinitive phrase cung co khi xuat hien ma khong co


dau hieu cua infinitive la tieu t TO. o la trng hp
infinitive theo sau mot so ong t nh see, hear, watch,
notice, perceive, dare, feel, let, make, please.
I saw him leave the house.
Have you ever heard her sing in English?
What makes you think so?
Lets go and see them.
We watch him jump over the fence.

Chc nang cua infinitive phrase:

Le Th Thuy Loan Trang 76


Phng Phap Dch VIET ANH - NG PHAP

Infinitive phrase c dung nh danh t, tnh t va


trang t.

8.1 THE INFINITIVE PHRASE AS NOUN


(Infinitice Phrase dung nh danh t)
Khi c dung nh danh t, infinitive phrase co cac
chc nang nh: chu ng, tan ng, bo ng va ong v
cach.

8.1.1 The infinitive phrase as Subject


cac th du di ay, ifinitive phrase la chu ng
cua ong t trong cau.

To become a doctor is her fondest wish.


Tr thanh bac s la niem c vong sau sac
nhat cua co ta.
To sing well requires much pratice.
Hat hay oi hoi nhieu luyen tap.
To become conscious of ignorance is a step toward
knowledge.
Y thc c s dot nat la mot bc tien en tri
thc.
To live without fear is to live without caution.
Song khong biet s la song khong canh giac.
To climb the cliff before us is impossible.
Leo vach nui trc mat chung ta la ieu khong
the c.

8.1.2 The infinitive phrase as Object


c dung nh danh t, infinitive phrase co the ng
lam tan ng cua ong t trong cau noi.
We want to play tennis on Saturday morning
Cum t la object cua ong t want.
She tried to finish the reports before noon.
Cum t la infinitive phrase, object cua ong t tried.
We dedided to visit Ha Long Bay.
She likes to read scientific books.
France has started to mine the oil and minerals under the
Sahara.

Le Th Thuy Loan Trang 77


Phng Phap Dch VIET ANH - NG PHAP

Phap a bat au khai thac dau va khoang san


di sa mac Sahara.

Mot infinitive co the co chu ng cua no va c


goi la chu ng cua ong t nguyen the (the subject of
the infinitive). Chu ng nay cung la thanh phan cua
infinitive phrase. Cung nen nh rang neu chu ng cua
infinitive la mot ai t, th ai t nay luon luon dang
object form (him, her, me, us, them.)

He invited her to dance with him


Toan bo cum t la infinitive phrase, tan ng cua
invited. Her la chu ng cua infinitive.
He asked his boss to reconsider the decision.
Anh ta yeu cau ong chu xet lai quyet nh.
(His boss la chu ng cua to reconsider)
She wants us to be happy.
Us to be happly la object cua ong t want.
Never advise anyone to go to war or to marry.
ng bao gi khuyen ai i chinh chien hoac lap gia
nh.

8.1.3 The Infinitive Phrase as Subject Complement


(Infinitive phrase dung lam bo ng)
Infinitive phrase co the dung sau ong t To Be e
lam bo ng va chc nang nay c goi la Subject
Complement.

His purpose is to get more money.


Muc ch cua anh ta la kiem c nhieu tien hn.
Mikes ambition is to master three languages.
Tham vong cua Mkie la quan triet 3 ngon ng.
Now the problem is to find the best man for the position.
Van e gi ay la tm ngi tot cho chc vu.
The function of an enzyme is to speed up chemical
processes.

Le Th Thuy Loan Trang 78


Phng Phap Dch VIET ANH - NG PHAP

Chc nang cua 1 enzyme la ay manh cac tien


trnh hoa hoc.

8.1.4 The Infinitive Phrase as Appositive


(Infinitive Phrase dung lam ong v cach)
Appositive la mot t, mot cum t, hoac mot menh
e c at sau mot danh t e cat ngha, giai thch
them ve danh t o. Nh vay khi dung nh mot danh
t, infinitive phrase co the dung lam t ong v va
c goi la appositive phrase.

Mr. Greens order, to put the piano in the corner, was not
obeyed.
Lenh cua ong Green, at cay an goc, a ko
c tuan theo.
Tims job, to test chemical substances, is very
dangerous.
Cong viec cua Tim, th hoa chat, rat nguy hiem.
He has repeated his promise many times, to lend me his
car.
(To lend me his car la ong v cach cho promise.)

8.2 THE INFINITIVE PHRASE AS ADJECTIVE


(I. P dung nh tnh t)
Khi c dung nh tnh t, infinitive phrase la yeu - to
- mo - ta cua mot danh t.

This is the book to read during your trip.


ay la quyen sach oc trong suot cuoc hanh trnh
cua ban.

His intention to become an engineer is clear to us.


D nh cua anh ta tr thanh ky s th qua ro oi
vi chung toi.

Le Th Thuy Loan Trang 79


Phng Phap Dch VIET ANH - NG PHAP

The project to build another highway is being considered.


e an xay dng mot xa lo khac ang c cu
xet.

Another attemp to conquer the mountain will be made


next year. / No lc khac e chinh phuc ngon nui se c
thc hien nam ti.

I had a plan to help my friends.


Toi co mot ke hoach giup ban be.

8.3 THE INFINITIVE PHRASE AS ADVERB


(I.P dung nh trang t)
Khi c dung nh trang t, infinitive phrase c
dung e bo ngha cho mot ong t, tnh t hoac mot
trang t khac. Vi chc nang nay, no la mot yeu -
to - mo - ta.

8.3.1 The Infinitive Phrase Modifying a Verb


cac th du sau ay, infinitive phrase c dung nh
trang t, bo ngha cho ong t trong cau noi.

The workers stopped to have lunch.


Cac cong nhan ngng lai e an tra.
(To have lunch la trang t ch muc ch)

He came to kill the King, not to praise him.


Ong ta en e giet vua, ch khong phai e ca
ngi vua.
(Ca hai cum t to kill the King/ not to praise him la hai
trang t ch muc ch bo ngha cho came).

I stepped aside to see better.


Toi bc qua mot ben e xem ro hn.

8.3.2 The Infinitive Phrase Modifying an Adjective


cac th du di ay, infinitive phrase c dung
nh trang t, bo ngha cho mot tnh t.

Le Th Thuy Loan Trang 80


Phng Phap Dch VIET ANH - NG PHAP

I am happy to see you here.

The nurse is ready to call the doctor.

I was sorry not to come for the party.

We were glad to have been invited to the reception.


Chung toi rat vui a c mi d buoi chieu ai.

8.3.3 The InFinitive Phrase Modifying an Adverb


cac th du di ay, infinitive phrase c dung
nh trang t, bo ngha cho mot trang t khac trong cau.

He is working hard to pass the next test.


(to pass the next test bo ngha cho trang t hard)

Bob came too early to see her.


(To see her bo ngha cho trang t too early)

The helicopter was too late to help the wounded climbers.


Chiec trc thang en qua tre, khong giup g c
cho nhng ngi leo nui b thng.

8.4 THE TENSE OF INFINITIVES


(Th Cua ong T Nguyen Mau)
Infinitive co cac th sau ay, tat ca eu thong
dung. Lay ong t invite va take lam th du:
Active Present : to invite, to take
Passive Present : to be invited, to be taken
Active Perfect : to have been invited, to have been taken.

Th perfect infinitive c dung e ch mot hanh


ong sm hn hanh ong cua ong t chnh. Th Present
Infinitive c dung e ch hanh ong ong thi vi
hanh ong cua ong t chnh.
We are happy to know you. (same time)
We were happy to know you. (same time)
I hope to arrive before midnight. (later time)
Toi hy vong se en trc na em.

Le Th Thuy Loan Trang 81


Phng Phap Dch VIET ANH - NG PHAP

Mr. Green plans to leave tonight. (later time)


Ong Green d nh se ra i em nay.
I am sorry to have told you a lie.
I was sorry to have told you a lie.
(Perfect Infinitive ch hanh ong sm hn trong hai
hanh ong cung qua kh).

LUYEN DCH:
Hien ai cung biet ong ta la mot iep vien.
Everybody knows the man to be a spy.
Hien ai cung biet ong ta trc ay la iep vien.
Everybody knows the man to have been a spy.
Toi rat tiec a khong d c buoi tiec.
I am sorry to have missed the party.
Vao thi iem o toi rat hai long a tham quan
thu o.
At that time I was glad to have visited the capital.
Toi hoan toan ngac nhien khi thay hnh cua ban
tren bao.
I was quite surprised to see your picture in the newspaper.
Nha khoa hoc tuyen bo a kham pha ra thuoc
cha benh ung th.
The scientists claims to have discovered a cure for cancer.
Ngi ta noi con cho cua anh ta can nam ngi
roi.
His dog is said to have five people.

Chu thch:
Khong bao gi chung ta dung Past Perfect va Perfect
Infinitive trong cung cau noi.
Sai: We had intended to have met you at the
game
ung: We had intended to meet you at the game.
Sai: Jeff planned to have given you the key.
Cau tren sai v hanh ong trao cha khoa xay ra
trc d nh.
ung: Jeff had planned to give you the key.
Hoac: Jeff planned to give you the key.

Le Th Thuy Loan Trang 82


Phng Phap Dch VIET ANH - NG PHAP

Exercise 8A: In each of the following sentences, the


infinitive phrase is used as a noun. Underline each phrase and
tell how it is used in the sentence (Subject, object, subject
complement, appositive).
moi cau ay, infinitive phrase c dung nh danh
t. Gach di infinitive phrase va cho biet chc nang
(Subject, object, subject complement, appositive)
Ex: The man wanted to change some money. (object)
To work well requires concentration. (subject)

To work hard is to be happy.


We saw Hanry play outside
To cohere is to stick together
To reject the scholarship or not depends on you
To lie him is impossible.
Our decision was to postpone the game.
His job, to test chemicals, is dangerous.
Teds instruction were to whistle three times.
My advice to you is to buy more books.
My greatest desire is to travel in Europe.
Our neighbour noticed the strange man come in.
Toms wish, to become a pilot, pleases his parents.
To make money is not always easy, to keep it is neve easy.
He wanted to choose medicine.
To decorate thi room will require time and money.
To be a good conversationalist is to be a good listener.
To climb the mountain before us without a guide is unwise.

Exercise 8B: Underline each infinitive phrase, Decide how


each phrase is used in the sentence (noun, adjective, adverb)
Gach di moi infinitive phrase. Cho biet t loai cua
cum t
Ex: The former, leader was please to see his men again.
(adv.)
After his internship, he can begin to practise medicine.
(noun)

Dad wanted to see the old home town.


Dads fondest wish was to visit the old home town.

Le Th Thuy Loan Trang 83


Phng Phap Dch VIET ANH - NG PHAP

An opportunity to visit the old home never materialized.


He stood up to see better.
He didnt have a dollar to spend on books.
The people rose to applause the winning team.
Not to accept his advice was unwise.
The pilot tried to radio for assistance.
He asked permission to go with us.
He had no attention to get the summer job.
His team is sure to win the game.
The fire was dreadful to see.
The mother was sad not to see her son again.
Cocoa butter is used ti make soap and cosmetics.
The setellites function is to study the clouds.
A plan to lunch another spaceship is being discussed.
I have no exercise to do today.
The Great Wall of China was built to keep the Southern
borders of china safe.
I regret that I have only one life to lose for my country.
To give only moral support is not enough.

Exercise 8C: Using Infinitive phrase as noun


Translation: Dung cau truc infinitive phrase e dch
phan (a) roi a vao cau noi phan (b)

1a. Thanh cong trong cuoc i, to succeed in life


b. Niem ao c cua co ta la thanh cong trong
cuoc i. (P.4)
Her desire is to succeed in life.
2a. Nhan li mi cua co ta hay khong
to accept her invitatio or not
b. Nhan li mi cua co ta hay khong tuy
ban.
To accept her invitation or not depends on you.
3a. a ngi & cac thiet b nghien cu khoa
hoc xuong ay bien
b. a ngi va cac thiet b nghien cu khoa
hoc la ieu co the c. (P.3) (co the c: to be
possible)
4a. noi chuyen vi cau ta/ thch cau ta ngay.

Le Th Thuy Loan Trang 84


Phng Phap Dch VIET ANH - NG PHAP

b. Noi chuyen vi cau ta la thch cau ta ngay.


5a. thi o ky thi ti
b. Niem hy vong cua cau ta la thi o ky thi
ti.
6a. theo khoa huan luyen nay
b. Theo khoa huan luyen nay se mat t nhat 6
thang.
7a. hieu biet qua kh
b. Hieu biet qua kh giup chung ta tien oan
tng lai.
(tien oan: to predict)
8a. c chap nhan vao ai hoc Harvard.
b. Tham vong cua co ta, c chap nhan vao
ai hoc Harvard, a c thc hien. (to be realized)
9a. hoc tap tot/ lao ong tot
b. Hoc tap tot va lao ong tot la nhng viec
nho hang ngay, nhng rat quan trong oi vi cac chau,
trai cung nh gai.
10a. i tm ve ep cua thien nhien
b. Muc ch cua nha th la i tm ve ap cua
thien nhien.
11a. Cung cap anh trang cho cac nha th lang man/
cung cap muc tieu cho cac nha khoa hoc vu tru.
b. Chc nang cua mat trang la cung cap anh
sang cho cac nha th lang man, nhng ngay nay chc
nang cua no la cung cap muc tieu cho cac nha khoa
hoc vu tru.
(nha th lang main: a romanticist, anh trang:
moonlight, nha khoa hoc vu tru: a space scientist)
12a. c chon lam hoi vien cua cau lac bo
b. c chon lam hoi vien cua cau lac bo la
mot danh d
13a. uon nan d luan quan chung
b. muc ch cua bai bao la uon nan d luan
quan chung.
14a. chon khoa hoc ky thuat
b. khuynh hng cua gii tre ngay nay la chon
khoa hoc ky thuat.

Le Th Thuy Loan Trang 85


Phng Phap Dch VIET ANH - NG PHAP

Exercise 8D: Using the Infinitive Phrase as Adjective and


Adverb.
Dch cac cau sau ay ra tieng anh, bien cum t
c in nghieng thanh infinitive phrase.
Ex: Ke hoach xay doing mot nha may phat
ien mi a c chap nhan.
The plan to built a new power plant has been approved.

Co y ta san sang goi bac s.


oi bong phai anh bai mua bong nay la oi
Napoli.
(anh bai: to beat, to defeat)
Nha may c trang b them may moc hien ai
e ay manh san xuat. (ay manh san xuat: to speed
up production).
Cac nha khoa hoc rat non nong tm ra mot vac sin
h hieu chong benh SIDA.
Chung toi rat au buon ve cai chet cua ong ay.
Cong ty a tang gap oi von au t trong no lc
tang gia san xuat xe o to lam tai noi a.
(tang gap oi: to double, von au t: investment,
lam tai noi a: home-made)
Jesse lam viec ngay em e bao boc gia nh bay
mieng an cua anh ta.
(bao boc: to support)
Cac binh s rat non nong tai chiem phong tuyen
a mat.
(tai chiem: to re-capture, phong tuye: the defense line)
Luat nhng ci xe va lai xe i ben phai phat xuat
t thi Napoleon.
(phat xuat, co nguon goc t: to date back to, thi
ai Napoleon: The Napoleon age, buoc: to force)
Phi cong cua the chien th nhat khong co may
vo tuyen e chuyen va nhan thong tin. (thong tin:
message).
Se con nhieu kho khan phai vt qua va nhieu
ke thu phai anh thang.
Co ta bay to niem mong c t kiem song

Le Th Thuy Loan Trang 86


Phng Phap Dch VIET ANH - NG PHAP

Chnh phu a ap dung mot loat cac bien phap e


giai quyet nan lam phat. (mot loat: aseries, ap dung
mot bien phap: to take a measure, lam phat: inflation)
Chung ta phai giai tr quan b e song con trong
thi ai nguyen t nay. (giai tr quan b: to disarm,
song con: to survive)
Ban a khong gi li ha viet cho toi thng
xuyen hn.
(gi li ha: to keep ones promise)
Toi se ch cho ban cach nhanh nhat e giai bai
toan nay.
Cach tot nhat e lam hoi sinh mot ngi la thoi
hi thou cua ban vao phoi ho. (lam hoi sinhL to revive,
phoi: lungs).
Bc vain ly trng thanh cua Trung Quoc c xay
e gi vung bien gii pha Nam Trung Quoc c an
toan. (Bc Van Ly Trng Thanh: The Great Wall of China,
vung bien gii pha Nam: the Sounthern borders).

FINAL NOTES ON PREPOSITIONAL PHRASE


AND VERBAL PHRASE.
(CHU THCH CUOI CUNG VE CHC NANG CAC
CUM T)

Nh ta a biet nhng bai hoc trc, chc nang


cua cac cum t nh P-group, V-ed group, V-ing group, To-
group la chc nang mo ta. Ta nh cac cum t nay e
m rong cac mau cau c ban e co the dien at y
tng ro rang va phong phu hn.

Ta quan sat cau c ban sau ay ch gom co hai t:


See eagles fly. (Pattern one). Neu ta them vao cac cum
t co cau tao khac nhau, vi chc nang khac nhau,
cau tren co the c m rong dai hn, nh th du di
ay:

Before the first hard frost of the winter the sea eagles fly to
the south to make their winter homes in warmer limate.

Le Th Thuy Loan Trang 87


Phng Phap Dch VIET ANH - NG PHAP

(trc cn gia lanh khac nghiet au tien cua mua


ong, nhng con o bien bay ve phng nam e xay to
mua ong vung kh hau am ap hn).

Cau tren cha 23 t va van la cau n (the simple


sentence), trong o ta a dung cac cum t co chc
nang ng phap nh sau:
before the hard frost : P-group/ adverb
of the winter : P-group/ adjective
to the south : P-group/ adverb
to make their winter homes: To-group/ adverb
in warner climate : P-group/ adjective.

9. THE CONSTRUCTION
FOR x (PRO)NOUN x TO-INFINITIVE

ay la cau truc phoi hp (the mixed construction)


gia P-group va To-group. Cum t gom co gii t
FOR c theo sau bi mot danh t hoac ai t va tiep
theo la mot infinitive phrase.

Cau tao cua Mixed construction:

FOR (Pro)noun To-infinitive


for the to study English
for students to save money
for him to choose a
for Mary carceer
for them to earn money
for the pilots to land their
the planes
rockets to reach the moon

Cac cum t thuoc dang nay co 3 chc nang: noun,


adj. va adverb.

THE MIXED CONSTRUCTION AS NOUN


(Cau truc phoi hp co chc nang cua danh t)

Le Th Thuy Loan Trang 88


Phng Phap Dch VIET ANH - NG PHAP

cac th du sau ay, cau truc phoi hp c


dung nh danh t va la chu ng cua ong t trong
moi cau. Cac tnh t coat th nh la Subject
Complement (hoac Predicate Word). ai t IT la chu ng
tam thi cua ong t trong cau noi.

Thng ta khong noi:


For her to live this area is dangerous.

Ta thng dung chu ng tam thi IT:


It is dangerous for her to live in this area.

It Subje Subject
x to be ct
Comp
lement
1 It hard for him to live on his
was small pension.
2 It difficu for anyone to be
s lt angry with her
3 It impos for him to lead an
is sible unhappy life
4 It a for the rich to help the
is common poor.
thing
5 It a rule for drivers to keep left
s in
Hong Kong.
6 Is easy for him to find a rich
it wife?
(Mixed Construction)

Chu thch:
That kho cho ong ta song vi mon tien tr cap t
oi o.
Kho ai gian c co ay.
e cau ta keo le kiep song bat hanh la ieu
khong the c.

Le Th Thuy Loan Trang 89


Phng Phap Dch VIET ANH - NG PHAP

Ngi giau giup ngi ngheo la chuyen bnh


thng.
o la mot qui luat Hongkong e ngi lai xe i
ben trai.
Co de e anh ta kiem v giau khong?

THE MIXED CONSTRUCTION AS ADJECTIVE


(CAU TRUC PHOI HP DUNG NH TNH T)
Vi chc nang cua adjective, cau truc phoi hp la
yeu - to - mo - ta cua mot danh t.

There will be two days for the tourists to visit the capital.
Se co hai ngay e cac du khach tham thu o.
Ta at cau hoi: Which two days?
Cau tra li se la: the two days for tourists to visit the
capital. Cum t for the tourists to visit the capital co chc
nang cua tnh t, bo ngha cho danh t days.
I asked permission for them to go out.
Toi xin phep cho chung ra ngoai.

Your plan for Mr. Brown to succeed Mr. Lee seems


excellent.
Ke hoach cua ban e ong Brown ke v ong Lee co
ve tuyet vi.

There may be an opportunity for you to see the manager.


Co le se co dp e ban gap ong giam oc.

THE MIXED CONSTRUCTION AS ADVERB


(CAU TRUC PHOI HP DUNG NH TRANG T)
Vi chc nang cua trang t, cau truc phoi hp la
yeu - to - mo - ta cua ong t hoac tnh t.

I will arrange for a taxi to meet you at the station.


Toi se thu xep e xe taxi on ong san ga.
Cum t c dung nh trang t, bo ngha cho will
arrange.

The problem was impossible for them to solve.

Le Th Thuy Loan Trang 90


Phng Phap Dch VIET ANH - NG PHAP

Cum t bo ngha cho tnh t impossible.

Everyone is anxious for the holidays to come.


Cum t bo ngha cho tnh t anxious.

Exercise 9A: Identifying the Mixed Construction


Underline each mixed contruction. Tell how each is used
(noun, adjective or adverb).
Gach di moi cau truc phoi hp. Cho biet cum t
c dung nh danh t, tnh t hay trang t.
Ex: I stepped aside for the others to see better.
(adverb)

I should be sorry for you to think I dont value your advice.


I am quite willing for your brother to come with you.
Here is another book for you to read.
It was the rule for men and women to sit apart.
The soldiers hard for their country to be liberated.
Mr. and Mrs. Green are not keen for their only daughter to
mary a poor school teacher.
The children were impatient for the bus to star.
Is it easy for a rich widow to find a handsome husband?
There will be a lot of good films for us to choose.
Everybody was anxious for you to accept his proposal.
There were several letters for the manager to sign.

Exercise 9B: Translation


Dung cau truc phoi hp e dch cac cau sau ay
sang tieng Anh

ay la nhng bai tap e cac hoc sinh lam


nha.
Tam chuyen t bao e moi ngi cung oc.
Toi a mua hai ve e cha me toi i Nha Trang.
Se co vai phut ong ho e cac nha phi cong vu
tru i bo trong khong gian.
Chau Au co nhieu ai hoc noi tieng e cac
sinh vien cho.

Le Th Thuy Loan Trang 91


Phng Phap Dch VIET ANH - NG PHAP

Ke hoach e mua nha may san xuat 30.000 o to


moi nam a c chap thuan.
o la thi iem tot nhat e cac chau en
trng.
Toi se a ban quyen sach e ban oc trong suot
cuoc hanh trnh.
Cuoc th nghiem e mot ve tinh nhan tao huy
diet mot ve tinh nhan tao khac trong khong gian a
thanh cong.
Ke hoch e ong Smith thay the ong Thompson trong
cng v giam oc a lam moi ngi hai long.
Vien thuyen trng ra lenh cho binh s cua ong ta
ban con tau ch.
Cuoc dan xep e cac nha lanh ao My, Chau Au
va Nhat Ban gap nhau tai Geneve a that bai.
Chu toi co niem ao c cho con trai i hoc y khoa
Phap.
Cac nha khoa hoc van cha tm ra ni ly tng e
ve tinh ap xuong be mat sao Hoa.
10. CONJUNCTIONS
LIEN T

Lien t la t dung e noi cac t, cac cum t,


hoac cac menh e trong cau lai vi nhau.

Co ba loai lien t trong tieng Anh:


a) the Co-ordinating Conjunction
b) the Correlative Conjunction
c) the Subordinating Conjunction.

10.1 THE CO-ORDINATING CONJUNCTION


Co ba t c dung ch e noi cac yeu to ng
phap giong nhau trong cau noi va chung c goi la
Co-ordinating conjunctions va gom co AND, BUT, OR. Cac
yeu to c noi bi lien t nay phai giong nhau ve
t loai, hoac hnh thc cau tao hoac chc nang ng
phap.

We are studying history and biology (noun)

Le Th Thuy Loan Trang 92


Phng Phap Dch VIET ANH - NG PHAP

AND noi vi hai danh t


Tomatoes and onions give a unique flavor to this meat dish.
Noun
The refugee was fightened and ill.
Adjectives
Dick worked rapidly but carefully.
Adverbs
Did you caome in the morning or in the afternoon?
Prepositional phrases
You can go now or wait for Helen.
Verbs
George has moved, but Margaret will remain here.
Clauses
FOR c dung nh mot co-ordinating conjunction ch
khi co noi cac menh e. NOR c dung nh co-
ordinating conjunction ch khi nao co mot t phu nh
khac ng trc no.

I called him immediately, for I was sure that he needed


help.
He will be a good lawyer, for he has a fine mind.
Cau ta se la mot luat s gioi v cau ta co bo oc
thong minh.
Michael did not come by car, nor did any of the other boys.
Michael khong en bang xe o to, cac cau be khac
cung vay.
The airline company did not plan our trip nor make our
reservations.
Mary did not tell stories, nor did she sing songs.
Mary khong ke chuyen, co ay cung chang hat.

10.2 THE CORRELATIVE CONJUNCTION (LIEN T CAP


OI)
Mot vai co-ordinating conjunctions c dung thanh
cap va c goi la Correlative Conjunction. Chung gom
co: not only... but (also), either... or, neither... nor, both.... and,
whether... or, the more... the more.
Ex: Both the house and the garage were flooded.
Neither Tom nor Mary answered the call.

Le Th Thuy Loan Trang 93


Phng Phap Dch VIET ANH - NG PHAP

Either chemistry or physics is required.


The test was not only difficult but unfair.
Both the cups and the saucers were cracked.
Ca tach lan a eu nt.
We must decide now whether to flight or give way.
Chung ta phai quyet nh hoac chien au hoac
au hang.
The more I analyzed my behavior, the more I felt at fault.
Cang phan tch cach c x cua toi, toi cang cam
thay sai trai.
Either your money will be refunded immediately, or I will
complain.
Hoac ngi ta se tra tien cho ban ngay, hoac toi
se khieu nai.
10.3 THE SUBORDINATING CONJUNCTION
Subordinating conjunction la lien t dung e gii
thieu mot menh e phu (a dependent clause) va lien ket
menh e nay vi mot menh e chnh (an independent
clause). Chc nang chu yeu cua lien t la lam ro
moi lien he gia hai menh e. Moi lien he co the la
thi gian (time), ni chon (place), nguyen nhan (cause),
hau qua (result), ieu kien (condition), v.v...

Cac subordinating conjunctions thng dung la:

after as sinc unless where


though e
althou becau so until wherev
gh se that er
as before tha whatev white
n er
as if if tho when
ugh
as long in till whenev
as order that er

cac th du di ay menh e phu c gach


di va subordinating conjunction c ong khung.
The telephone rang as we were leaving the apartment.

Le Th Thuy Loan Trang 94


Phng Phap Dch VIET ANH - NG PHAP

The doctor asked how I felt.


Linda broke her promise when she read her last letter.
The Red Ross gives help wherever it is needed.

Ngoai ba loai lien t ke tren, ta con co hai t loai


khac cung co chc nang noi ket. Chung cung noi hai
menh e lai vi nhau. o la conjunctive adverbs va
relative pronouns.

10.4 CONJUNCTIVE ADVERBS (LIEN TRANG T)


Mot so trang t c dung e noi cac menh e
chnh lai vi nhau. Khi c dung vi chc nang nay,
chung c goi la conjunctive adverbs (lien trang t).
Cac lien trang t thng dung la:

accordingl do o neverthel tuy


y ess nhien
consequen bi otherwise neu
tly the, cho khong th
nen
furthermor va lai, then vay,
e hn na the th
hence v vay, therefore v vat,
do o do o
however tuy Yet tuy
nhien the, nhng
moreover hn also cung
na vay

10.5 RELATIVE PRONOUNS (LIEN QUAN AI T)


Mot so menh e phu cung c gii thieu bi
cac lien quan ai t (who, whose, whom, which, that).
Chung c dung e noi menh e chnh. Mot relative
pronoun thng ng lam chu ng hoac tan ng cua
menh e phu.
Ex: Mary bought the dress which she preferred.
The man who hired them gave them the direction
Here comes the man whose seat you took.
Mary is the girl whom you met at my home.

Le Th Thuy Loan Trang 95


Phng Phap Dch VIET ANH - NG PHAP

Exercise 10A: Underline the conjunctions and conjunctive


adverbs. Tell what kind each joining word is.
Ex: Kathy bought some red shoes, but she didnt like
them.
(Co-ordinating Conjunction)
I do not know either how to skate or how to ski.
(Correlative Conjunction)

Phil looked as if he had seen a ghost.


I have neither the money nor the desire to go.
She screamed till she wasblue in the face.
The Precident visited the town where he had been born.
You must have ypur receipt, or no refund will be given.
I shall do provided you promise to drive carefully.
The road was almost impassable; nevertheless, we kept
driving.
You must do whatever you think best.
Either you or I should go.
Before he started, he investigated airplane schedules.
Since his wife was without a car, he bought a Flat.
I came early; therefore, I left early.
Henry studied so that he could take advanced mathematics.
My test score are high; consequently, I should go to college.
Cape Cavaveral is the moon rockets are fired.

Exercise 10B: Find the conjunctions and underline the equal


constructions which they join.
Ex: Mary looks well and happy (2 adjective)
Ex: Edgar tried to ask the librarian, but she was busy. (2
clauses)

The speaker was bold but confused.


Lincoln was awkward in appearance bt persuasive in
speech.
She came in the morning, in the afternoon, and in the
evening.
Mary has written the report and has revised it.
He knew when to talk and when to be silent.

Le Th Thuy Loan Trang 96


Phng Phap Dch VIET ANH - NG PHAP

I walked across the highway and along the tracks.


The question was whether to stop here or the drive on to
the beach.
Neither the book nor the movie was worthwhile.
The alarm clock sounded, and Harry painfully opened his
eyes.
Neither Frank nor Ralph could find his hat and coat.
One girl said a long engagement enable a prospective bride
to discover a mans bad habits and accept them.
Her father is a professor of philosophy and social science,
and her mothe teachers French literature.
The telephone operator said thet the line was busy and that
she would ring us back.
Mr. Grudge knew where the mistake lay and how to correct
it.
The young campers sing songs and tell stories.
The decision of the judges is final and irrevocable.
Listen to my directins carefully, or you are certain to get
lost.
The tour leader told us when to leave and when to return.
The sandwiches were hamburgers and hot dogs.
He has a stern manner but a good heart.

11. THE S-GROUP


(THE SUBORDINATE CLAUSE)

S la ch au cua Subordinator. Ta dung t nay e


goi chung cac t co chc nang gii thieu mot menh
e phu (the subordinate clause) va noi menh e phu nay
vao mot menh e chnh (the main clause).
Nh vat subordinators gom co:
a) Cac lien t ma ta goi la subordinating conjunction,
co the ch thi gian, ni chon, nguyen nhan, muc ch,
v.v....

Time: as, after, before, since, until, when


Cause or reason: because, since
Comparision: as, as much as, than
Condition: if, although, though, unless, provided

Le Th Thuy Loan Trang 97


Phng Phap Dch VIET ANH - NG PHAP

Purpose: so that, in order that


b) Cac RelativePronouns nh:

Neu To la dau hieu cua ong t nguyen mau 9the


infinitive). The la au hieu cua s xuat hien mot danh
t, th Subordinate la dau hieu cua s xuat hien mot
menh e phu.

nh ngha:
S-Group la mot menh e phu bat au bang mot
Subordinator.
Sau ay la cac v du cho thay cac Subordinator va
cac S-Group tng ng:

because because he was sick


when when she came
as if as if he knew something
though though the boys are poor
that that there is no electricity
so that so that he can improve the
where situation
until where he is living
who until he arrived
whom who wants to succeed
which whom we all respect
whose which stands near the corner
as long as whose car I bought
as long as there is someone to
listen.

S-Group co ba chc nang: Adverb, Adjective va Noun.

11.1 THE S-GROUP AS ADVERB (The adverb clause)


Vi chc nang cua trang t, S-Group la yeu to
mo ta cua ong t, tnh t hoac trang t trong cau
noi. Vi chc nang nay, S-Group c goi la menh e
trang t (the adverb clause).
Tuy theo lien t gii thieu menh e phu ma ta goi
ten menh e trang t. Neu menh e c gii thieu

Le Th Thuy Loan Trang 98


Phng Phap Dch VIET ANH - NG PHAP

bang mot lien t ch thi gian nh when, before, after,


while, until, as th ta goi menh e o la menh e trang
t ch thi gian (the adverb clause of time). Sau ay ta lan
lt nhan dien cac loai menh e trang t cung vi
cac lien t gii thieu chung.

11.1.1 TIME (Menh e ch thi gian)


Conjunctions: when, before, after, since, while, until, as
She left before I could recognize her
Co ta ra i trc khi toi co the nhan ra co ay.
The mechanic wiped his hands as he walked out of the
garage.
When food is oxidized in muscle cells, physical energy is
released.
Khi thc an b oxy hoa trong cac te bao c bap,
nang lng c phat sinh.
I havent seen him since he left school.
When you see lighting, it has missed you.
Khi ban thay tia chp th no a anh trat ban roi.
The messaenger wouldnt leave until Mrs. Smith signed the
receipt./ Mai cho en khi Ba Smith ky vao bien nhan th
ngi a tin mi chu ra i.

11.1.2 MANNER (Menh e ch the cach)


Conjunctions: as, as if, as though
He writes as if he knew something
Anh ta viet nh the anh ta biet c ieu g.
Please do the work as you have been told to do it
Xin hay lam cong viec nh ngi ta bao ban lam.
The small boat bounced in the waves as if it were made of
rubber.
Chiec thuyen con trong tranh tren song nc nh
the no c lam bang cao su.

11.1.3 PLACE (Menh e ch ni chon)


Conjunctions: where, wherever
Quickly she hid the letter where no one could find it.
I shall meet you wherever you want me to.
We parted where the paths separated.

Le Th Thuy Loan Trang 99


Phng Phap Dch VIET ANH - NG PHAP

11.1.4 RESULT (Menh e ch ket qua)


Conjunctions: that, so that
He was so late that he missed the lecture.
(menh e trang t bo ngha cho trang t so)
It rained all night, so that the garden flowers were ruined.
I drove off the road so that the truck could pass.
Toi lach sat vao lo e xe tai vt qua.

11.1.5 CAUSE (Menh e ch nguyen do)


Conjunctions: because, since, as, now that, seeing that
She quit school because her mother was ill.
Co ta ngh hoc v me co ta b benh.
Since she could not pay the fine, she could not drive the
car.
V co ta khong co kha nang nop phat, co ta khong
the chiec o to c.
I neednt tell you as he has told you already.
Toi khong can noi cho ban biet v anh ay a noi
vi ban roi.
Without the farmer, all industries would be impossible
because the factory worker must eat.
Khong co nha nong, tat ca cac nen cong nghiep
khong the ton tai c v cong nhan nha may phai
an.
Now that hes left, theres nothing to do.
Bi le anh ay a ve nen khong co g e lam ca.
Seeing that we are all here, we might begin the work.
V tat ca chung ta eu co mat ay nen chung
ta co the bat au cong viec.
As he was such a fool, I refused to listen to him.
Bi anh ta la ke ngu an, toi a khong nghe anh
ay.
That needs no explaination since it is self-evident.
ieu o khong can phai giai thch v no a ro
rang.

11.1.6 PURPOSE (Menh e ch muc ch)

Le Th Thuy Loan Trang 100


Phng Phap Dch VIET ANH - NG PHAP

Conjunctions: that, in order that, so that


They died that their countrymen might live.
Ho a chet e ong bao ho c song.
I drove slowly so that we could enjoy the view.
We have to ears and only one tongue in order that we may
hear more and more and speak less.
Chung ta co hai tai va ch mot li e nghe nhieu
va noi t.
He worked hard in order that he might support his big
family.
Han lam viec nang nhoc e co the nuoi mot gia
nh ong uc

11.1.7 CONDITION (Menh e ieu kien)


Conjunction: if, unless, provide that, on condition, that
Stop me if you have heard this before.
You can return the lamp if you dont like it.
If you cant say something good, dont say something bad.
Neu ban khg the noi mot ieu g hay, th ng noi
ieu g d.
I shall go provided that you promise to drive carefully.
Toi se i mien la ban ha lai xe can than.
You may take the book tonight on condition that you return
it tomorrow.
Ban co the lay sach toi nay vi ieu kien ban
gi tra lai vao sang mai.

11.1.8 CONCESSION (Menh e ch s nhan nhng)


Conjunctions: although, thought, even if, in spite of the the
fact that.
Although he is small, he plays basket ball vey well.
Our car is dependable even if it is old.
I decided to accept the job in spite of the fact that the
salary was low. / Toi quyet nhan viec du ong lng
thap.
Though Brown has lived for five years in France, he does
not speak French well.

11.1.9 COMPARISON (Menh e ch s so sanh)

Le Th Thuy Loan Trang 101


Phng Phap Dch VIET ANH - NG PHAP

Conjunctions: as, than


Gold is heavier than iron is.
Vang nang hn sac
This test is as hard as the first one was.
(menh e trang t bo ngha tnh t hard)
The Mekong Delta produces more rice than any other
section of the country.
ong bang song Cu Long san xuat nhieu gao hn
bat c vung nao cua at nc.
Smells are surer evidence than sounds and sights (are).
Mui la chng c chac chan hn am thanh va hnh
anh.
There are not so mch people here as I expected.
ay khong co nhieu ngi nh toi mong i.

11.2 THE S-GROUP AS ADJECTIVE (THE ADJECTIVE CLAUSE)


Vi chc nang cua tnh t S-Group la yeu to
mo ta cua mot danh t va c goi la Adjective
clause. V adjective clause c gii thieu bang cac
Relative pronouns (who, whom, whose, which, that) hoac cac
Relative adverbs (where, when, why) nen menh e nay
cung con c goi la relative clause.
cac th du di ay mui ten ch danh t ma
menh e tnh t bo ngha.
Tom bought the shirt which he likes best.

Music is the art form that we all love


Nhac la mot loai hnh nghe thuat ma tat ca
chung ta yeu thch.
Beethoven, whose music you are listening to, is a fine
composer
Mr. Markus is the graduate student whose paper you liked.

The man whom you met works for a construction company.


The man who first introduced mordern mass-production
methods changed life in America.

Le Th Thuy Loan Trang 102


Phng Phap Dch VIET ANH - NG PHAP

Ngi au tien gii thieu nhng phng phap san


xuat ai tra hien ai a thay oi i song Chau
My.

May 7 is the date when I leave.

We visited the village where the great poet lived and is


burries.

The reason why the accident occurred is not known.


Ly do tai sao tai nan xay ra th cha biet.

Nay ngi ta co khuynh hng dung cac menh e


tnh t ma khong can en relative pronoun hoac relative
adverb trong trng hp relative pronoun ng lam tan
ng cua ong t.

Thay v viet Ta co the viet


The house that I live The house I live in...
in... The food we eat...
The food that we The book I am
eat... reading...
The book which I am The man you saw...
reading ...
The man whom you
saw...
Sau ay la cac v du trong o menh e tnh t co
cha mot relative pronoun c bo i.
The doctor you want is on vacation.
This is the book you need.
Is this the novel you were telling me about?
I remember the house I live in my childhood.
The book I am studying is written by famous write.

THE S-GROUP AS NOUN. (The Noun Clause)


Khi ta dung nh danh t, S-group khong phai la yeu
-to - mo - ta. Vi t cach la danh t, menh e co cac
chc nang sau ay trong cau: Chu ng, tan ng cua
ong t, tan ng cua gii t, bo ng sau ong t

Le Th Thuy Loan Trang 103


Phng Phap Dch VIET ANH - NG PHAP

Be, hoac ong v cach. Khi c dung nh danh t,


menh e c goi la Menh e danh t (The noun clause).

Nh a noi phan m au bai hoc, menh e


danh t co the c gii thieu bi cac t ma ta dung
e gii thieu mot menh e trang t nh when, where.
Nhng khi c dung nh danh t, nhng t nay khong
c coi la Subordinating conjunction, ma ch c coi nh
la trang t trong menh e danh t.

Mot cach tng t, menh e danh t co the gii


thieu bi nhng t c dung e gii thieu menh e
tnh t nh: who, whom, whose, which, that. Nhng khi c
dung trong menh e danh t, cac t nay khong c
coi nh Relative pronouns (lien quan ai t). Chung
khong thay the cho t nao trong menh e chnh ma ch
co the ng lam chu ng hoac lam tan ng trong
menh e danh t.

THE S-GROUP AS SUBJECT.


(Menh e danh t ng lam chu ng.)
cac v du di ay cac menh e danh t la
chu ng cua ong t trong cau noi.
Who invented this machine is unknown.
Ai phat minh ra may nay th cha c biet en.
That he won the prize surprises everybody.
Anh ta oat giai lam moi ngi ngac nhien.
What happened to the missing tourists was never
discovered.
Viec g xay ra cho cac du khach mat tch a khong
bao gi c kham pha.

It is a strange thing that he always comes late.


(That he always comes late la chu ng cua ong t
IS. ai t IT c dung nh chu ng tam thi).
It is unknown whether it is caused by rain or wind.

THE S-GROUP AS OBJECT.


(Menh e danh t ng lam tan ng).

Le Th Thuy Loan Trang 104


Phng Phap Dch VIET ANH - NG PHAP

cac v du di ay cac menh e danh t ng


lam tan gn cua ong t trong cau noi.
He said that there is no electricity.
I understand what he said.
Cac du khach hoi ho co thi gian e tam ra
trc ba an chieu nay hay khong.
A good teacher knows whether or not you study.
Mot giao vien kinh nghiem biet ban co hoc hay
khong.
Confucius believed that a man cannot conceal his character.
Khong t tin rang con ngi khong the che dau
ban chat cua mnh c.
The policeman states that the driver will be responsible for
damages.

THE S-GROUP AS OBJECT OF PREPOSITION.


(Menh e danh t ng lam tan ng cua gii
t.)
cac v du di ay menh e danh t la tan
ng (tuc t) cua gii t ng trc no.
Ex: I didnt agree with what he had done.
Toi khong ong y vi ieu anh ta a lam.
He always thinks of where he will live.
That depends on who is chosen.
ieu o tuy thuoc vao ai se c bnh chon.
Miss Roger can substitute for whoever is on vacation.
Co Rogers co the the cho bat c ai ang ngh he.
We gave boo lists to whoever asked for them.
THE S-GROUP AS SUBJECT COMPLEMENT
(Menh e danh t ng lam bo ng).
Ta nh lai cau tao cua Mau c ban so 4: Subject x
be x noun. Danh t dung sau To be c goi la Subject
Complement.
Mot menh e danh t co the thay the cho danh t
nay va menh e danh t cung van c coi la Subject
Complement.

The problem is where this piano should be put


Van e la cay dng cam nay nen at au.

Le Th Thuy Loan Trang 105


Phng Phap Dch VIET ANH - NG PHAP

What I want to know is how this machine works


ieu toi muon biet la co may nay van hanh nh
the nao?
The most important thing to me is why he left so suddenly
The best explanation was that the cold winds had dried out
the plants.
Li giai thch tot nhat la nhng cn gio lanh a
lam cho cay heo ua
The question was where we would go for our vacation.
Cau hoi at ra la chung toi se i ngh au?

THE S-GROUPP AS APPOSITIVE.


(Menh e danh t ng lam ong v cach)
bai hoc ve chc nang cua Gerund phrase, ta co
noi ve chc nang ong v cua cum t nay. Mot cum
t co chc nang ong v c goi la Appositive phrase.
Mot cum t co chc nang ong v c goi la
Appositive phrase. Mot cach tng tr, mot menh e danh
t co chc nang ong v c goi la Appositive clause.
cac th du di ay, cac menh e danh t co
chc nang ong v.
The fact that he is good at mathematics is not true.
S the anh ta gioi toan khong ung.
My idea that we should go early met with disapproval
The new that her son had been killed was a great shock to
her.
Tin cho rang con ba ay chet la mot cu soc ln.
He expressed a hope that you would soon be well again.
21. You cant get what you want without making an
effort.
22. The dog that barks the loudest usually bites the
least.
23. The children followed the Kitten wherever it went.
24. The man to whom you spoke is the director of the
museum.
25. There can be no doubt that he is intelligent.
26. We waited where were told.
27. Uncle Albert says that his business is steadily
improving.

Le Th Thuy Loan Trang 106


Phng Phap Dch VIET ANH - NG PHAP

28. An older man spoken up as though he were in


charge.
29. She requested a transfer so that could be with her
husband.

Exercise 11B: Using the S-group as Noun. (Menh e danh


t).
Dung cau truc S-group e dch phan (a) roi a vao
cau noi phan (b) hoac (c).

1a. Ai se lanh ao cuoc nh cong


Who will lead the strike?
b. Ai se lanh ao cuoc nh cong th cha biet.
Who will lead the strike in unknown.
2a. Rang khong co s song tren sao Hoa (that x
clause).
b. Gia thuyet cho rang khong co s song tren
sao Hoa a c chng minh.
3a. Ai se c chon la cau thu hay nhat cua
mua bonng nay.
b. Khong ngi nao co the tien oan ai se
c chon la cau thu hay nhat cua mua bong nay.
4a. Rang ien co the c truyen tai khong
can day dan.
b. Cac nha khoa hoc rat lac quan se gia
thuyet cho rang ien co the truyen tai ma khong can
day dan.
(lac quan: opitimistic (adj); hopeful).
5a. Nhng g ong ta a kham pha (what x clause).
b. Nha khoa hoc a t choi cong bo nhng g
ong ta a kham pha.
6a. Tai sao han hanh ong nh the.
b. Toi khong hieu noi tai sao han hanh ong
nh the.
7a. Co ay se chon nganh hoc nao/ nganh hoc
o se mat bao lau (what branch of study x clause/ how long
x clause)
b. Toi a hoi co ay se chon nganh hoc nao
va nganh hoc ay se mat bao lau.

Le Th Thuy Loan Trang 107


Phng Phap Dch VIET ANH - NG PHAP

8a. Ong ay se en nc nao/ se am phan


vi ai/ se thoa thuan van e g.
Which country x clause/ who x clause/ what problems x
clause.
b. Nha ngoai giao khong tiet lo ong ay se
en nc nao, se am phan vi ai va se thao luan
van e g.
9a. Ai a phat minh ra knh hien vi (the microscope)
b. Anh co biet ai a phat minh ra knh hien vi
khong?
c. Ai a phat minh ra knh hien vi, ca lp khong
ai biet
(Dung mau so 3: Subject x be x adjective)
10a. Lieu ban co muon tham gia chuyen tham quan
hay khong (Whether or not x clause).
b. Van e la lieu ban co muon tham gia
chuyen tham quan hay khong?
11. Nhng ngi me tn tin rang tieng set
la vo kh cua than thanh (me tn: superstitious (adj),
than thanh = gods)
Co that anh ta noi ba ngoai ng khong
ien la mot nguon nhiet co the c nhn thay ni
ban ui ien va bep ien.
(Dung That x clause e tao chu ng; mot nguon
nhiet = a source of heat)
Ai cung ngh rang ke hoach o khong thc tien va
no se that bai, (thc tien: practical).
Thomas Edison tin rang benh iec cua ong a lam
tang them sc tap trung cua ong ay.
(benh iec: deafness; sc tap trung: powers of
concentration).

S nghien cu lch s th can thiet cho bat c ai


quan tam en tng lai cua the gii chung ta.
(Dung menh e danh t lam tan ng cua gii t
for: To be necessary for whoever x clause)
Toi a hoi tham co ta song au, ang lam g
e kiem song va tai sao co ta ri bo thanh pho.

Le Th Thuy Loan Trang 108


Phng Phap Dch VIET ANH - NG PHAP

(Dung 3 menh e danh t: where x clause, what x


clause va why x clause)

Exercise 11C: Using the Adjective clause.


Dung mot Relative pronoun (who, whom, which, that)
hoac Relative adverb (where, when, why) e tao thanh
menh e tnh t e dch phan c in nghieng cua cac
cau sau ay:
Ex: Khong phai tat ca nhng g lap lanh eu
la vang.
Not all that glitters is gold.
Ex: Nguyen Hue la v anh hung dan toc ma
tat ca chung ta eu ton knh.
Nguyen Hue is a natinal hero (whom) we all respect.

Nhng bc anh nay ch ro cac mieng nui la


ma cha ai tng trong thay.
(mot bc anh: an aerial photograph; mieng nui la:
crater)
Nang lng ma chung ta ang s dung phat xuat
t mat tri.
Ban co u tien ban can cho chuyen i Ha Noi
khong?
Toi khong the hieu noi ly do tai sao anh ta lai noi
the.
Bc Van Ly Tng Thanh cua Trung Quoc mot
trong bay ky quan cua the gii la vat duy nhat tren
trai at ma ngi ta co the nhn thay t mat trang.
(bc Van Ly Tng Thanh: The Graet Wall of China;
mot ky quan: a wonder)
Muc tieu ma chung ta phan au la nen hoa bnh
the gii.
Chan chac cau chuyen (ma) anh ta a ke cho
chung toi khong co chut s that nao.
Phong trien lam nghe thuat trng bay cac bc
danh hoc ma ngi ta a su tap khap ni tren the
gii.
o la mot ngoi lang, ni nha th a chao i.

Le Th Thuy Loan Trang 109


Phng Phap Dch VIET ANH - NG PHAP

Nguyen to hoa hoc la mot chat (ma) khong the


tach thanh nhng chat n gian hn.
(nguyen to hoa hoc = a chemical element; n gian =
simple)
Thai Lan la mot nc duy nhat ong Nam A
cha bao gi b cai tr bi mot cng quoc Chau Au.
Nhng s that a ra anh sang lam moi ngi ngac
nhien.
Nhng ngi ang hng li nhieu nhat la nhng
nha ai cong nghiep.
(hng li: to benefit; mot nha cong nghiep: an
industrialist)
Nhieu ngi (ma) chung ta thay trong cuoc dieu
hanh la nhng cu chien binh.

Exercise 11C: Using the Subornating conjunction


Dung mot lien t dang Subordinating conjunction e
tao mot S-group phan (a) roi a vao cau noi
phan (b) hoac (c).
Ex. a. S song ch ton tai ni nao co nc.
Life exists only where there is water.
b. au co nc la o co s song.
There is life where there is water.

1a. Tr phi thi tiet toi (unless x clause)


b. Chung toi se khong i cam trai tr phi thi
tiet toi.
2a. Neu khg co mot th trng cho cac san
pham (ma) no lam ra.
b. Mot nen cong nghiep khong the phat at
neu khong co mot th trng cho cac san pham ma
no lam ra.
(phat at: thrive, prosper, flourish)
3a. Mac du chung ngheo
b. Cac em nho nay c giao duc tot mac du
chung ngheo.
4a. e anh ta co the bao boc gia nh (in order
that x clause).

Le Th Thuy Loan Trang 110


Phng Phap Dch VIET ANH - NG PHAP

b. David rat can viec lam e anh ta co the


bao boc gia nh.
5a. He khi co mot tai nan (wherever x clause)
b. He khi co mot tai nan la am ong tu tap
lai.
6a. Khi con th au
b. Khi con th au, Mozart a bieu lo mot nang
khieu am nhac.
(nang khieu am nhac: musicianship, an aptitude for
music).
7a. V chung khong co chanh ngha
b. Ke tu cua chung ta a b anh bai v
chung khong co chanh ngha (chnh ngha: righteous
cause, justice)
8. Phi c phan lc khong the bay tren (ngoai)
bau kh quyen v kh oxy trong bau kh quyen rat can
thiet e ot chay nhien lieu trong ong c.
(phi c phan lc: jet planes, nhien lieu: fuel)
9. oi vi toi hnh nh la ngay he ngan hn
trc ay nhieu.
10. Trong lc tren mat trang sau lan nho hn
trong lng tren trai at.
11. Nhng ngi song vung cao co so lng
hong cau trong mau ln hn nhng ngi song vung
thap hn.
(hong cau: red corpuscles)
12. Con cho trong co ve nh muon tan cong
chung toi.
(as if x clause)
13. Hay o chiec xe bat c ni nao ban co
the tm c cho.
14. Rat kho chup hnh hanh tinh Venus v no
thng xuyen b che bi nhng am may day ac.
(che = to veil).
15. Zidane a chiec cup bac len cao e moi
ngi nhn thay.
16. Khi cn au bat au, hay uong hai vien
aspirin.
17. Tony cung co nang lc nh ch cau ta vay.

Le Th Thuy Loan Trang 111


Phng Phap Dch VIET ANH - NG PHAP

(co nang lc: to be capable)


18. Hang trieu ngi b chng ap huyet cao v
ho an qua d tha.
19. V thuoc khong hieu qua nen cong ty dc
pham a ngo li xin loi va rut lai vac-xin.
20. Thng vu se tiep tuc cho en khi tat ca o
to c ban het.
21. Ban khong the trong ch nhan th tr phi
ban chu viet th. (unless x clause).
12. THE RULE OF PROXIMITY
(QUI TAC TNG CAN)

Trong tieng Anh, t Proximity (nearness) co ngha la


gan. The Rule of Proximity c dch la Qui tac tng can
hay Luat tng can.

Qui tac tng can qui nh rang khi mot yeu - to -


mo - ta bo ngha cho t ng nao, th no phai c
at gan t ng o, du yeu - to - mo - ta o la mot
t ng n oc (a single word), mot t co dang P-group,
V-ing group, To-group hoac S-group.

Th du:
The morning paper... ; garden flowers
The coming years ; the years to come...
A used car... (xe a qua s dung) ; cars made in
Japan.
A man of talent ; the man in the blue shirt
A large family to support : the boy siting on the bench
The engineer whom you saw
He didnt come because he was busy.

Cac cau sau ay thay oi han y ngha khi ta thay


oi v tr cua t only.
Only Gary hit his classmate on the nose
Ch co Gary anh ban cau ta vao mui
Gary only hit his classmate on the nose
Gary ch anh ban cau ta vao mui thoi
Gary hit only classmate on the nose.

Le Th Thuy Loan Trang 112


Phng Phap Dch VIET ANH - NG PHAP

Gary ch anh mnh ban cau ta vao mui


Gary hit his only classmate on the nose.
Gary ch anh ngi ban duy nhat cua cau ta vao
mui.
Gary hit his classmate only on the nose
Gary anh ban cau ta ch vao mui tho.i
Gary hit his classmate on his only nose.
Gary anh vao cai mui duy nhat cua ban.

Trng hp tren thng xay ra vi cac yeu - to -


mo - ta cua danh t. Co nhng trng hp mot yeu -
to - mo - ta co the c at gan nh bat c v tr
nao trong cau hoac trong menh e ma khong vi pham
qui tac tng can.

Verb modifier:
Slowly the bus came down the hill
The bus slowly came down the hill
The came slowly down the hill
The bus came down the hill slowly

Sentence modifier:
Consequently, he left the conference
Bi the ong ta ri bo hoi ngh.
He, consequently, left the conference.
He left the conference consequently.

Mot yeu - to - mo - ta thng c at sai v tr


trong trng hp co hai (hoac nhieu) t trong cau ma
no co the bo ngha.
Di ay la cac th du cho thay cac yeu - to - mo
- ta c at sai v tr.

We want a piano for a lady with mahogany legs.


(Mahogany: go ao hoa tam, mot loai go qu)

Cau tren co y ngha khong chnh v khong ap


dung Qui tac tng can. Cum t with mahogany legs la

Le Th Thuy Loan Trang 113


Phng Phap Dch VIET ANH - NG PHAP

mot P-group, co ch71c nang tnh t. c at v tr


tren, cum t ng nhien c coi nh bo ngha cho danh
t lady , va cau tren co ngha la mot phu n co
chan bang go.

Ap dung Qui tac tng can, ta sa lai:


We want a piano with mahogany legs for a lady.
di ay yeu - to - mo - ta la mot V-ing group
b at sai v tr.
Tom saw some high mountains sitting in the train.

Nhan xet:
Cum t sitting in the train la mot participial phrase
c dung nh tnh t. Cum t nay phai la yeu - to -
mo - ta cua chu ng Tom trong cau noi. Ap dung Qui
tac tng can, ta at yeu - to - mo - ta ke ben t ma
no mo ta.

Sitting in the train, Tom saw some high mountains.

cau sau ay yeu - to - mo - ta la mot menh


e tnh t c at sai v tr:
We rented a house near the lake that had two bedrooms.
(Cau tren co ngha la cai ho co hai phong ngu!)

Ap dung Qui tac tng can ta di v tr menh e


tnh t va sa lai cau:
We rented a house that was near the lake and that had two
bedrooms.

Chu thch:
Trong trng hp mot danh t c bo ngha bi
mot cum t (a phrase) va mot menh e (a clause), ta
thng at cum t trc menh e.
We talk to the man at the store whom we met yesterday.
(ca hai yeu - to - mo - ta la tnh t, bo ngha cho
man)

Le Th Thuy Loan Trang 114


Phng Phap Dch VIET ANH - NG PHAP

Trong tieng Anh, cac loi mac phai do at sai v tr


cua yeu - to - mo - ta c coi la Misphaced modifiers.

Ta quan sat them mot so th du:


1. Misplaced prepositional phrases
Misplaced:
John and Mary talked in while I studied in whispers.
Correct:
John and Mary talked in whisper while I studied.
Misplaced:
I saw the results of the volcanic eruption on television.
Correct:
I saw on television the results of the volcanic eruption.
Misplaced:
In a shop we saw bicycles with adjustable seats for girls.

2. Misplaced infinitive phrases:


Misplaced:
To see well, glasses should be cleaned carefully.
Correct:
Glasses should be cleaned to see well.
Misplaced:
He bought five traps after his cat died to catch mice.
Cau nay co ngha la con meo anh ta chet e bat
chuoc!
Correct:
He bought five traps to catch mice after his cat died.

3. Misplaced caluses:
Misplaced:
Peter gave the statue to a friend that was made of plaster.
Cau tren co ngha mot ngi ban lam bang
thach cao!
Correct:
Peter gave a friend the statue that was made of plaster.

4. Misplaced participial phrases:

Le Th Thuy Loan Trang 115


Phng Phap Dch VIET ANH - NG PHAP

Misplaced:
Beer must not be sold to a student containing more than
4% alcohol.
Correct:
Beer containing more than 4% alcohol must not be sold to
a student.
Bia cha hn 4% o con khong c ban cho sinh
vien.

Exercise 12: Identifying misplaced modifiers


Each of following sentences aontains a misplaced modifier.
Correct the sentence by placing the misplaced modifier in its
proper position.
Moi cau di ay cha mot yeu - to - mo - ta
c at sai v tr. Sa lai cau bang cach at yeu - to
- mo - ta vao ung v tr.

The fire was extinguished before much damage had been


by the brave fireman.
Every child awaits the time when he can go to school with
excitement.
The operation was performed before much harm was done
by the doctor.
I sold a guitar to a beautiful girl with steel strings.
My friend agreed on the next day to help me.
I continued to live at my uncles home when I went to
college to save money.
I prefer always to read scientific books.
The boys sat at the parade in the window.
For three days I almost ate nothing.
Aunt Bertha met the man she later married by accident.
Harry bought a car from a little old lady that had been
drivew less than ten thousand miles.
She wore a decoration in her hair which was made of
cherries, feathers, and wild rice.
George killed the mad dog that sprang at him with a long
wooden stick.
Jame witnessed the shoplifter take the diamond necklace
with her own yeys.

Le Th Thuy Loan Trang 116


Phng Phap Dch VIET ANH - NG PHAP

My father keeps the madals and decorations that he won


during the war of the Gulf in the buffet.
George met his wife when he seturned from an office party
on the subway.
Teachers who trust students occasionally are disappointed.
Some of the native women carried large baskets on their
heads which were filled with soiled laundy.
People who go to the beach often dont like to swim.
One summer our fruit stand was closed because we had
sold apples to customs that had worm.
All if the spectators cannot get into the stadium.
Mary was praised for her heroism by the mayor.
Dorothy was learning to drive slowly.

13. THE RULE OF PARALLELISM


(QUI TAC SONG HANH)

T parallelism trong tieng Anh co ngha la s tng


ong, i oi hoac song song. Ta dch Rule of Parallelism la
Qui tac song hanh/ Luat song hanh.
Lien t AND noi cac yeu to giong nhau trong cau
noi. No noi hai danh t, hai tnh t, hai cum t hoac hai
menh e lai vi nhau. Cac yeu to c noi lai nh the
c goi la tng ong (parallel).

The Rule of Parallelism:


Cac yeu to trong cau noi co cung chc nang ng
phap nh nhau phai c trnh bay bang nhng hnh
thc cau tao giong nhau. Hnh thc cau tao giong nhau
o co the la mot t ng n oc, mot cum t co
dang P-group, V-ing group, V-ed group, To-group hoac S-
group.

Le Th Thuy Loan Trang 117


Phng Phap Dch VIET ANH - NG PHAP

13.1 Luat song hanh va cac lien t


Subordinating conjunctions (and, but, or)
Qui tac song hanh c ap dung chat che khi ta can
dien at mot loat cac s viec, cac hanh ong hoac
cac y tng tng t va co lien quan vi nhau.

Trong cac cau sau ay ta quan sat s giong nhau ve


t loai cua cac t, ve cau tao cua cac cum t va
menh e khi chung c noi bi lien t AND.
He enjoys reading plays and poetry. (two nouns)
She sang and danced beautifully. (two verbs)
He looked in the drawers and under the beds.
(two prepositional phrases)
He is a man who has worked hard and whom we all respect.
(two adjective clauses)
He will come back to the city, and his wife will go to Hanoi.
(two main clauses)
Khi hai yeu to trong cau co cung chc nang ng
phap c noi bi lien t AND ma khong giong nhau
ve hnh thc cau tao, ta pham phai loi goi la Faulty
Parallelism.
Ta quan sat them cac cau sai v khong ap dung Qui
tac song hanh.
Faulty: The Tokyo Bank has branches in Hong Kong and
Vietnam. (cau nay sai hai trang t ni chon khong giong
nhau ve hnh thc cau tao.)
Correct: The Tokyo has branches in Hong Kong and in VN.
Faulty: We called the meeting to elect new members, for
raising money, and that we could plan for the dance.
Chung toi trieu tap buoi hop e bau hoi vien
mi, e gay quy va len ke hoach buoi khieu vu.
Cau tren sai v 3 muc ch cho cuoc trieu tap buoi
hop khong c trnh bay bang ba cau truc giong nhau.
Ta sa lai, dung 3 To-group.
We called the meeting to elect new members,
to raise money, and
to plan for the dance.

Le Th Thuy Loan Trang 118


Phng Phap Dch VIET ANH - NG PHAP

Faulty:
She likes to go to the movie, hearing chamber music, and
plays. / (co ta thch xem phim, nghe nhac thnh phong va
xem kch)
Correct: Dung 3 To-group song hanh
She likes to go to the movie
to hear chamber music, and
to see plays.
Hoac: Dung 3 V-ing group song hanh
She likes going to the movie,
hearing chamber music, and
seeing palys.

Faulty:
To have a definite time-table and following it are essential in
school. (co mot thi khoa bieu xac nh va ap dung
no rat can thiet trong nha trng)
Cau tren sai v lien t AND noi hai chu ng khong
giong nhau ve hnh thc cau tao. Ta sa lai bang mot
trong hai cach sau ay.
Cho hai T-group i song hanh:
To have a definite time-table
and
to follow it are essential in school.
Cho hai v-ing group i song hanh
Having a definite tim-table
and
following it are essential in school.

Faulty: She learned how to type, keeping books and taking


dictation in shorthand.
Correct: She learned how to type
how to keep books, and
how to take dictation in shorthand.

Faulty: My uncles is young, tall, and having black hair.


Correct: My uncle is young, (adj)
tall, and (adj)

Le Th Thuy Loan Trang 119


Phng Phap Dch VIET ANH - NG PHAP

black haired (adj)

Luyen dch: Applying the Rule of Parallelism


1. Hau het ngi ta chi quan vt e luyen
tap, giai tr va giao tiep xa hoi.
Most people play tennis for exercises,
for pleasure, and
for social contacts.
2. Xin cho toi biet ban muon i au, tai sao ban
quyet nh ra i, ban d tnh se lam g va khi nao ban
se tr ve.
Please tell me where you want to go,
why you have decided to go,
what you intend to do, and
when you will come back.

3. Chc nang cua khong quan la giang on


phan cong anh tra phi c ch, yem tr bo binh va
tuan tra bau tri cua to quoc.
The function of the Air Force.
is lauching counter-attacks against enemy planes,
supporting the Army, and
patrolling the skies of our fatherland.

4. Chung ta can mot lanh ao co u thong


minh e quyet nh chnh xac, u can am e quyet
nh tao bao, va u nhay ben e quyet nh nhanh
chong.

Ta dung 3 S-group i song hanh


We need a leader who has intelligence enough to
make correct decisions,
who has courage enough to
make bold decisions, and
who has wit enough to make
rapid decisions.

Le Th Thuy Loan Trang 120


Phng Phap Dch VIET ANH - NG PHAP

5. Moi ngi eu ngh cau ta co bo oc tuyet


vi, cau ta se hoc ai hoc va se tr thanh mot bac
s gioi.
Everyone thinks that he has a fine mind.
that he will go to university, and
that he will become a good
doctor.

6. Bi le toi khong the i c, khong muon


i va khong co thi gi e i, nen toi a khong i.
Cho 4 v ng i song hanh menh e phu
Since I was unable to go
did not wish to go, and
did not have time to go.
I did not go.

7. Nhieu kim loai khac nhau c them vao hp


kim nay e lam no ran chac hn, ben hn, chu nhiet
hn hoac chong g set nhieu hn.

Many different metals are added to this alloy.


to make it stronger,
more durable,
more heat-resistant, or
more rust-resistant.

8. a be can ngu va an ung gi


The child needs sleep and
Food at regular hours.

13.2 Qui tac song hanh va cac lien t


Cap oi (Correclative conjunctions)
Ta a biet qua ve cac lien t cap oi (correclative
conjunctions) bai hoc so 10. Chung gom co:
Either or Hoac hoac
Neither nor Khong cung khong
Both and Ca lan
Not only but (also) Chang nhng ma con
Wherther or Hoac hoac

Le Th Thuy Loan Trang 121


Phng Phap Dch VIET ANH - NG PHAP

Khi ta s dung cac lien t cap oi. Luat song hanh


phai c ap dung chat che. Moi lien t cap oi
gom co hai thanh phan, au va cuoi. Cac yeu to theo
sau moi thanh phan cua lien t cap oi phai giong
nhau ve t loai hoac ve hnh thc cau tao. Noi khac i,
chung phai i song hanh. Nguyen tac nay c ap
dung chung cho tat ca cac lien t thuoc nhom nay.
Ta lay cap lien t Either... or lam th du. Neu thanh
phan au la Either dan theo sau co mot danh t, th
thanh phan sau la Or cung dan theo sau no mot danh
t. Neu sau Either la mot P-group hoac S-group, th sau Or
phai la mot P-group hoac S-group.
Bang di ay minh hoa Luat song hanh c ap
dung vi cac lien t cap oi.
Either Or
Either Or
Both and
Not only but (also)
Whether or
Neither nor

1. BOTH AND: Ca lan


Both the moon and the planet mars dont have an
atmosphere.
Ca mat trang lan sao Hoa khong co bau kh
quyen.
He came both in the morning and in the afternoon.
Anh ta en ca buoi sang lan tra.
The workers demanded both an increase in wages and
better working conditions.
Cac cong nhan yeu sach ca viec tang lng lan
ieu kien lam viec tot hn.
The regugee children were both cold and hungry.
Cac trai t nan va lanh va oi.

2. NOT ONLY BUT (ALSO): Chang nhng ma con


Her temperature was not only high but also alarming.

Le Th Thuy Loan Trang 122


Phng Phap Dch VIET ANH - NG PHAP

Nhiet o cua co ta chang nhng cao ma con


ang ngai.
Hai phong is not only a port city but an important
industrial center.
Hai phong chang nhng la thanh pho cang ma
con la trung tam cong nghiep quan trong.
He not only sings songs but (also) composes music.
Chang nhng ong ta hat ma con soan nhac.

Chu thch:
Khi at not only au cau, ta phai ao ng
Not only does he sings songs but he also composes music.
Cau sau ay sai v cac yeu to co cung chc nang
ng phap nhng khong c trnh bay bang hai cau truc
giong nhau.
Sai: The new book is not only longer, but also it is more
difficult.
ung:The new book is not only longer (adj)
but also more difficut.
(adj)
Hoac: Not only is the new book longer (adj)
but it is also more difficult.
(clause)

3. EITHER... OR: hoac hoac


Either chemistry or physics is required.
Ngi ta oi hoi hoac hoa hoc hoac vat ly
You can return the books to the library either on Friday or on
Monday.
You can come either before eight oclock or after ten
oclock.
You must decide now either to keep the car or to sell it.
Ban phai quyet nh hoac gi lai chiec xe hoac
ban no.
You may either wash or dry-clean this coat.
Ban co the hoac giac hoac hap chiec ao nay.

Cau sau ay sai v khong ap dung Qui tac song


hanh.

Le Th Thuy Loan Trang 123


Phng Phap Dch VIET ANH - NG PHAP

He is either selfish or he is stupid


(hoac anh ta ch ky hoac anh ta ngu an)

ung: He is either selfish


or stupid.
Hoac: Either he is selfish
or he is stupid.

4. NEITHER... NOR: Khong... cung khong


The trip to Ha Long Bay is neither long nor short.
Chuyen tham quan Vnh Ha Long khg dai cung khg
ngan.
Neither time nor tide waits for any man.
Thi gian va thuy trieu cha ch ai ca.
She neither did her homework nor helped at home.
Co ta chang lam bai tap cung chang giup viec
nha.

Chu thch:
Khi neither at au cau ta phai ao ng neu
no dan au mot menh e va bo ngha cho ong t.
Neither did she do her homework not did she help at home.
They are sending neither money nor clothing.
Ho chang gi en tien hay quan ao.

5. WHETHER... OR: hoac... hoac


Edward did not know whether he should write the man a
letter or he should see him in person.
Edward khong biet hoac cau ta nen viet th cho
ngi an ong o hoac cau ta nen trc tiep gap ong
ta.
We are deliberating whether to say in Hue or to go to
Hanoi.
Chung toi ang an o hoac lai Hue hoac i
Ha Noi.

Luyen dch: Using parallelism.

Le Th Thuy Loan Trang 124


Phng Phap Dch VIET ANH - NG PHAP

Khong kh chang nhng cung cap cho ta oxy ma con


giup truyen am thanh.
The air not only provides us with oxygen
but also helps to transmit sound.
Cac cong nhan hoac phai lam viec sieng nang
ho se b sa thai.
a. Ta cho hai predicates i song hanh:
The workers either must work hard
or will be sacked
b. Ta cho hai clause i song hanh:
Either the workers must work hard (clause)
or they will be sacked (clause)
Gii chu, gii cong nhan chang ai en d buoi
hop ca.
Neither the bosses nor the workers came for the meeting
Khong phai cai nhan cach tao nha cua anh ay,
cung khong phai tien bac, cung khong phai a v xa
hoi cua anh ay lam cho moi ngi ay biet en anh
ta.
Neither his elegant personality,
nor his money,
nor his social position made him popular
here.
Ban phai quyet nh hoac tiep tuc hoc hoac tm
viec lam.
You must decide either to continue to study
or to find a job.
Dau mo c kham pha hoac bang cach truyen
am thanh, hoac th nghiem t trng, hoac khao sat
a chat.
Oil is discoved either by sounding,
or by magnetic testing
or by study of the rocks,
Moi tre em eu can chang nhng thc an bo
dng, ngu nhieu ma con can cam giac c thng
yeu na.
Every child needs not only good food,
plenty of sleep,
but also the feeling of being loved.

Le Th Thuy Loan Trang 125


Phng Phap Dch VIET ANH - NG PHAP

Cau ta rut ngan chuyen ngh he mot phan v


met, mot phan v het tien.
He cut short his summer vacation party because he was
tierd and partly because he had no money.
Cau ay, v cau ay, bo me cau ay chang ai co
mat san bay e on toi ca.
Neither he, nor his wife, nor his parents were at the airport
to meet me.
Ong huan luyen vien trng day cho moi cau thu
ca cach lam viec cung ong oi lan cach bieu lo
tinh than the thao moi luc
The coach taught every player both how to work with the
team and how to show good sportsmanship at all time.

13.3 AND WHO; AND WHICH


Moi lan ta thng pham phai khi ap dung Qui tac
song hanh la vi cac lien quan ai t Who, Which va
That. Nen nh la lien t And khong bao gi c s
dung trc nhng t nay tr khi Who, Which hoac That
a xuat hien trc o trong cau.
Sai: John is an engineer and who works for a big
company.
ung: John is an engineer who works for a big company.

Sai: There is a sign at the crossroads and which will


direct you to our farm.
ung: There is a sign at the crossroads which will direct
you to our farm.
Sai: We went to interview the actor and who answered
our questions willingly.
ung: We went to interview the actor who answered our
questions willingly.

Luyen dch:
1. Bac Ho la ngi yeu thng va a hy sinh
ca cuoc i cho s nghiep giai phong dan toc.

Le Th Thuy Loan Trang 126


Phng Phap Dch VIET ANH - NG PHAP

Uncle Ho was a man who loved his countrymen


and
who devoted his life to the liberation of the nation.
2. Quang Trung la mot v anh hung co thien tai
quan s va c quan chung knh yeu.
Quang Trung was a hero who had a genius for
military art and
who was respected by
people.

Exercise 13A: Correct the faulty parallelism in these


stencences
Sa loi sai ve qui tac song hanh cac cau sau
ay:

Betty prefers parties to see plays and playing the piano.


He plays foodball and skillfully.
The outside of the school was wooden, brick, and stone.
John is an engineer and who works for a car company.
He told us to go by train and coming back soon.
The paiting was neither attractive nor was it enjoyable.
Toms job was driving trucks anf to repair engines for a
company.
Not only is the book expensive but also uninteresting.
We like leaders who are honest and who are fearless.
Most successful pictures are well acted, tell an exciting
story and having a strong central theme.
He did the work and without mistakes.
This applies to our living conditions, what we eat, and our
clothing.
Her duties were greeting guests and sometimes to write
letters.
The policeman told the driver to park his truck and that he
must go to the police station.
The class in reading learned to read faster and also reading
with greater comprehension.
We are losing money at the factory, we must either raise
our prices or we must close down.

Le Th Thuy Loan Trang 127


Phng Phap Dch VIET ANH - NG PHAP

The teacher told the students to write the answers to the


questions and that they must finish within an hour.
He is a pianist and plays trumpet, too.
I like beer, to play cards, and singing.
The guest was told to pack his bags and that he should
leave immediately.
Gloria has nice hair, lovely eyes, and wears beautiful
clothes.
Jerry is ambitious, and has seemingly unlimited energy.
We like the contents of the book and how it was written.
He was neither ready for a job nor did he want a further
education.

Exercise 13B: Applying the rule of parallelism.


Dch cac cau sau ay sang tieng Anh

Ca mat trang va sao Hoa eu khong co bau kh


quyen
(dch hai cach, dung both... and/ neither... nor)
Anh ta chang nhng t choi gap chung toi ma
con t choi viet th cho chung toi na.
Toi cha tin li giai thch ma cung cha tin li ha
cua han.
V kch khong hay, khong vui, cung khong co tnh
xay dng.
David vt Tommy ca toan lan vat ly.
(vt: to excel somebody in...)
Ga tai xe khong nhan loi, cung khong nhan sa
cha thiet hai.
Ca tre con lan ngi ln eu hai long vi buoi
trnh dien.
Hoac ban phai hoc hanh cham ch hoac se hong
ky thi ti.
Anh ta than phien la cha ai giup , cong viec qua
nhieu oi vi anh, va anh ta cau c giup hn e
hoan tat cong viec
Han can chang nhng chien thang ma con thang
ve vang.

Le Th Thuy Loan Trang 128


Phng Phap Dch VIET ANH - NG PHAP

Nhng li phat bieu cua ong ay chang nhng


khong chnh xac ma con khong can thiet na.
Ni cong vien toi thay cac cu gia anh c, cac
ba me ay xe ch cac tre th va cac sinh vien oc
sach.
(cho 3 cau truc Noun x V-ing group i song hanh. ay
xe tre th: to wheel baby carriages)
buoi tiec co ay chang ca hat, chang ke
chuyen vui, cung chang tro chuyen vi ai.
San bay Changi Singapore chang nhng rong hn,
hien ai hn ma con ep hn san bay Thng Hai.
Toi ngh anh ta chap nhan viec lam phan v anh
thch cong viec phan v ong lng cao.
Nhng ngi nhap c a o xo en vung hoang
vang o v ho mong tm c vang, lam giau oi i.
Neu chung ta muon au ky thi ti, chung ta phai
hoc hanh cham ch, va hoc quyet liet (quyet liet:
intensively)
Anh ta a i ti ket luan nh the bi nhng g anh
ta a nghi ng, nhng g anh a nghe, va nhng g anh
a that s thay.
(nghi ng: to suspect). Dung gii t because of va 3
cau truc i song hanh: What x clause).
Anh ta thch song thanh th hn la chan nuoi,
hoac trong cay an qua hoac song gan thien nhien.
(song gan thien nhien: to live close to nature)
Cong viec cua nha s la tram, nho rang va lap
rang gia.
Ngoi sao cua buoi trnh dien cao, manh khanh va
co mai toc bach kim.
Khoa hoc se day ban cach s dung ong c diesel,
cach sa cha va bao tr chung. (ong c diesel: a diesel
engine)
Toi se hoc ai hoc e co kien thc, ket ban mi
va trang b cho mnh mot nghe.
14. PHRASE, CLAUSE AND SENTENCE

bai hoc nay ta se on lai ve hnh thai va chc


nang cua cac cum t va menh e.

Le Th Thuy Loan Trang 129


Phng Phap Dch VIET ANH - NG PHAP

14.1 The Phrase:


Phrase la mot cum t c dung trong cau noi nh
mot t loai (a part of speech). Ngha la co the dung trong
cau noi nh mot danh t, tnh t hoac trang t.

Phrase khong cha ng mot chu ng va ong t.


Neu cum t co chc nang cua danh t, ta goi no la
noun phrase. Neu cum t co chc nang cua tnh t, ta
goi no la adjective phrase. Neu no co cha nang cua
trang t, ta goi no la adverb phrase. Ta cung goi toan bo
cum t gom tr ong t va ong t chnh la verb
phrase. Ta a noi ve cau tao va chc nang cua phrase
cac bai hoc P-group, Ving-group va to-group.

Mot lan na ta nhac lai cac danh xng va nhiem


vu ng phap cua cac cum t.

14.1.1 The Prepositional phrase


Cum t nay c dung nh tnh t, trang t va
danh t.
The news of the strike spread quicly
Cum nay co chc nang cua tnh t.

He ordered a hamburger with onions. (adj)


The couple danced across the floor. (adv)
He has sure of himself. (adverb)
After dinner is the best time for a long walk.
Cum t dung nh danh t, chu ng cua is.

14.1.2 The Participial Phrase


Cum t nay luon luon c dung nh tnh t
The boy wearing the red jacket is Larry.
Knowing the exact route, Joe didnt get lost.
Stopped at the border, the travelers had to show their
visas.
This scholarship, awardws annually, is worth $ 2,000.
Hoc bong nay c cap hang nam, tr gia 2.000
o la.

Le Th Thuy Loan Trang 130


Phng Phap Dch VIET ANH - NG PHAP

14.1.3 The gerund phrase


Cum t nay luon luon c dung nh danh t.
Taking in the audience disturbs the speaker. (subject)
Owning a car gives a person many responsibilities.
S hu mot chiec xe khien chu nhan co nhieu
trach nhiem.
I enjoyed watching the sunset on the beach.
He found excitement in using a powerful telescope.
Lindas mistake was reading the directions carelessly.

14.1.4 The Infinitive Phrase


Cum t nay c dung nh tnh t, trang t hoac
danh t.
The man wanted to cash a check. (noun)
To write good poetry is his ambition. (noun)
I am happy to accept your invitation. (adverb)
To work hard is to be happy. (noun)
Thomas Jefferson had to sell his library to pay his debts.

14.1.5 The Appositive Phrase


Appositive la mot cum t c at sau t khac e
giai thch hoac nhan dien no.
Nguyen Du, the Vietnamese poet, wrote Kim Van
Kieu
Dr. Thompson, the mayor, spoke first.
Appositive phrase gom co appositive va cac yeu -
to - mo - ta cua no. Yeu - to - mo - ta t no co the
la cum t hoac menh e.
Mr. Greens car, an old Toyota that had been newly
painted, was stolen. (Ca cum t la appositive phrase)
The fair, an annual event that attracts thousands,
opens next week.
The chairman presented the trophy, a tall siver cup with
handles.
Ong chu tch trnh bay chien tch, mot chiec cup
bac ln co quai.

14.2 The Clause (Menh e)

Le Th Thuy Loan Trang 131


Phng Phap Dch VIET ANH - NG PHAP

Clause la mot top t ng co cha mot chu ng


hoac ong t.
V cac top t ng sau ay thieu mot chu ng
hoac ong t hoac thieu ca hai nen khong phai la
menh e.
Was running a race (thieu chu ng)
The man to see (thieu mot ong t)
In the middle of the town (thieu chu ng va ong
t)

Theo nh nh ngha luc ban au, mot cau n la


mot menh e. That vay, mot cau n oi khi c
nh ngha la cau gom co mot menh e chnh (hoac
menh e oc lap). Tuy nhien se n gian hn khi hieu
clause c dung trong cau noi tng ng nh mot t
loai; ngha la no co the c dung nh tieng noun,
adjective hoac adverb nh ta a biet bai hoc ve S-
group.
Moi phan cua cau kep eu co ong t va chu
ng. Nhng thanh phan cua cau kep v the la nhng
menh e.
Moi menh e trong mot cau kep co the c
tach ra va viet nh mot cau n.
Mot menh e t no co the ng mot mnh nh
cau noi la mot menh e chnh (a main clause).
Chung ta a nh ngha mot cau kep la cau co
cha hai hay nhieu cau n ket hp lai vi nhau. Bay
gi chung ta cung co the nh ngha cau kep la cau
co cha hai menh e chnh.
Mot menh e t no co the ng mot mnh nh
cau noi la mot menh e phu (a subordinate clause).
Dau hieu e nhan ra menh e phu la mot
subordinator (When, if, until, because, who, which, that, v.v.)
ng trc no.

Cac th dung sau ay la menh e phu:


When he asked me (what happened?)
If you have the strength (Then what?)
While you were away (What?)

Le Th Thuy Loan Trang 132


Phng Phap Dch VIET ANH - NG PHAP

Unless it rains a lot

Nh vay mot menh e phu khong the ien at


mot y tng ay u c. No ch lam nhiem vu
tng ng vi mot t loai trong toan bo y tng c
dien at. Ta so sanh tng cap sau ay:
He is an hornest man (adjective)
He is a man who is honest (adjective clause)
He came early (adverb)
He came before the class started (adverb clause)
I know the formula (noun)
I know that water is made up of hydrogen and oxygen.
(noun clause).
Trong tieng Anh, menh e phu ng n oc mot
mnh la mot loi trong ng phap goi la Fragment, tc la
mot mang, mot manh vun cua cau noi hoan chnh.
Fragment thng xay ra khi ta dung dau cham cau
khong ung hoac khi ta e cac cum t nh prepositional
phrase, participial phrase, infinitive phrase hoac appositive
phrase ng n oc mot mnh.
Fragment: At the start of the second game. If began to rain.
Sentence: At the start of the second game, it began to rain.
Fragment: The girls worked hard. At the start of the year.
Sentence: The girls worked hard at the start of the
year.
Fragment: Linda has a plan. To study medicine in Paris.
Sentence: Linda has a plan to study medicine in Paris.
Fragment: Hemingway, a famous American writer.
Sentence: Hemingway was a famous American writer.
Fragment: Jacqueline was thriied. To be chosen as a
class president.
Sentence: Jacqueline was thrilled to be chsen as a class
president.
Fragment: Althought Jane had her licence. She was
afraid to drive.
Sentence: Although Jane had her licence, she was afraid
to drive.
Fragment: The rocket, the heaviest ever launched.

Le Th Thuy Loan Trang 133


Phng Phap Dch VIET ANH - NG PHAP

Sentence: The rocket, the heaviest ever launched,


roared toward the moon.

Tom tat:
1. Phrase la mot cum t c dung trong cau noi
nh mot t loai. Mot phrase co the c dung nh danh
t, ong t, tnh t hoac trang t. Phrase khong cha
mot chu ng va ong t.

2. Clause co the c dung trong cau nh mot


danh t, tnh t hoac mot trang t.
Phrase: Walking to the station
Clause: While we were walking to the station.

3. Mot menh e chnh co the ng mot mnh


nh cau noi. Mot menh e phu khong the t ng
mot mnh nh cau noi c.

(Main clause) (Main clause)


The sky was cloudy all day, but no rain fell.
The sky was cloudy all day. (complete)
No rain fell. (compltete)

(Subordinate clause) (Main clause)


Although the sky was cloudy all day, no rain fell.
No rain fell. (complete)
Although the sky was cloudy all day (incomplete)

Phrase va Clause la mot phan cua cau noi

14.3. The Sentence


Cau noi (the sentence) c nh ngha nh sau:
Cau la mot cum t.
Dien at mot y tng hoan chnh,
Co cha mot chu ng va ong t,
Khong phai la thanh phan cua mot cau truc ng
phap khac.

Exercise 14A: Identifying Phrase and Clause

Le Th Thuy Loan Trang 134


Phng Phap Dch VIET ANH - NG PHAP

In each of the following sentences, decide whether each


underline groupof words is a phrase or a clause.
moi cau sau ay, quyet nh xem moi cum t
c gach di la phrase hay clause.
Ex: The short man with the sunglasses is an F.B.I agent
(phrase)
Most of the people who frequent this restaurant are actors.
The list of things to be packed are on the table.
What Mr. Green needs is a doctor and a rest cure
To photograph the ocean bottom, two men descended in a
metal sphere.
We could not see who was at the door.
After leaving high school, Hoa will go to a business school.
Scientists are now hopeful for finding a cure for AIDS.
He is a man of considerable business ability.
When Mozart was eight, he composed his first symphony.
He was determimed to be a great success in his new job.
An attempt to launch the space vehicle will be made Friday.
Watching television can be both entertaining and
educational.
I laughed because the situation was ridiculous,
Since the draw of time, man has been seeking freedom.
Bacteria are usually so small that they can be seen only
under a microscope.
Planning for a career requires foresight.
To see the earth properly, it is necessary to leave its
surface.
John came home late, carrying a mysterions sack on his
shoulder.
Not until morning did we learn what was in the sack.
We prefer flying to going by boat.
Because he worked nights, he was sleepy in class.
Bats find their way by means of the highly efficient radar
system.
Present Harrison died one month after his inauguration.
John Hancock was the first to sign the Declaration of
Independence,
As soon as I am oil enough, my father will byu me a
motorcycle.

Le Th Thuy Loan Trang 135


Phng Phap Dch VIET ANH - NG PHAP

Exercise 114B: Find the subordinate clause in each sentence


below. / Tm menh e phu moi cau di ay.
Ex: Here is the list of people who have bough tickets.

You dont have to take the job if you dont like this kind of
work.
I left because I had a headache.
I could see that the boys had made great progress.
The project looked as if it would fail.
Aladdin could remember what secret word opned gate.
I cannot trust a man who soes not acknowledge mistake.
When food is oxydize in muscle cells, physical ener is
released.
It was now clear that the bus could not be repaired.
How migrating animals find their way on their long journey
is a great mystery.
That electrocity is a source of heat may be seen in the
electric iron and the electric stove.
When Benjamin in Europe.
A planet whose atmosphere containers water vapor may
support some form of life.
What he had done saddened his parents.
That they were in need of food was easy to see.
It rained heavily when we left the house.
Wait until you receive the signal.
Check the telephone number before you direct dial to
Spore.
My guess is that he will refuse.
What really made the party a success was the outlandish
customes.

Exercise 14C: Find fragments. Correct them by changing the


punctuation or by adding the words needed to make a sentence.
Ex: Heart disease, one of the chief causes of death.
Heart is one of the chief causes of death.

Harry likes to dance. In spite of the fact that he is lame.

Le Th Thuy Loan Trang 136


Phng Phap Dch VIET ANH - NG PHAP

Sixty million Americans are overnight. According to a


survey.
Address the envelopes. As you habe been instructed.
Id like try to remain neutral. If I were you, Jim.
Ships, large and small came into the harbor. From many
contries.
I must redign my position. Since I have no other choice.
Although the pianist played with fervor. He made many
mistake.
I will mow the lawn this afternoon. If I werent so lazy.
Starting in the garage. Flames spread quickly through the
house.
After the dentist began drilling. I became more nervous.
Exercise 14D: Rewrite the word groups below to eliminate
fragments.
Ex: He heard the new. When he reached his office.
He heard the new when he reached his office.

We never could find out. Who ran over our dog.


The defeat was hard for Paul to accept. Since he had worked
so hard for so long.
The final victory came as a surprise. Although it had never
been in doubt.
Dr. Ban decided to return to Hawaii. Where he had spent so
many years.
Come again. Whenever you can.
If the men are still sleeping. Do not disturb them.
Do not remove the cap. Until you are ready to use the ink.
Litle is known about Shakespeares lift. Except to few facts.
I figured out thr trouble. Dirt in the carburetor.
We decidednot to get an American car. The difficulty getting
spare parts.
The wives of the mainers anxiously awaiting news at the
mouth of the mine.
The announcement, a warning to all who lived in the village.
Watching the movies, a story of early days in Montana.
No one liked Toms suggestion. To raise money for the class.
Bob was delighted. To have a chance to meet and talk with
the great scientist.

Le Th Thuy Loan Trang 137


Phng Phap Dch VIET ANH - NG PHAP

15. KINDS OF SENTENCES

Chung ta a biet rang cau co the c xep loai


tuy theo muc ch cua ngi noi: cau tran thuat
(declaretive), cau menh lenh (imperative), cau nghi van
(interrogative) va cau cam than (exclamatory). Cac xep
loai nay rat hu ch trong van e cham cau.
Nhng co mot cach xep loai khac tien dung hn.
o la cach xep loai theo hnh ve. Co bon loai cau trong
tieng Anh:
Cau n (the simple sentence)
Cau kep (the compound sentence)
Cau phc (the complex sentence)
Cau kep phc (the compound complex sentence)

15.1 The simple sentence (Cau n)


Ta lay hai nh ngha tieu bieu nhat ve cau n:
Cau n la cau ch co mot menh e oc lap.
Cau n la cau co cha mot chu ng va v ng.

Nhng ta biet rang ca hai yeu to chu ng (Subject)


va v ng (Predicate) co the la yeu to kep (the
compound element). Neu Subject la yeu to kep, ta goi o
la compound subject. Neu Predicate la yeu to kep. Ta goi
no la Compound predicate. Ngoai ra, cac yeu to khac
trong cau nh: verb, object, object of preposition, v.v... eu
co the la yeu to kep.

Ta quan sat cac yeu to kep trong cau n cac th


du sau ay:
1. Compound Subject:

Le Th Thuy Loan Trang 138


Phng Phap Dch VIET ANH - NG PHAP

The telegraph the electric doorbell works on similar


principles.
ien bao va chuong ien van hanh theo cung
nguyen tac.
Mercury and alohol is used in a thermometer.
Thuy ngan va con c dung trong han th bieu.
The sun, the moon and the stars gave ancient man a lot of
things to think.
Mat tri, mat trang va cac v sao cho ngi xa
nhieu th e suy ngh.

2. Compound verb:
They came and stayed
The boys ran, jumped, and shouted.
The audience cheered and applauded the play.

3. Compound Direct Object:


We bought newspapers and magazines.
She needs flour, sugar and eggs.

4. Compound Indirect Object:


She told the boys and the girls funny stories.
I gave Steve and Dick tickets.
He gaves tennis, golf, and dancing all his time anf
thought.

5. Compound object of Preposition:


We talked about our careers and our hopes.
They drove through the heavy rain and fog.

6. Compound Subject Complement:


She was tired and hungry
That car looks old but dependable.

7. Compound Predicate:
Mary turned off the radio and began her homework.
Heavy rains flooded the streets and damaged many
homes.
The boys and the girls washed the dishes and dried them.

Le Th Thuy Loan Trang 139


Phng Phap Dch VIET ANH - NG PHAP

15.2 The Compound Sentence (Cau kep)


Cau kep gom hai hoac nhieu cau n ket hp lai.
S ket hp co the nh sau:
Hai cau n noi vi nhau (1) bang mot dau phay
va mot trong cac lien t nh: And, But, Or, For, Nor, So
hoac (2) bang mot dau cham phay (a semicolon).
The flowers were beautifull, but she did not buy them.
Tom has a pleasing personality, and he knows when to use
it.
Mr. Hung knew Dalat well, for he had taught here.
You can take five hours by car, or you can get there in an
hour by plane.
Jeeps are light, so they run very fast.
Jeff could not do the tango, nor could he do the cha-cha-
cha.
Jeff khong the nhay ieu tango, anh ay cung khong
the nhay ien cha-cha-cha.
Mr. Hung knew Dalat well; he had taught there.
Mary washed the dished; Betty dried them.
I cam; I saw; I conquered. (Caesar)
The bank failed; Mr. Green disappeared.

Mot lien trang t (conjunctive adverb) nh then,


however, moreover, hence, comsequently, therefore, ets. Cung
c dung e noi hai cau n e tao thanh cau kep.
Dau cham phay c at trc conjunctive adverb.
I want to go shopping; however, I have no money.
His advice was not taken; consequently, he was angry.
The wood was wet; therefore, it was useless for building
Go b t; do o khong dung xay dng c.
Harry didnt meet us at the appointed time; nevertheless,
we waited.
Read the directions carefully; then begin the examination.
oc ky cac li hng dan roi bat au lam bai.

15.3 The Complex Sentence (Cau phc)

Le Th Thuy Loan Trang 140


Phng Phap Dch VIET ANH - NG PHAP

Cau phc gom co mot menh e chnh va mot


hoac nhieu menh e phu. Trong mot cau phc, menh
e phu luon luon c dung nh mot t loai hoac nh
mot yeu - to - mo - ta. Neu menh e la yeu - to -
mo - ta th no bo ngha cho mot t menh e chnh.
Luon nh rang menh e phu luon luon c gii
thieu bi mot Subordinator nam trc menh e.
Although he knew the answer, he did not raise his hand.
Read the paper while you are waiting.
(menh e phu bo ngha cho read)
This is the music that I like.
(menh e phu bo ngha cho music)
If the emergency continues, water will be rationed.
(menh e phu bo ngha cho will be rationed)
Neu trnh trang khan trng tiep tuc, nc se c
cap theo nh mc.

Cac cau phc co cha mot menh e danh t th


co hi khac. Menh e danh t co the c dung nh
mot danh t trong menh e chnh. Noi khac i, menh
e danh t la mot phan cua menh e chnh; ngha la
no ng lam chu ng, tan ng hoac bo ng trong
menh e chnh.
What you read influences your thinking.
(menh e danh t la chu ng cua influences).
No one understood what she was saying.
(menh e danh t la tan ng cua understood)
The teachers felt responsible for what the students did.
(menh e danh t la tan ng cua gii t for)
Where he is living is not know.

Trong cac cau noi tren, ca menh e chnh lan


menh e danh t eu khong the t ng mot mnh.
Tuy nhien, mot cau co cha menh e chnh va mot
mot menh e danh t c coi nh cau phc.
15.4 The Compound-Complex Sentence (Cau kep
phc)
Cau kep phc gom co hai hoac nhieu menh e
chnh va mot hoac nhieu menh e phu.

Le Th Thuy Loan Trang 141


Phng Phap Dch VIET ANH - NG PHAP

Loai cau nay eu co tnh chat cua cau kep lan


cau phc. Cac cau menh e chnh c noi vi nhau
bi mot co-ordicating conjunction, mot conjunctive adverb
(lien trang t) hoac ch bi mot dau cham phay.
Menh e phu bo ngha mot t nam trong mot menh
e chnh.

I will I will bring a If he has a


come and friend day off
(Main (Main (Subordinate
clause) Clause) clause)
We
however, we before the
missed the
arrived home others did
bus
(Main (Subordinate
(Main
Clause) clause)
Clause)
I ran
and there I that I had
back to
found the key lost
school
You can but dont be if you hear
stay here surprised strange noises

Exercise 15A: Identify the compound parts in the following


sentences. Look for compound subjects, compound verbs and
compound predicates.
Nhan dien cac yeu to kep nhng cau sau ay.
Tm chu ng kep,ong t kep va v ng kep.
Ex: Mr. Berstein conducted, played, and lectured
(compound verb)

Botany and zoology are branches of biology.


Colombus took with him a compass, an astrolable, and his
own courage.
A huge truck had overturned and blocked the road.
My dog barks but never bites.
In the East great civilizations flourished and died
thoudsands of year ago.
For five hours I read and wote.

Le Th Thuy Loan Trang 142


Phng Phap Dch VIET ANH - NG PHAP

The spaceman will need courage, patience and stamina to


survive.
Valleys, ridges, and mountains have been charted on the
ocean floor.
The moon turns on is axis every 28 days and therefore
always presents the same surface to the earth.
Experts examined the old paintings and declared them
priceless
Both the manager and his assistant were injurted in the
crash.
Halleys comet last appeared in 1986 and is expected again
in the year 2062.
The committee will hire an orchestra and arrange for
decorations.
VanGogh died impoverished but painted pictures worth
millions
The advertising department apologized and refunded the
money
Decaying leaves and animal matter make the richest soil.
The turtle lays eggs and never gives the next generation a
backward glance.
The earthquake destroyed the city and left thousands
homeless.
Both of his sons studied hard and won scholarships.
The orchestra and chorus rose and took a bow.
Neither the man nor the womwn gave information.
The police arrived and roped off the entrance.
The instructor handed out the tests and left the room.
Grasshoppers hatch out in the spring & die before cold
weather.
Kathy will stay with us and will cook our meals.

Exercise 15B: Combine each pair of sentences into one


simple sentence, using a compound predicate.
Ket hp moi cap sau ay thanh mot cau n vi
v ng phoi hp.
Ex: Paul worked as a drugstore delivery boy after
school, and he was a lifeguard at the city pool last summer.

Le Th Thuy Loan Trang 143


Phng Phap Dch VIET ANH - NG PHAP

The book is long, and it requires careful reading.


The band uniforms have been delivered. Everyone likes
them.
The telegram was delivered this morning, but someone put
it in the wrong basket.
Politeness costs nothing. It often wins everything.
We were delayed by a flat tire, and we missed the show.
Heavy rain flooded the streets. They damaged many homes.
The elevator operator stated the elevator. He called the
police.
The astronaut waved to the reporters. He entered the
spaceship.
The word democracy comes from the Greek. It means
government by people.
Suddenly Jerry heard a noise. He hid behind a tree.

Exercise 15C: Identifying kinds of sentences


Indicate wherther each sentence below is simple,
compound, complex, or conpound complex sentences.
Ex: She dances expertly and sings well, too (Simple)

The sun gives us light, and keeps us warm.


The sun gives us light, and it keeps us warm.
Scientists searching for new medicines have found valuable
drugs used by ancient people.
During the long winter at the South Pole, the sun does not
shine for six months.
Every body stopped talking when the docter entered the
room.
Today radar has eliminated the danger of icebergs and fog.
Ill go to England; however, dont be surprised if I get to
Danmark, too.
Two small moons circling the planet Mars are said to be
hollow.
The light that reaches our eyes from some stars started on
its way before Columbus discovered America.
The man to whom you spoke is the director of the museum.
We rounded the bend, and we saw a truck that had roiled
into the ditch.

Le Th Thuy Loan Trang 144


Phng Phap Dch VIET ANH - NG PHAP

We finally found the gate; however, it had been locked.


Either Tom must enter the contest, or we shall lose the prize
The planet Earth is not quire a sphere but is slightly
flattened.
An atom of oxygen weighs sixteen times as much as an
atom of hydrogen.
The book says that there is no water at all on the moon.
I ran back to the camping site, and there I found the shoes I
had lost.
The newest discoveries inscience are affecting our way of
life.
All high school students should decide early upon the
college they wish to attend.
Because the weather looked threatening, we postponed our
picnic.
The book, anovel about a small town, has caused a nation-
wide sensation.
The driver, who had fallen asleep at the wheel, was killed
when his car crashed into a tree.
A distanve of 250,000 miles separates the earth fron the
moon.
We walked through the cemetery, trying to be as brace as
possible.
Jerry will go by train; Harry will take the bus.
Envy and Anger destroy friendships and shorten life.
A number that is supposed to bring bad luck is thirteen.
To the people of Japan, Fujiyama is a sacred mountain and
many people ascend to its crater.
Tell Louis to write his term paper and to submit it to
Professor Lee.
The fight against tuberculosis is growing harder because the
germs have become resistant to the new drugs.

16. SOME SPECIAL EXPRESSIONS


(VAI HNH THAI DIEN AT AC BIET)

Le Th Thuy Loan Trang 145


Phng Phap Dch VIET ANH - NG PHAP

1. ai ma, he ai, he who, anyone


who,
bat c ai ma, one who, whoever,
nhng ai ma, those who,

He ai khong hieu xin gi tay?


Would he who doesnt understand raise his hand?
Ai nha kieng khong nen nem a.
One who lives in a glass house shouldnt throw stones.
Ai cha tng phuc vu khong the ch huy.
He who hasnt served cannot command.
Bat c ai cung c.
Anyone would do.
Ai noi the la ke noi lao.
Whoever says that is a liar.
Nhng ai suy ngh sau xa thng suy ngh rat
cham.
Those who think depply think slowly.
Nhng ai d nh lam cong tac khoa hoc phai hoc
toan.
All those who intend to work in science must study
mathematics.

2. bang khong, neu otherwise


khong, ngc lai

Tien thue th cao, ngc lai can nha th ep.


The rent is high, otherwise, the house is fine.
Chung ta phai ri ngay lap tc, bang khong se
b tre.
We must leave at once; otherwise, we should be late.
Hay lam nhng g giao vien a bao, bang khong
ban se b phat.
Do what your teacher has told you; otherwise, you will be
finished.
Hay nha neu ban benh; con neu khong,
chung toi se i ban sang mai.
Stay home if youre sick; but otherwise well expect you
early tomorrow.

Le Th Thuy Loan Trang 146


Phng Phap Dch VIET ANH - NG PHAP

3. bi, do, bi le, v because, because


of, since, for,
now that, on account of, due to,
owing to.

Toi biet chiec xe rat ky, bi toi tng la chu


cua no.
I knew the car well, for I had once owned it.
Bi lai xe au anh ta a gay tai nan.
Owing to his careless driving, he caused an accident.
Anh ta b tre la bi ket xe.
His lateness was due to traffic jam.
V co v cua anh ay ay, toi cha noi g ve
viec o ca.
Because of his wife being there, I said nothing about it.
Lch s la s tiep noi nhng cuoc chien tranh do
con ngi co nhng quyen li khac nhau, co nhng
tham vong oi nghch nhau va do con ngi khong tin cay
lan nhau.
Because men have diffirent interests, because man have
conficting ambitions, because man distrust one another, history
is a succession of wars.

4. Chang qua only

Co ay lam the chang qua e me co ay bang


long.
She did it only to please her mother.

Cai o chang qua la e ong ay khoi mat s


dien o thoi.
That was only to save his face.
oi vi nhng ngi bi quan, lch s chang qua la
chuoi tiep noi nhng cuoc chien tranh.
To pessimistic people, history is only a succession of wars.
5. chang chong th chay, sooner or later,
khong sm th muon. In time, one day

Le Th Thuy Loan Trang 147


Phng Phap Dch VIET ANH - NG PHAP

Chang chong th chay ban cung biet cach van


hanh co may nay
Youll learn how to operate the machine in time.
Ban nen noi that vi co ta v sm muon g co ta
cung se biet.
You should tell her the truth, because shell know it sooner
or later.

6. chnh ma It is/ was to-group


It is/ was who
It is/ was.,. V-ing group
It is/ was P-group

Chnh v an ma ho en ay.
It was to eat that they cam here.
Chnh la chi bong lam cau ta khoe.
It is playing football that makes him strong.
Chnh ong Green a goi ien.
It was Mr. Green who telephoned.
Chnh ngoi lang nho be o ma nha th v ai
a chao i.
It was in that small village that the great poet was born.
Chnh James Watt a phat minh ra ong c hi nc
au tien.
It was James Watt who invented the first steam engine.
Chnh la nh co may ien toan giup ma cac
nha khoa hoc co the tnh quy ao cac phi thuyen va
ve tinh.
It was with the help of aomputers that scientists can
calculate the orbits of spaceships anf satellites.

7. ch khong but not, but no

Viet Nam co voi va cop ch khong co s t


Vietnam has elephants and tigers, but no lions.
Anh ta co tai ch khong co c
He has many talents, but no virtue.
Cau ta thch co ay v s thong minh cua co ay
ch khong phai v sac ep.

Le Th Thuy Loan Trang 148


Phng Phap Dch VIET ANH - NG PHAP

He likes her for her intelligence but not for her beauty.

8. cang... cang the more... the


more
the more... the less
the x comparative adjective.

Mot ngi cang hieu biet ho cang khiem ton.


The more learned a man is, the more modest he usually is.
Cang hoc tieng Anh, toi cang thay thch.
The more I study English, the more I like it.
Cang lai o bao nhieu lau, toi cang thch ni
o bay nhieu.
The longer stay there, the moreI like the place.
Chuyen bay cang dai, phi c cang phai mang nhieu
nhien lieu.
The longer the fligh is, the more fuel a plane must carry.
Cang nghe anh ta noi, toi cang t tin anh ta.
The more I listened to him, the less I belived him.
Cang chn cang ngon.
The riper, the tastier.

9. bao lau, bao lau ma, so long as,


chng nao ma as long as

Chng nao con co ngi nghe co ta cang tiep tuc


noi.
She will go on talking as long as there is someone to listen.
Bao lau tri con sang, ho con chi tennis.
They played tennis as long as there was sunlight.
(They played tennis as long as the light lasted)

10. dan da gradually,


step by step,
little by little.

Le Th Thuy Loan Trang 149


Phng Phap Dch VIET ANH - NG PHAP

Dan da ho tr thanh ban than vi nhau.


Gradually they become good friends.
Dan da roi ho cung se quay ve co hng.
Gradually theyll come back home.
Ong ta dan da roi cung quen luat i ben phai
cua Viet Nam.
He become step by step familiar with the right-hand rule
in VN

11. do o, do vay, consequently, as a


consequence
bi the, bi le o as a result, hence,
therefore

Mat trang khong co bau kh quyen va nc, do


o khong co hnh thai nao cua s song ton tai o.
The moon doesnt have an atmosphere and water, as a
consequence, no form of life exists there.
Con tau vu tru a lac quy ao, bi the moi lien
lac vo tuyen a mat.
The spaceship lost its orbit, hence, all contacts by radio
stopped.
Xe toi hong do vay ma toi b tre.
My motorcycle broke down; consequently, I was late.
Da Lat co nhng rng Thong, do o ngi ta
thng goi no la Thanh Pho Thong.
Da Lat has large pine tree forests; therefore, it is often
called the Pine Tree City.
Cau ta chang hoc hanh g ca, bi the ma thi
rout.
He didnt study; as a result, he failed the test.
Chiec xe cham lai luc len eo, v vay ma toi
phai tra so.
The car lost speed going uphill; consequently, I had to
shift gear.
12. du, cho du, dau ma though, although,
even if,
dau rang no matter what, whatever,
no matter how, however,

Le Th Thuy Loan Trang 150


Phng Phap Dch VIET ANH - NG PHAP

for all x noun,


adjectve (adverb) x as

Cho du ieu g a xay ra, ho van c la ban.


No matter what had happened, they remained friends.
Du anh ta a noi va lam g i na, toi cung
khong tin.
No matter what things he said and did, I didnt believe.
Mat du Tom a song Phap nhieu nam, cau ta
khong noi c tieng Phap troi chay.
Though Tom has lived for many years in France, he doesnt
speak French fluently.
Cho du phai mat sau thang, toi cung cng
quyet hoan thanh cong viec.
Even if it takes me six months, Im determined to finish
the job
Cho du co ay lam cua, co ta van khong hanh
phuc.
For all her wealth, she doesnt feel happy.
Du ban co noi ieu g i na, toi van thch ong
ay.
For all you say, I still like him.
Du han giau bao nhieu chang na, toi cung cha
can.
Rich as he is, I dont care.
Mac du chung ta knh trong ong ay bao nhieu
chang na, chung ta cung khong the ong y rang
ong la nha chnh tr khon ngoan.
Much as we respect him, we cannot agree that he is a
wise politician.
Du co xay ra viec g i na, ban se luon hai
long la mnh a co het sc.
Whatever happens, you will always be glad that you tried
to do your best.

13. he, he ma, mot khi, each time,


whenever
moi lan, he khi once

Le Th Thuy Loan Trang 151


Phng Phap Dch VIET ANH - NG PHAP

He ba ay noi gian la ba ay hay ap bat a.


Each time she gets angry, she usually breaks dishes.
He Pele ghi mot ban thang th ong oi cong
kenh anh.
Whenever Pele scored a goal, his teammates carried
him astride on their shoulders.
Mot khi ban biet c qui luat ban se khong thay
kho.
Once you understand the rule, you will have no further
difficully.
He khi co ta en la co ta dat theo ban.
Whenever she comes, she brings a friend.
Moi khi ban han t choi giup , han thng
noi xau ban.
Once his friend refused to help, he often spoke ill of him
Moi lan ma th mai dot.
The roof leaks whenever it rains.
Ban se xoay s the nao khi ma tien a het?
How would you cope once the money had gone?

14. hn the na whats more,


furthermore,
moreover

Han giau co, hn the na han rat thong minh.


He is rich; furthermore, he is very intelligent.
An qua mc lam ngi ta ph non kho coi.
Overeating makes a person unattractively fat; whats
more it harms ones health.
Chung toi a oc tat ca nhng ban tin, hn the
na chung toi a oc tat ca mau quang cao.
We had read all the news stories; moreover, we had even
read all the advertisements.
15. huong chi, let alone, much less,
huong ho, na la not to mention

Tieng anh, tieng Phap anh ta con khong noi c,


huong ho tieng Tay Ban Nha.
He cant even speak English and French, let alone Spanish.

Le Th Thuy Loan Trang 152


Phng Phap Dch VIET ANH - NG PHAP

Mot ban thang anh ta con ghi cha c huong chi


la 2 hay 3.
He didnt score a goal, much less two or three.
Khg co cho cho chung toi, huong ho cho cho con
cho cua anh.
There isnt enough room for us, let alone your dog.

16. mien la provide (that)


vi ieu kien on condition that

Anh ta se hoc ai hoc mien la anh ta c hoc


bong.
He will go to university provided (that) he wins a
scholarship.
Ban co the lay quyen sach hom nay mien la
ban tra lai chieu mai.
You may take the book today provided that you return it
tomoreow evening.
Cac em co the chi ay vi ieu kien la cac
em khong lam on.
You all can play here on condition that you shouldnt make
a noise.
Toi se giup ban vi ieu kien ban phai co het
sc mnh.
Ill help you on condition that you must try your best.

17. mai cho en... th not until,


it is/ was not until... x that-
clause

Cho en sang th Bay th han gap ong ay.


Dont see him until Saturday morning.
Mai cho en thang Sau va qua th ong ta mi
sang Viet Nam.
He didnt arrive in Vietnam untl last June.
Mai cho en gan ay ngi ta mi chup c hnh
sao hoa.
It was not until recent times that the pictures of Mars were
taken.

Le Th Thuy Loan Trang 153


Phng Phap Dch VIET ANH - NG PHAP

Mai cho en thang Tam nam 1945 chien tranh the


gii th hai mi ket thuc.
It was not until August, 1945 that the Second World War
came to an end.

18. mot phan v... partly because

Anh ta khong tham gia chuyen i mot phan v ban,


mot phan v khong u tien.
He didnt join a trip, party because he was busy, party
because he didnt have enough money.

19. ngo hau, that, so that


e nham muc ch in order that,
in order to.

Anh ta sang Paris ngo hau hoan chnh mon tieng


Phap cua anh.
He went to Paris so that he sould pefect his French.
Nhng ngi chien s vo danh a chet e ong
bao ho c song trong t do.
Those unknown soldiers died that their countrymen might
live in freedom.
Ngan ng Trung Quoc co cau, Ngi ta co hai tai
va mot cai li e nghe nhieu va noi t
A Chinese proverb says, We have two ears and only one
tongue in order that we may hear more and speak less.

20. nh, nh , thanks to


nh vao because of

Cuoc cam trai rat thanh cong nh thi tiet tot.


The picnic was a great success thanks to the fine weather.
Nh cac trang thiet b ien t hien ai, con ngi
a chinh phuc mat trang.

Le Th Thuy Loan Trang 154


Phng Phap Dch VIET ANH - NG PHAP

Thanks to mordern electronic equipment, man has


conquered the moon.
Nh cac dc pham hien ai, benh lao va
benh sot ret khong con nguy hiem nh trc ay na.
Thanks to modern medicines, cubertulosis and malaria are
no longer dangerous as they used to be.

21. phong khi, nh khi in case, for


fear
e rang in the event that, lest

Ban nen mang theo du nh khi tri ma.


Youd better take an umbrella in case it rains.
Ong trong tai eo hai ong ho phong khi mot
chiec b hong
The referee wears two watches in case one of them stops.
Ngi s quan ch huy quyet nh rut lui ngay lap
tc, e rang binh s cua ong b phuc kch.
The commanding officer decided to retreat at once lest his
soldiers should be ambushed.
Toi an mot ba no bung trc khi ra i phong khi
khong kiem c g e an khi en o.
I took a good meal before I went for fera I should not be
able to get any food there.

22. nen, bi... cho so,


nen, v the co nen thats why

Xe zp nhe nen nhanh.


Jeep are light, so they are las.
Bi no chang hoc g ca nen mi thi hong.
He didnt study, so he failed in the exam
Ngoi lang nam doc theo b bien nen hau het
dan c song bang nghe ca.
The village lies along the coast; thats why most of the
villagers make a living by fhishing.

Le Th Thuy Loan Trang 155


Phng Phap Dch VIET ANH - NG PHAP

23. s d... la v the reason


why

S d co chuyen o la v ho hieu nham toi.


The reason why that happened was that they
misunderstood me.
S d co ta bo hoc la v co ta lam benh nang.
The reason why she left school was that her mother got
seriously ill.

24. song; song, tuy nhien however, but yet

T trai at, bau tri trong xanh, song nhn t mot


phi thuyen, bau tri trong en.
From the earth the sky looks blue; however, from a
spaceship it looks black.
Chiec xe cu con s dung c, tuy nhien toi a
mua khong no.
The old car was usable, but I didnt buy it.
Chng c nghe co ve xac thc, song toi tin han ta
vo toi.
The evidence sounded convincing, yet we believed him
innocent.
Leonardo ve ch vai bc tranh, song chung la
nhng kiet tac.
Leonardo painted only a few pictures, but they are all
masterpieces.
25. va chang, va lai besides,
moreover

Chiec ao at qua, va chang mau khong phu


hp vi ban.
The shirt is too expensive; besides, the colour doesnt suit
you.
Toi se co khg tranh cai vi ong chu, va chang
chang ch li g ca
I wont try to argue with the boss; besides, it wont do any
good.

Le Th Thuy Loan Trang 156


Phng Phap Dch VIET ANH - NG PHAP

26. v for the sack of


for his/ her sack
because of

Anh ta hanh ong nh the ch v tien.


He acted like that just for the sack of money.
Ban khg nen lam hong tnh bang hu ch v mot
mon tien nho.
You shouldnt spoilthe friendship for the sack of a small
amount of money.
Ho a nhng bo v nen hoa bnh.
They made concessions for the sack of peace.

27. va mi th no sooner than


hardly when
as soon as

Nha phi hanh va mi nhn thay Chau Au th ong


lai nhn thay Chau A .
The astronaut had no sooner seen Europe when he saw
than he saw Asia.
(The astronaut had hardly seen Europe when he saw Asia)
Cuoc thao luan nay va ket thuc cuoc thao
luan khac lai bat au
One discussion had hardly been over when another
started.
Va mi an xong th anh lai cam thay oi.
No sooner had he eaten than he felt hungry again.
(Hardly had he eaten when he felt hungry again.)
17. THE METHOD OF DIAGRAMING SENTENCES
(PHNG PHAP HNH O)

AI CNG VE PHNG PHAP HNH O


ay la phng phap trnh bay mot cau tieng Anh
ma ta muon phan tch len tren mot hnh o (a diagram)
gom mot he thong nhng ng nam ngang, ng
thang ng, ng nghieng, ng t oan v.v... theo
nhiem vu cua moi yeu to trong cau.

Le Th Thuy Loan Trang 157


Phng Phap Dch VIET ANH - NG PHAP

Phng phap hnh o rat cu the. Cac yeu to co


lien quan vi nhau trong hnh o, bat chap v tr cua
chung trong cau noi. Khi nhn vao hnh o, ta co the
nhan ra ngay moi lien he cua cac t va cac menh
e trong cau noi. Do o ta co the nhan ra ngay nhiem
vu ng phap cua chung, ac biet nhat la khi cau noi
qua dai.

Phng phap hnh o nham tach mot cau noi ra


thanh nhng manh vun. Moi manh vun la mot yeu to
c at len mot net ve cua hnh o. Ch nhn vao
net ve la ta co the biet ngay c nhiem vu cua moi
yeu to trong cau noi. Noi tom lai, ay la phng phap
giai phau trong ng phap.

17.A. HNH O CUA MAU C BAN SO 1:


Trong hnh o chu ng va ong t c at tren
ng nam ngang va hai yeu to nay c ngan cach
bang ng thang ng.

Subject Verb

Spring has come Dogs bark.


Spring has come Dogs bark

17.B. HNH O CUA MAU C BAN SO 3 VA 4:


Ta nh lai Pattern 3: Subject x be x predicate
adjective
Pattern 4: Subject x be x predicate noun c goi
chung la Subject complement.

Trong hnh o, ong t be (hoac cac ong t ch


trang thai khac) c ngan cach vi Subject complement
bi mot ng nghieng goc.

Subject be subject complement

Le Th Thuy Loan Trang 158


Phng Phap Dch VIET ANH - NG PHAP

Geoege is president Grass is green


George is president Grass is green

I feel sleep The story sounded


untrue
I feel sleepy story sounded untrue
The

17.C. HNH O CUA MAU C BAN SO 5:


Trong hnh o, na ng thang ng ngan cach
Verb va Object.

Subject Verb Object

They visited Halong Bay. The capital attracts many


tourists
They visited Halong Bay capital attracts
tousists
The many

17.D. HNH O CUA MAU C BAN SO 6 VI


DIRECT OBJECT VA INDIRECT OBJECT

Subject Verb Direct object


Indirect object

Father gave me a dollar.


Father gave dollar
me a

17.E. HNH O CUA MAU C BAN SO 7 VI


MOT OBJECT COMPLEMENT

Subject Verb Object (Object complement)

Le Th Thuy Loan Trang 159


Phng Phap Dch VIET ANH - NG PHAP

They elected Mr. Green (chairman)

They have named the baby Richard.


They have named baby (Richard)
the

17.F. HNH O CUA CAC MODIFIERS DI DANG 1


T N.
Cac yeu - to - mo - ta c at tren mot ng
nghieng nam ben di t c mo ta. Mot tnh t
mo ta mot danh t. Mot trang t bo ngha mot ong
t, tnh t, hoac mot trang t.

Subject Verb Object


adj adj adv adj adj
adv

James Watt invented the worlds first famous steam


engine
James Watt invented engine
the worlds first famous steam
The bright red automobile stopped very quickly.
automobile stopped
The red quickly
bright very

The red automobile stopped quickly.


automobile stopped
The red quickly

17.G. HNH O CUA CAC P-GROUP


Trong hnh o, preposition c at tren mo ng
nghieng va object cua no c at tren ng thang
lien hp.

17.G.1 P-group c dung nh Adjective:


The door to the right is open

Le Th Thuy Loan Trang 160


Phng Phap Dch VIET ANH - NG PHAP

door is open
The to right
the

He was a man of action


He was man
a of action

17.G.2 P-group c dung nh Adverb:


We walk for miles. They ran into the house
we walked they ran
for miles into house
the

The manager arrived at the airport in the morning.


manager arrived
The at airport in morning
the the

17.H. HNH O CUA PARTICIPIAL PHRASE DUNG NH


TNH T:

The player passing the ball is Zidance.


player is Zidance
The passing ball
the

17.I. HNH O CUA INFINITIVE PHRASE (TO-GROUP)


Trong hnh o, infinitive phrase c at tren mot
ng lien hp co chan e neu cum t c dung nh
danh t.
Khi cum t c dung nh Adjective hoac Adverb, no
c at tren mot ng lien hp va nam ben di
t ma no bo ngha.

I wanted to go at noon I told him to take the book


to to
go him take book

Le Th Thuy Loan Trang 161


Phng Phap Dch VIET ANH - NG PHAP

at the
I wanted noon I told

John went to call his father. I shall be glad to leave this


place
John went I shall be glad
to to
call father leave place
his this

17.J. HNH O CUA GERUND PHRASE


Gerund c at tren mot ng tam cap.

Rowing a boat is exercise. Seeing is believing.


Rowing boat Seeing believing
a
is exercise is

17.K. HNH O CUA APPOSITIVE


Appositive c at trong ngoac nam sau t ma
no giai thch.

Mr. Jones, the grocer, hurriedly, sold his business.


Mr. Jones (grocer) sold business
the hurriedly his

17.L. HNH O CUA CAU KEP


Cac lien t nh and, but, or, ets c at tren ng
t oan noi hai cau n.

The girls washed the dishes, and the boy drived them.
girls washed dishes
the the
and
boys drived them

Either Tom must enter the contest or we shall lose the prize.
Tom must enter contest
Either
or

Le Th Thuy Loan Trang 162


Phng Phap Dch VIET ANH - NG PHAP

the

We shall lose frize


the

17.M. HNH O CUA CAU PHC VI MENH E


PHU
17.M.1 Hnh o cua cau phc vi menh e tnh t
Noi chung, menh e chnh va menh e tnh t c
trnh bay nh hai cau rieng biet. Menh e phu c at
ben di menh e chnh va c noi vi no bang
mot ng t oan ke t tieng danh t ma menh
e tnh t bo ngha.

I saw an automobile which liked


I saw automobile
an
I liked which

The man who came to the door was my uncle


man was uncle
the my
who came
to
door
the

17.M.2 Hnh o cua cau phc co menh e trang


t.
Khi ve hnh o cac menh e trang t, ta noi ong
t cua menh e trang t vi t ma no bo ngha
nam trong menh e chnh. Lien t nam tren ng t
oan noi hai menh e.

I will leave Dalat when my friend comes


I will leave Dalat
when
friend comes

Le Th Thuy Loan Trang 163


Phng Phap Dch VIET ANH - NG PHAP

my

The lesson was so difficult that we could not finish it.


lesson was difficult
the so
that
we could finish it
not

He was older than his brother was.


He was older
than
brother was
his

Jetplaned cannot fly above the atmosphere because the


oxygen in the atmosphere is necessary for the buring of their
fuel.
Planes can fly
Jet not above
atmosphere
the because
oxygen is necessary
the in for
atmosphere burning
the the of fuel
their

17.M.3 Hnh o cua cau phc vi menh e danh t


V mot menh e danh t thng khong co lien t
noi vi phan con lai cua cau nen no c trnh bay
len hnh o nh mot menh e tach biet, nhng c
noi vi menh e chnh bang mot ng t oan tai
mot v tr thch hp tuy thuoc vao chc nang cua no
la chu ng hay tan ng.

Le Th Thuy Loan Trang 164


Phng Phap Dch VIET ANH - NG PHAP

I know he is a thief. I know that he is a thief.


he is thief he is thief
a that a
I know I know

That he was a thief was never double


he was thief
that a
was doubled
never

17.N. HNH O CUA CAU KEP PHC


Cau kep phc co ca ac tnh cua cau kep lan
cau phc.
I liked the Dalat climate because it was mild, but Tam
preferred the cold winters of Hanoi.
I like climate
because the Dalat
but it was mild
Tam preferred winters
the cold of
Hanoi

MASTERY TEST
PARTS OF THE SENTENCE

Test 1: Draw a line under the verb and two lines under the
subject.

Le Th Thuy Loan Trang 165


Phng Phap Dch VIET ANH - NG PHAP

Ex: Promply at three oclock the bank closes

Where is the sencond section of this newspaper?


Is your brother majoring in chemistry at college?
Will the Presents speech be televised?
There are over 25.000 diffirent kinds of fish in this watery
world.
Do you really want low-salaried job?
There might have a bad accident here?
Would one of you help me with this suicase?
The supply of water has already been exhausted.
In the earth crust there is more aluminum than any other
metal.
Throughout the performance the man slept.
Down the street at last came the parade.
Why has our car been having so much engine trouble?
The mayor must have prepared his speech very carefully.
Symbolic of America is the Statue of Libery.
On the floor behind the desk were the missing papers.

Test 2: Find these parts of the following sentences: subject,


verbs, direct object, indirect object, and subject complement.
Jack is giving the car a good washing.
The policeman asked us a few questions.
Professional hockey has become quite popular in recent
years.
The rescue team began an intensive air-sea search.
Anns apathy toward everything gave the doctors cencern.
The Vayager I setellite was visible that day.
He teaches several classes in English literature.
The bad weather had delayed our trip.
The African lon in repose seems only a big, friendly cat.
In my opinion overlong movies are tiresome.
Yellowstone is famous for its geysers and hot springs.
The actors best friend is not always his scriptwriter.

Test 3: Find the compound parts in each sentence below.


Identify the compound part as subject, verb, direct object,
indirect object, object of preposition, subject complement.

Le Th Thuy Loan Trang 166


Phng Phap Dch VIET ANH - NG PHAP

Two of Janets favorite hobbies are buying skirts and buying


shoes.
One must know the difference between astronomy and
astrology.
Dogs have five toes on the front feet and four on the rear.
Do you want to rearrange the furniture or play basketball?
Astronomy and astrology should never be confused.
The Sphinx has the body of a lion and the head of a woman.
We have neither an upstaire maid nor a downstairs maid.
When he meets people, he is a loof and cold.
Is Sue the one in the black skirt or the one in the grey suit?
Stephen Crane was a ministers son, a fourteenth child, a
famous writer at twenty-two, and a dead man at twenty-nine.
The British ship struck her colours and bowed to the
American ship.
California produces more fruit and vegetables than any
other states.
My grandfather has a keen mind, a strong will, and a fiery
temper.
Thomas Jefferson was archited, inventor, farmer, and
landscape gardener.
Scientists have named and described over 600,000 different
kinds of insects.
We brushed cobwebs from the walls and ceilings.
Mr. Rostow gave Jack and me boxing lessons.
The test was difficult but fair.
Dr. Dooley and his staff set up two jungle hospitals in Laos.
The chocolate cake requires fewer eggs and less sugar.

Test 4: Find complete verbal phrase in eah sentence.

The Savannah is the first merchant ship powered by a


nuclear reactor.
Barcelona has been penalized five yards for delaying the
game.
Raising beautiful flowers is my aunts main interest.
Stans ambition is to become a pilot.

Le Th Thuy Loan Trang 167


Phng Phap Dch VIET ANH - NG PHAP

Expelled from New York University, James Cooper served for


these years in the Navy.
All channels will broadcast the launching of the spaceship.
He claims to have discovered a new chemical substance.
Joe is capable of earning high marks.
It must be wonderful to be able to read French.
Her inviting me to dinner was complete surprise.
He was determined to be a great success in his new job.
Janet enjoyed reading the biographies of men who
overcame poverty.
Another attempt to explore the planet Mars will be made.
The senator favors increasing the sales tax.
To make prompt decisions is essential in business.
He devotes an hours each day to exercise outdoors.
We call the hospital, fearing the worst.
Finding lodging for the night was the hikers next problem.
Having read the papers, Mr. Green knew the whole story.
Randy wants to tell his side of the story.
Signing the papers took a comparatively short time.
The science of growing plants without soil is called
hydroponics.
The boy standing besides the diving board is a champion.
Our instruction was to accept only money.
To give only moral support is not enough.

Test 5: In each of the following sentences one phrase is


underlined. In the first blank write the number which identifies
the kind of phrase.
Preposition phrase 3. Participial phrase
Gerund phrase 4. Infinitive phrase
In the second blank write the number, which describes the
function of the phrase in the sentences.
1. Adverb 2. Adjective 3. Noun
Ex: After the storm, the game was resumed 1 1
Ex: To delay a minute longer will be disastrous 4 3

The girl in the picture is my cousin Jane. - -


I shall enjoy seeing you in the dance. - -

Le Th Thuy Loan Trang 168


Phng Phap Dch VIET ANH - NG PHAP

The skaters were gliding across the frozen - -


pond. - -
Seeing Europe was her pricipal summer - -
objective. - -
Having played the entire game, he was - -
exhausted. - -
I have nothing to say to all of you. - -
Studying American history is my pastime. - -
Up the narrow path we walked in complete - -
silence. - -
The heroes of the American Revolution are - -
legendary. - -
To dance cha-cha-cha was Sams great desire.
Keeping a budget requires great patient.
We could see Jack standing by the old - -
Mercedes. - -
At the top of the hill stood an old pagoda. - -
I write home quickly to please my mother. - -

- -

Can he eat three steaks at a single meal? - -


His main objective is mastering two - -
languages.
The Ref Cross was praised for giving aid to - -
the victims. - -
After dinner is the best time for the long walk.
His reputation has spread beyond his own
university.
Because of the floods, the highway was
declared impassable.
Receiving no further encouragement, I
withdrew my application.
Sylvia entered the room carrying a big
bouquet.
Worried about the missing child, the
babysister called the police.
The man renting the apartment paid in
advance.

Le Th Thuy Loan Trang 169


Phng Phap Dch VIET ANH - NG PHAP

The sale advertised in todays Time will last


until Friday

SENTENCE AND CLAUSE

Test 1: Decide whether each sentence below is a simple


sentence or a compund sentence. In simple sentences find all
compound predicates.

Janice learned French and got a job at the United Nations.


Millions of Americans have high blood pressure and dont
know it.
The attorney questioned the bank president and examined
his files.
Jack sanded the table and then refinished it.
Jefferson was shy in speaking, but he wrote in a powerful
style.
Judy left the room, and the whole meeting seemed to break
up.
Martinique is an island in the West Indies and a department
of France.
Keith works harder than Dick but accomplished less.
I dropped the cup, and the handle broke off.
Dr. Fleming was knighted and later won the Nobel Prize.
You can enroll at the state university, or you can get a job.
Harrison received a smaller popular vote than Cleveland but
won the election.
That puzzle is difficult, but it can be solved.
We arrived at midnight and went straight to bed.

Test 2: Underline the subordinate clause in each sentence


below.

Here is a list of the people who are on the committee.


The only thing that will cut a diamond is another diamond.
Hailstones are raindrops that froze in the atmosphere.
Dickens is probably the most popular English novelist.
Adjust the thermostat before you leave.

Le Th Thuy Loan Trang 170


Phng Phap Dch VIET ANH - NG PHAP

Flies can walk on ceiling because they have stickly pads on


their feet.
Wait until you receive the signal.
The experts predicated that 50,000,000 people would vote.
All the students who got grades lower than B decided to
strike
Ill go early provided that I can get a ride.
Dad usually files when he goes to Chicago on business.
That they were in need of food was easy to see.
Some of us remained until all of the crowed had left the hall.
The kitten ate as if he had been without food for days.
Jim scowls when Janie tells him his faults.
A speleologist is a man who explores caves.

Test 3: Find each noun clause. Decide whether it is used as


subject, direct object, object of a preposition, or subject
complement.

She asked what I was doing


Sally apologized for what she had done.
Al has decided that wants to become an architect.
Who wrote the letter is still a mystery.
I doubt that teaching machines will ever replace teachers.
The fact that I am his son makes no difference.
Linda thought a fiord is a Swedish automobile.
He had all the habits of what he supposed was so high
society.
That the major had resigned was a shock to the city.
Lindas secret is that she is engaged to be married.
Everyone knows that Linda is engaged to be married.
That she wore no make up fascinated me.
My guess is that he will refuse.
The author says that there is a shortage of mans work in
our society.

Le Th Thuy Loan Trang 171


Phng Phap Dch VIET ANH - NG PHAP

Test 4: Combine two sentences to make one ggod sentence,


using a noun clause.

Poets and painters show us the beauty of everyday things.


Do you agree to this statement?
We may believe very strongly in the theory. This, however,
is no proof of its truth.
Ther may be other uninhabited planets. This possibility is
hard for me to believe.
Lighting is an electric discharge. By flying a kite during a
thunderstorm Benjamin Franklin proved this fact.
Approximately ten million meteors strike the earth
everyday. This has been estimated by scientists.
Almost everybody should drink six or eight glasses of water
a day. Modern doctors agree on this.
The label of drug indicates the presence of any dangerous
compounds. The law requires such a label.

Test 5: Combine two sentences to make one good sentence


by changing on of the predicate verbs to a participle.

Figo passed the ball to Gomez. Figo danced back.


The bathysphere is a steel sphere. It weights five thousand
pounds.
Grand Colee Dam is one of the great engineering triumphs.
It was begun in 1933 and completed in 1941.
The ostrich can outrun any other African animal. It takes
strides of twelve feet ot more.
George Washington retired to Mount Vernon.
Roosevelt Dam was completed in 1911. it irrigates thirteen
thousand farms in Salt River Valley.
Kipling traveled in America. He spent some time in San
Francisco and then he came slowly east.
In ancient times a mirror was a polished sheet of metal. This
was called a speculum.
Kerosene and gasoline are two liquid fuels. They are
obtained from petroleum.
The money was returned. It was taken from the desk.

Le Th Thuy Loan Trang 172


Phng Phap Dch VIET ANH - NG PHAP

PHAN GIAI BAI TAP

Exercise 1A:
*3) Pattern three. *4) Three. *5) One. *6) One.
*7) One. *8) Three. *9) Five. *10) One.
*11) Seven. *12) Six. *13) Four. *14) Three.
*15) Five. *16) One *17) One *18) One.
*19) Three *20) Four *21) Four *22) One.
*23) Seven *24) Seven *25) Five *26) Two.

Exercise 1C:
*1) at the station *2) to a concert *3) late
*4) in a hurry *5) soundly *6) off
*7) in the distance *8) at work *9) in a car
*10) with their friends *11) in the air *12) there
*13) across the river *14) very hard *15) in a
restaurance
*16) on the chair

Exercise 3A:
some sour green eating apples
her small, round pink face
those two very famous university professoers
Lindas three pretty yellow cotton shirts.
a very large rectangular swimming pool.
these two large glass front doors.
an old, large cigar box
an enormous red and yellow umbrella
the first three questions

Le Th Thuy Loan Trang 173


Phng Phap Dch VIET ANH - NG PHAP

Exercise 3B:
6. They have auctioned off Mr. Greens ten very rare,
ancient gold coins.
7.
Gold is a precious, malleable yellow metal.
The villagers saw an unusual triangular gray flying object.
The moon is the giant, airless, waterless, and uninhabited
sphere.
He has chosen the inexpensive brown deerskin shoes.

Exercise 4A:
*1) into a tree/ adverb. *2) for the meeting/ adjective;
within a week/ adverb. *3) of glue/ adjective. *4) on the corner/
adjective; since July/ adverb. *5) of the engine/ adjective; in this
plant/ adverb. *6) for Mr. Horst/ adjective; in his desk/ adverb.
*7) beside the river/ adverb; in the canyon/ adjective. *8) until
the day/ adverb; of the family reunion/ adjective. *9) during the
crisis/ adverb. *10) into the sea/ adverb. *11) of accident/
adjective; on the front page/ adverb. *12) at the airport/ adverb;
on time/ adverb. *13) on the horizon/ adverb. *14) of modern
times/ adjective. *15) with difficulty/ adverb. *16) of the general
strike/ adjective; throughout the country/ adverb. *17) of the
lung/ adjective. *18) of the astronauts/ adjective; before noon/
adverb. *19) for the Primer Minister/ adjective; in the city hall/
adverb. *20) near the gate/ adjective. *21) of fish/ adjective. *22)
of water/ adjective. *23) in the air/ adverb. *24) on the site/
adverb; of the old curch/ adjective. *25) into the lunar surface/
adverb. *26) since 1988/ adverb; of building material/ adjective.

Exercise 4B:
6. We can use the sunlight as a soure of fuel for
electric plants.
7. What we learn at school, from friends, in society are
helpful.

Le Th Thuy Loan Trang 174


Phng Phap Dch VIET ANH - NG PHAP

8. In time of war, they used this shop as a floating


hospital.
9. Thanks to science many laws of nature have been
discovered.
Columbus began his voyage across the Atlantic Ocean at
the end of the 15th Century.
Scientists all over the world are testing a new vaccine
against AIDS.
Thanks to modern medicines and medical equipment,
people can live longer than they used to.
Hiroshima had once been a big city with a famous
shipbuilding industry.
Heart disease is one of the chief causes of death.
A person without a job usually lives in boredom anf poverty.
To my children, feeding sugar canes to the elephants is part
of the fun of going to the zoo.
The hat is floating on the water at the bottom of the well.
Surgeons can use laser beams as accurate scalpels.
The revolution of the earth around the sun xauses the four
seasons.
Scientists cant calculate the orbits of spaceships without
the use of computers.

Exercise 4C:
Tam went with his mother to the city for the first time.
The bomb exploded in the air at the height of twenty metres
over the brigde.
With the help of electronic calculating machines, we can
control machines, calculate the orbits of satellites and
spaceships and even translate languages.
The flight into space for the study of the earths atmosphere
has been postpond.
The men outside the factory are the sacked workers.
He didnt come for the meeting on account of a bad cold.
The movement for peace and against nuclear war spead
throughout Europe.
The government in power at that time was young and weak.
We need a market for our agricultural products.
The film was successful beyond everyones expectations.

Le Th Thuy Loan Trang 175


Phng Phap Dch VIET ANH - NG PHAP

The pagoda on the top of the mountain was built in the 15th
Century.
Columbus was a navigator of ralent and much ambition.
The Mekong II left the harbour yesterday for Singapore.
The fight against corruption and smuggling hasnt been
successful.
The man with the microphone is marketing a new product.
The bank had films showing the robbery in progress.

Exercise 5A:
*1) complaining/ onject of preposition. *2) whispering/
subject/ *3) skiing/ subject. *4) talking, sleeping/ object of
preposition. *5) quitting/ subject. *6) arguing/ subject. *7)
understanding/ object of preposition. *8) ringing/ subject. *9)
dieting/ subject complement. *10) riding, golfing, swimming/
object of preposition. *11) swimming/ subject. *12) hunting/
object of preposition.

Exercise 5B:
*1) curving/ lines. *2) burning/ skin. *3) tinkling/ sound. *4)
depressing/ defeat. *5) promising/ sources. *6) injured/
passengers; passing/ automobile. *7) delayed/ arrival. *8)
hurried/ inspection; stolen/ motocycle. *9) roaring/ frames. *10)
uneducated/ children. *11) advanced/ student. *12) vanquished/
warriors; ravaged/ villages. *13) pierced/ ears. *14) crying/ baby.
*15) stolen/ money.

Exercise 5C:
1. to leave/ noun 2. to compromise/
noun
3. to delay/ noun 4. to survive/
noun
5. to forget/ adj 6. to escape/ adv
7. to spend/ adj 8. to talk/ adv
9. to emphasize/ adj 10. to rest/ noun
11. to impose/ adj 12. to win/ adv
13. to applaud/ adv 14. to deliver/ adj
15. to come/ adj; to leave/ adj.

Le Th Thuy Loan Trang 176


Phng Phap Dch VIET ANH - NG PHAP

Exercise 6A:
swimming too soon after a heavy meal/ subject.
the shooting of white bears/ object
reading good books/ subject
barking at the mailman/ object
telling a joke effectively/ subject
discussing the matter at the next meeting/ object
pursing this button/ object of preposition
jogging in the morning/ object
swimming in the afternoon/ object
asking my permission/ object of preposition.

Leaving the house/ object of preposition


imitating others/ object of preposition
weaving straw hats/ raising cacao/ subject complement
clearing up after a party/ subject
getting ready for a party/ subject
dancing when you are tires/ subject
being provider for a family/ subject
writing sports events for The Time/ object
parking too ong/ object of preposition
directing a play/ subject
his winning the first prize/ object of preposition
reading many books/ object
complaining about the weather. Subject
gardening/ object of preposition
brushing your teeth/ subject.

Exercise 6B:
driving the tractors: participial phrase
testing jet engines: gerund phrase
orbiting the earth: paticipila phrase
dismissing the trouble makers: gerund phrase
raising banabas: gerund phrase
showing the far side of the moon: participial phrase
hoping to get tickets: participial phrase
forcasting the weather: gerund phrase
selling gas for the filling: gerund phrase

Le Th Thuy Loan Trang 177


Phng Phap Dch VIET ANH - NG PHAP

driving to th city: participial phrase


conquering the bottoms of the sea: gerund phrase
having lost the original orbit: participial phrase

harvesting the crop: gerund phrase


earning high marks: gerund phrase
climbing Mont Blanc: gerund phrase
being disagreeable: gerund phrase
talking in the audience: gerund phrase
standing quietly: participial phrase

Exercise 6C:
Havong studied for a week, Mary felt she was ready for the
test.
Having read about the new construction work
Having explored the cave,
Having bought a ticket,
Having missed the first act
Having reached the wall
Having won the match
Having signed the treaty
Being clumsy
Having been famous

Exercise 6D:
2. The function of the satellite is studying natural
resources on the earth's surface.
3. Studying while you are watching television isnt
effective.
4b. My friend likes solving difficult mathematical
problems.
4c. Solving difficult mathematical problems takes a lot
of time.
5b. Studying medicine is her wish
5c. My friend doesnt like studying medicine.
6. You can start the machine by pushing this red
button.
7. Swimming before breakfirst is very good for your
heslth.

Le Th Thuy Loan Trang 178


Phng Phap Dch VIET ANH - NG PHAP

8. Sending an astronaut to the planet Mars and


bringing him back is possible.
9. Fighting the enemy and protecting our country is
the duty of the youth in time of war.
10. Painting and writing are the arts that require talent
and persistence.
11a. We should always avoid polluting the environment.
11b. In many European and American countries, the law
forbids polluting the environments.
12. Her inviting me to dinner surprised everybody.
13. Scolding the children wont do any good.
14. Making promp decisions is necessary in business.
15. Persevering after than you have failed is a test of
character.
16. Spending more than you earn is the shortest road to
poverty.
Scientists have dreamed of transmitting power without the
use of wires.
Her mistake was reaching the directions carelessly.
Climbing Mount Sapa gave me a memorable experience.
Running a summer camp is a heavy responsibility.
Exercise 6E:
being an aviator/ Tom
waiting for the postman/ she
swinging the racket/ Don
passing the ball/ player
having heard the signal/ runners
having arrived late/ Teff
knowing what you need/ you
knowing the exact route/ Joe
hoping to finish the report/ I
having finished their work/ nurses
waiting for the show ti begin/ audience
pushing hard/ he

Exercise 6F:
These are the pictures showing the entire Ha Long Bay
Thinking that he will die, Roy Williams wants to come back
home.

Le Th Thuy Loan Trang 179


Phng Phap Dch VIET ANH - NG PHAP

Hoping to find a job in the city, the young man left his
native village.
Having finished the sixth orbit, Gagarin began to radio
signals to the control station on the earth.
Having dropped bombs, the F-5 jet fighter turned
immediately and flew off at full speed.
Taking the boys temperature, the nurse began to take care
of him.
Rallying his footsoldiers anf horses, Le Loi fought and swept
to victory over the enemy at Chi Lang.
Mr. Lee, driving vegetables and fruit for a company, earns
high wages.
Having decided to become a diplomat, Erik began to study
foreign languages intensively.
The hills appearing in the distance reminded Tom of his
peaceful country village.

Thinking while everyone is playing, Henry has solved the


problem.
Having read the directions, Elma tried to assemble the
loudspeaker.

Exercise 7A:
written hurriedly/ letter
crowded with cars/ street
Build over 80 years ago/ house
struck by lightning/ cottage
stopped at the border/ travelers
once used as a library/ building
written yesterday/ news
guided by radar/ pilot
shown here last week/ film
having been badly defeated in the last election/ senator
seen through the water/ strick
covered with snow/ mountains
placed in a good position/ television set
stung by his critics/ mayor
exhausted from the climb/ hikers
neatly dredded/ children

Le Th Thuy Loan Trang 180


Phng Phap Dch VIET ANH - NG PHAP

armed with a knife/ hunter.

Exercise 7B:
4. Flowers grown in a greenhouse wilt sooner than
those grown in a garden.
Cars made in Japan are often cheaper than those
manufacture in Europe.
Guided by rada, the pilot landed safety in bad weather.
The Oil Man and the Sea. Written by Hemingway, is very
exciting.
The products produced by this factory are of good quality.
These are three students chosen among the best ones.
Penicillin, discovered by Dr. Fleming, has saved millions of
people.
Glass is made from melted sand mixed with soda and lime.
Columbus had to employ criminals released from prisons to
make up crews.
Cellulose is a subtance easily found in the nature, in trees
and plants, in vegetables and fruit.
Rom, Paris and London are big cities in Europe visited every
year by millions of tourists.
Begun in 1939 and ended in 1945, the Second World War
caused the death of millions of people all over the world.
The motion picture is an art form loved by many people.
The plane plane shot down by the guerrillas was a jet-
fighter F5.
Shepard was the first American astronaut ever launched
into space.
The super-highway suggested by foreign engineers has
been rejected.
The bridge destroyed by the last flood hasnt been rebuilt.
With two atomic bombs dropped over Hiroshima and
Nagasaki, Japan was brought to her knees.
Repulsed in New York, Washington led his dwindling forces
across New Jersey.

Execise 8A:
*1) to work hard/ subject. *2) Henry play outside/ object. *3)
to cohere/ subject; to stick together/ subject complement. *4) to

Le Th Thuy Loan Trang 181


Phng Phap Dch VIET ANH - NG PHAP

reject the scholarship or not/ subject. *5) to lie him/ subject. *6)
to postpone the game/ subject complement. *7) to test
chemicals/ appositive. *8) to whistle three times/ subject
complement. *10) to travel in Europe/ subject complement. *11)
the strange man come in/ object. *12) to become a pilot/
appositive. *13) to make money/ subject; to keep it/ subject. *14)
to choose medicine/ object. *15) to decorate this room/ subject.
*16) to be a good conversationist/ subject; to be a good listener/
subject complement. *17) to climb the mountain before us/
subject.

Exercise 8B:
to see the old home town/ noun
to visit the old home town/ noun
to visit the old home/ adjective
to see better/ adverb
to spend on books/ adjective
to applause the winning team/ adverb
not to accept his advice/ noun
to radio for assistance/ adverb
to go with us/ adjective
to get the summer job/ adjective
to win the game/ adverb
to see/ adverb
not to see her son again/ adverb
to make soap anf comestic/ adverb.
to study the clouds/ noun
to lunch another spaceship/ adjective
to do today/ adjective
to keep the southern borders of China safe/ adverb
to lose for my country/ adjective
to give only moral support/ noun

Exercise 8C:
3. To send men and scientific equipment to the
bottoms of the sea is possible.
4. To talk to him is like him.
Her hope is to pass the next examination.
To attend this training course will take at least six months.

Le Th Thuy Loan Trang 182


Phng Phap Dch VIET ANH - NG PHAP

To understand the past helps us to predict the future.


Her ambition, to be accepted at Harvard University, has
been realized.
To study and labor well are small but very important daily
deeds to you, boys as well as girls.
The poets aim was to go and search for the beauty of
nature.
Years ago the moons function was to provide moonlight for
the romanticists, but now its function is to provide a target for
the space scientists.
To have been chosen a member of the club was a great
honor.
The purpose of the article was to mold public opinion.
The tendency of the youth of today is to choose science and
technology.

Exercise 8D:
The nurse is ready to call the doctor.

The factory is equippedwith more modern machinery to


speed up production.
Scientists are anxious to find out an effective vaccine
against AIDS.
We are very grieved to hear about his death.
The company has doubled the investment in an effort to
increase the production of homemade cars.
Jesse worked night and day to support his family of seven
members.
The soldiers were very anxious to re-capture the lost
defence line.
The law to force riders and drivers to keep right dated back
to the Napoleonic age.
The pilot of World War I had no radio to send or receive
messages.
There will be more difficulties to overcome and many
enemies to defeat.
She expressed her wish to earn her own living.
The government has taken new measures to deal with
inflation.

Le Th Thuy Loan Trang 183


Phng Phap Dch VIET ANH - NG PHAP

We should disarm to survive in the nuclear age.


You havent kept your promise to write to me more
frequently.
Ill show you the fastest way to solve this mathematical
problem.
The best way to receive a man is to blow your breath into
his lungs.
The Great Wall of China was built to keep the southern
borders of China safe.

Exercise 9A:
for you to think I dont value your advice/ adverb
for your brother to come with you/ adverb
for yoy to read/ adjective
for men and women to sit apart/ adjective
for their country to be liberated/ adverb
for their only daughter to many a poor school teacher/
adverb.
for the bus to start/ adverb
for a rich window to find a handsome husband/ noun
for us to choose/ adjective
for you to accept his proposal/ adverb
for the manager to sign/ adjective

Exercise 9B:
These are the exercises for the pupils to do at home.
Tam paased the newspaper for everyone to read.
I have bought two tickets for my parents to go to Nha Trang.
There will be a few minutes for the astronaut to walk in
space.
In Europe there are many famous universities for students
to choose.
The plan for the factory to produce 30,000 cars per year has
been approved.
That is the best time for children to go to school.
I will give you the book for you to read during the trip.
The test for a manmade satellite to destroy another one in
space was a success.

Le Th Thuy Loan Trang 184


Phng Phap Dch VIET ANH - NG PHAP

The plan for Mr. Smith to replce Mr. Thompson as the


director pleased everybody.
The captain gave an order for his men to fire the enemy
ship.
The arrangement for the American, European and
Japnanese learders to meet in Geneva wa a failure.
My uncle has a wish for his son to study medicine in France.
The scientists havent found an ideal place for the satellite
to land on the planet Mars.

Exercise 10A:
*1) as if: subordinating conjunction. *2) neither nor:
correlative conjunction. *3) till: subordinating conjunction. *4)
where: relative adverb. *5) or: subordinating conjunction. *6)
provide: subordinating conjunction. *7) nevertheless: conjunctive
adverb. *8) whatever: relative pronoun. *9) either or:
correlative conjunction. *10) before: subordinating conjubction.
*11) since: subordinating conjunction. *12) therefore:
counjunctive adverb. *13) so that: subordinating conjunction.
*14) consequently: conjunctive adverb. *15) where: relative
adverb.

Exercise 11A:
because we were tired/ adverb.
what the law requires/ noun
as we drove up/ adverb
who discovered America/ noun
when the bell rings/ adverb.
whoever is qualified for the job/ noun
wher president Ho had lived anf worked/ adjective
unless it rains/ adverb
what you want/ noun; what I want/ noun
which was bombed heavily in 1945/ adjective
that he would win the game/ noun.
that her husband had been dismissed/ noun
that science will lead ti a better life for man/ noun
where and when an earthquake is coming/ noun
when he comes/ adverb
wherever you can find a place/ adverb

Le Th Thuy Loan Trang 185


Phng Phap Dch VIET ANH - NG PHAP

as if he had been without food for days/ adverb


whom we hired this morning/ adjective.
whose pin she was wearing/ adjective
that Marion could get up to morrow/ noun
what you want without making an effort/ noun
that barks the loudest/ adjective.
wherever it went/ adverb
whom you spoke to/ adjective
that he is intelligent/ noun (appositive)
where we were told/ adverb
that his business is steadily improving/ noun
as though he were intelligent/ adverb
so that she could be with her husband/ adverb

Exercise 11B:
2. The theory that there is no life on the planet Mars
has been proved.
3. Nobody can predict who will be chosen as the best
player of this football season.
4. Scientists are very optimistic about the theory that
electricity can be transmitted without the use of wires.
5. The scientist refused to discose what he had
discovered.
6. I cant understand why he acted like that.
7. I asked her what branch of study she would choose
and how long it would take.
8.
which country he wuld go to, who he would negotiate with,
and what problems he would discuss.
9b. Do you know who invented the microscope?
9c. Who invented the microscope is unknown to the
whole class.
10. The problem is whether or not you want to join the
trip.
11. Superstitions people believe that lightning is a
weapon of gods.
12. Is it true that he can speak three languages?
13. That electricity is a source of heat may be seen in
the electric iron and the electric stove.

Le Th Thuy Loan Trang 186


Phng Phap Dch VIET ANH - NG PHAP

14. Everybody thinks that the plan is not practical and


that it will fail.
15. Thomas Edison believed that his deafness increased
his power of concentration.
16. The study of history is necessary for whoever id
interested in the future of our world.
I asked where she lived, how she earned her own living, and
why she had left the city.

Exercise 11C:
The aerial photographs showed the craters ehich no one
had ever seen.
The energy which we are using came from the sun.
Do you have enough money (which) you need for the trip to
Hanoi?
I cant understand the reason why he said so.
The Great Wall of china, one of the seven wonders in the
world, is the only thing on earth which can be seen from the
moon.
The goal for which we strive is world peace.
Srely the story he has told us is not true at all.
The art gallery exhibited the famous paintings which they
had collected all over the world.
That is a small village where the poet was born.
A chemical element is a substance which cannot be
separated into any simpler substances.
Thailand is the only country in Southeast Asia which has
never been ruled by a European power.
The facts which came to light startled everybody.
Thw people who are benefiting most are the great
industrialists.
Many of the men we saw in the parade were war veterans.

Exercise 11D:
We wont go camping unless the weather is fine.
An industry cant thrive if there isnt a market for the goods
that is produces.
These boys are educated well akthough they are poor.

Le Th Thuy Loan Trang 187


Phng Phap Dch VIET ANH - NG PHAP

David needed a job badly so that he could support his


family.
Whenever there is an accident, acrow gathers.
When Mozart was a child, he showed his great musicianship
Our enemies were defeated because they didnt have a
righteous cause.
Jet planes cannot fly above the atmosphere because the
oxygen in the atmosphere is necessary for the burning of their
fuel.
It seems to me that the summer days are shorter than they
used to be.
The gravity on the moon is six times smallet than that on
the earth.
Persons living in high places have a large number of red
corpuscles in blood than those living in lower places.
The dog looked as if he would attack us.
Park the car wherever you can find a place
It is very difficult to take the picture of Venus because it is
constantly veiled by dense clouds.
Zidance raised the silver cup so that everyone could see it.
When the pain starts, take two aspirin.
Tony is as capable as his sister is.
Millions os people have high blood pressure because they
overeat.
The drug company expressed its regrets and withdrew the
vaccine because it wasnt effective.
The sale will contibue until all the cars are sold.
You cant expect to receive letters unless you write some.

Exercise 12:
The fire was extinguished by the brave firemen before much
damage had been done.
Every child awaits with excitement the time he can go to
school.
The operation was performed by the doctor before much
harm was done.
I sold a guitar with steel strings to a beautiful girl.
My firend agreed to help me on the next day.

Le Th Thuy Loan Trang 188


Phng Phap Dch VIET ANH - NG PHAP

I continued to live at my uncless home to save money


when I went to college.
I always prefer to read scientific books.
The boy sat in the window looking at the parade.
The invention, which was not at first accepted, proved to be
a very valuable time saver.
For three days I ate almost nothing.
Aunt Bertha met by accident the man she later married.
Harry bought a car that had been driven less than ten
thousand miles from a little old lady.
She wore a decoration, which was made of cherries,
feathera, and wild rice.
With a long wooden stick, George killed the mad dog that
sprang at him.
Jane witnessed with her own eyes the shoplifter take the
diamond necklace.
My father keeps in the buffet the medals and decorations
that he won during the War of the Gulf.
George met his wife on the subway when he returned from
an office party.
Teachers who trust students are occasionally disappointed.
Some of the native women carried on their heads large
baskets, which were filled with soiled laundry.
Often people who go to the beach dont like to swim.
One summer our fruit stand was closed because we had
sold to customers apples that had worms.
Not all of the spectators can get into the stadium.
Mary was praised by the mayor for heroism.
Slowly Dorothy was learning to drive.

Exercise 13A:
Betty prefers to go to parties, to see plays, and to play the
piano.
He plays football skillfully.
The outside of the school was made of wood, brick, and
stone.
John is an engineer who works for a car company.
He told us to go by train and (to) come back soon.
The apinting was neither attractice nor enjoyable.

Le Th Thuy Loan Trang 189


Phng Phap Dch VIET ANH - NG PHAP

Toms job was driving trucks and repairing engines for a


company.
Not only is the book expensive but it is also uninteresting.
The book is not only expensive but also uninteresting.
We like leaders who are honest and fearless.
Most successful pictures are well acted, tell an exciting
story and have a strong central theme.
He did the work without mistakes.
this applies to our living conditions, our food, and our
clothing.
Her duties were greeting guests and writing letters.
The policeman told the driver to park his truck and to go to
the police station.
The class in reading learned to read faster and to read with
greater comprehension.
We are losing money at the factory; we must either raise
our prices or close down.
The teacher told the students to write the answers to the
questions and to finish within an hour.
He is a pianist and trumpeter.
I like drinking beer, playing cards abf singing.
The guest was told to pack his bags and to leave
immediately.
Gloria has nice hair and lovely eyes and wears beautiful
clothes.
Jerry has ambition and seemingly unlimited energy.
We liked the contents and the style of the book.
Neither was he ready for a job nor did he want a further
education.

Exercise 13B:
Neither the moon nor the planet Mars has an atmosphere.
Both the moon and the planet Mars dont have an
atmosphere.
He refused not only to see us but also (to) write to us.
I believed neither interesting, nor amusing, nor instructive.
David excelled Tom both in mathematics and in physics.
The driver accepted neither the mistakes nor the repair of
the damage.

Le Th Thuy Loan Trang 190


Phng Phap Dch VIET ANH - NG PHAP

Both children and adults were pleased with the show.


Either you must study hard, or you will fail in the next
examination.
He complained that no one had helped him, that the work
had been too much for him, and that he needed more help to
finish.
She needed not only victory but also glory.
His statements were not only incorrect but also
unnecessary.
In the park, I saw old men playing chess, mothers wheeling
baby carriages, and students reading books.
At the party she neither sang songs, nor told funny stories,
nor talked with anybody.
Changi Airport in Singapore is not only larger, more modern
but also more beautiful than shanghai Airport.
I think he has accepted the job because he likes it and
because the income is high.
The immigrants rushed to those deserted areas because
they hoped to find gold, to become rich and to change their life.
If we want to pass the next examination, we must study
hard and intensively.
He came to that conclusion because of what he had
suspected, what he had heard, and what he had actually seen.
He prefers living in a city to raising cattle, growing fruit
trees, or living close to nature.
The dentists work id filling, taking out teeth, and fitting
artificial teeth.
The star of the show was tall, slender, and blond-haired.
The course will teach you haw to use diesel engines and
how to repair and maintain them.
Im going to university to acquire knowledge, to make new
friends, and to equip myself for a career.

Exercise 14A:
*1) clause. *2) phrase. *3) clause. *4) phrase. *5) clause.
*6) phrase. *7) phrase. *8) phrase. *9) clause. *10) phrase. *11)
phrase. *12) phrase. *13) clause. *14) phrase. *15) clause. *16)
phrase. *17) phrase. *18) phrase. *19) clause. *20) phrase. *21)
clause. *22) phrase. *23) phrase. *24) phrase. *25) clause.

Le Th Thuy Loan Trang 191


Phng Phap Dch VIET ANH - NG PHAP

Exercise 14B:
if you dont like this kind of work
because I had a headache
that the boys had made great progress.
as if it would fail
what secret word opened the gate
who doesnt acknowledge his mistakes
when food is oxidized in muscle cells
that the bus could not be repaired
how migrating animals fond their way on their long
journeys.
that electricity is a source of heat
when Benjamin Franklin lived in Paris
whose atmosphere contains water vapor
where she goes
what he had done
that they were in need of food
when we left the house
until you receive the dignal
before you direct dial to Singapore
that he will refuse
what really made the party a success.

Exercise 15A:
*1) compound subject. *2) compound object. *3) compound
predicate. *4) compound verb. *5) compound predicate.
*6) compound verb. *6) compound verb. *7) compoun object.
*8) compound subject. *9) compound predicate. *10) compound
predicate. *11) compound subject. *12) compoud predicate.
*13) compound predicate. *14) compound predicate.
*15) compound predicate. *16) compound subject. *17)
compound predicate. *18) compound *19) .. *20)
compound subject and compound predicate. *21) compound
subject. *22) compound predicate. *23) compound predicate.
*24) compound predicate. * 25) compound predicate.

Exercise 15B:

Le Th Thuy Loan Trang 192


Phng Phap Dch VIET ANH - NG PHAP

The book is long and requires careful reading.


The band uniforms have been delivered and are liked by
everyone.
The telegram was delivered this morning but was put in the
wrong basket.
Pliteness costs nothing but often wins everything
We were delayed by a flat tire and missed the slow.
Heavy rains flooded the streets and damaged many homes.
The elevator operator started the elevator and called the
police.
The astronaut waved to the reporters and entered the
spaceship.
The word demoncracy comes from the Greek and means
government by the people.
Suddenly Jerry heard a noise and hid behind a tree.

Exercise 15C:
*1) Simple. *2) Compound. *3) Simple. *4) simple. *5)
Complex. *6) Simple. *7) Compound-complex. *8) Simple. *9)
Complex. *10) Complex. *11) Compoun-complex.
*12)Compound. *13) Compound. *14) Simple. *15) Complex.
*16) Complex. *17) Compound-complex. *17) Compound-
complex. *18) Simple. *19) Complex. *20) Complex. *21) Simple.
*22) Complex. *23) Simple. *24) Simple. *25) Compound. *26)
Simple. *27) Complex. *28) Compound. *29) Simple. *30)
Complex.

MUC LUC

Li m au
Phng phap
The Seven Basic Sentence Patterns
(07 Mau Cau C Ban)
The single Word Modifier of Nouns
(Yeu - to - mo - ta)

Le Th Thuy Loan Trang 193


Phng Phap Dch VIET ANH - NG PHAP

Modifiers: Word Order


(V tr cac Yeu - to - mo - ta)
The P-group (The Prepositional Phrase)
Verbals
The V-ing group
The V-ed group
The To-group
The construction For x (pro)noun x To-infinitive
(Cau truc phoi hp)
The Conjunction
(Lien t)
The S-group (The Subordinate Clause)
The Rule of Proximity
(Qui tac tng can)
The Rule of Parallelism
(Qui tac song hanh)
Phrase, Clause, Sentence
Kinds of Sentences
Some Special Expressions
(Vai hnh thai dien ta ac biet)
The Method of Diagraming Sentences
(Phng phap hnh o)
Mastery Test
Phan giai bai tap

Le Th Thuy Loan Trang 194

Anda mungkin juga menyukai